Die Feld-Interpretation hat den Vorteil, dass Gravitonen jetzt nicht mehr etwas deutlich anderes zu sein scheinen als zum Beispiel Photonen – Photonen sind die Teilchen des em-Feldes, Gravitonen die des Schwerefeldes. Gedanken darüber, wie die Raumzeitkrümmung aus “Teilchen” bestehen kann, braucht man sich in diesem Bild nicht zu machen.

Aber wie gesagt, die Feld-Interpretation ist nur eine andere Interpretation der Gleichungen der ART – alles, was man sich in dieser Interpretation überlegen kann, gilt auch, wenn man beim Bild der Raumzeitkrümmung bleibt. Und ich habe oben ja versucht, einigermaßen plausibel zu machen, in welchem Sinne man sich vorstellen kann, dass die “Raumzeitkrümmung” aus Teilchen bestehen kann.

 Nachweis von Gravitonen

Zum Abschluss stellt sich natürlich noch die Frage, wie man Gravitonen denn nachweisen soll, wenn es sie denn – laut Quantenmechanik – geben muss. Das ist leider extrem schwierig. Der Grund dafür ist, dass die Energie, die in Gravitationswellen steckt, sehr gering ist. Laut Wikipedia erzeugt zum Beispiel der Umlauf der Erde um die Sonne Gravitationswellen mit einer Energie von 300 Watt – die gesamte Energie der von Erde und Sonne abgestrahlten Gravitationswellen reicht also gerade mal, um ne Mikrowelle im Auftau-Modus zu betreiben. Selbst der Nachweis von Gravitationswellen ist also schon schwierig.

Noch viel schwieriger ist es, Gravitonen nachzuweisen. Das liegt daran, dass die Energie eines Gravitons – genau wie die eines Photones – von der Frequenz der Welle abhängt. Und weil sich zur Erzeugung von messbaren Gravitationswellen große Massen bewegen müssen, liegen die Frequenzen, um die es hier geht, im Bereich von Sekunden (wenn zum Beispiel zwei Neutronensterne sich umkreisen) oder Jahren (wie bei Erde und Sonne). Die Energie, die ein Graviton enthält, das von der Bewegung der Erde um die Sonne abgesandt wird, sollte demnach etwa 20 Trilliarden (eine 2 mit 22 Nullen) mal kleiner sein als die eines Photons des sichtbaren Lichts, wenn ich mich nicht verrechnet habe. (Und selbst bei zwei Neutronensternen, die sich in einer hundertstel Sekunde umkreisen, ist der Faktor noch eine Billion.) Ein Detektor, der solche winzigen Energiemengen aufnehmen soll, müsste also unglaublich empfindlich sein. (Deswegen waren die Physikerinnen ja so begeistert, als es letztes Jahr schien, dass man in der kosmischen Hintergrundstrahlung Spuren von Gravitonen entdeckt hätte – leider hat sich das aber ja als Irrtum herausgestellt.)

Fazit

Die Raumzeitkrümmung kann Energie und Impuls tragen – nach den Regeln der Quantenmechanik sind diese Größen aber quantisiert und können zum Beispiel aus einer Gravitationswelle nur in Paketen bestimmter Größe entnommen werden. Diese “Pakete” verhalten sichdeshalb so, wie es auch andere Teilchen (zum Beispiel Photonen) in der Quantenmechanik tun. Deswegen kann es sinnvoll sein, von “Gravitonen” zu sprechen, den “Teilchen der Raumzeitkrümmung”.

Wie die Quantengravitation der Gravitonen funktioniert (oder eben nicht) und wo die Schwierigkeiten dabei stecken, das verschiebe ich auf ein anderes Mal.

Anmerkung: Wie so oft in letzter Zeit stehen die Dinge, die ich hier erklärt habe, so in keinem Buch (warum, weiß ich nicht). Ich denke, ich habe mir alles richtig zusammengereimt, übernehme aber keine Garantie, falls ihr euch in eurer Physikprüfung auf meinen Blog beruft.Ach ja, ebenfalls wie so oft in letzter Zeit verwende ich im Text ein generisches Femininum – klickt hier, wenn ihr wissen wollt, warum (und falls ihr etwas zum diesem Thema sagen wollt, hinterlasst den Kommentar bitte auch dort, nicht hier.)

1 / 2 / 3 / 4 / 5

Kommentare (212)

  1. #1 rolak
    12. April 2015

    ein kleines Stückchen Raumzeitkrümmung?

    Selbst ohne den guten Text hintendran hätte sich das Lesen wg dieser Formulierung gelohnt 😉

  2. #2 MX
    12. April 2015

    @ rolak: Vermutlich ein Raumzeitkrümmel.

  3. #3 Volker Birk
    https://blog.fdik.org
    12. April 2015

    Ich hab da mal eine Frage – vielleicht eine naive, weil ich kein Physiker bin:

    Soweit ich die Sache verstehe, gibt es doch mehr als eine Möglichkeit, ein und dieselbe Sache mathematisch korrekt zu beschreiben. Was wäre denn, wenn in einem solcher Modelle die Gravitation eine Scheinkraft ist, weil sie als Raum(zeit)-Krümmung gerechnet wird, in einem anderen Modell, das ansonsten äquivalent ist, jedoch ein Teilchen? in einem solchen (hypothetischen) anderen Modell müsste der Vektorraum dann anders aufgespannt sein und nicht als Raumzeit.

    Wäre so etwas nicht denkbar? Und falls es denkbar ist: wäre das eine Richtung, in der ein Graviton zu suchen wäre?

  4. #4 MartinB
    12. April 2015

    @Volker
    “gibt es doch mehr als eine Möglichkeit, ein und dieselbe Sache mathematisch korrekt zu beschreiben.”
    Das ist ja letztlich das, was hie rpassiert – wir können die Gleichungen in zwei unterschiedlichen Arten schreiben – im dritten teil der “Gibt es Raumkrümung”-Serie habe ich das ja ein bisschen erklärt.
    Es geht aber nicht darum, das Graviton in der Theorie zu suchen – wir wissen ja, wie es aussehen und welche Eigenschaften es haben muss. Das dumme ist nur, dass die Theorie, wenn man sie zu Ende rechnet, Blödsinn auswirft, aber dazu schreibe ich hoffentlich demnächst was.
    “in einem solchen (hypothetischen) anderen Modell müsste der Vektorraum dann anders aufgespannt sein und nicht als Raumzeit.”
    Im Feldbild ist die raumzeit einfach die Standard- Minkowski-Raumzeit der SRT – nur dass man das nie direkt beobachten kann, weil das Feld alle Maßstäbe entsprechend verzerrt.

  5. #5 Alderamin
    13. April 2015

    @Martin

    Danke, sehr anschaulich.

    Kann man auch begründen, warum das Graviton ein Spin-2-Teilchen sein soll? Das muss wohl damit zusammenhängen, dass es nur einen Gravitations”pol” gibt, während es deren zweie beim Elektromagnetismus gibt, aber wie das mit dem Spin des Vermittlerteilchens zusammenhängt, ist mir nicht klar.

  6. #6 Andreas Müller
    13. April 2015

    Hallo Martin

    Was ich schmerzlich vermisst habe in Deinem Artikel über Gravitonen ist ein Hinweis auf ihren Spin 2 (was sie von den Photonen mit Spin 1 unterscheidet) sowie auf Stringtheorien (in denen Gravitonen geschlossene Strings sind) und die die Loop-Quantengravitation (in denen Gravitonen die “Raumzeit-Atome” einer körnigen Raumzeit wären). Hier könnte man noch einiges dazu sagen.

    Beste Grüße,
    Andreas

  7. #7 MartinB
    13. April 2015

    @Alderamin und Andreas
    Das mit dem Spin-2 kommt ja noch, keine Sorge. Letztlich sieht man es schon am Bild der Welle – die em-Welle wird ins ich selbst überführt, wenn ich sie um 360° drehe, die Grav-Welle, wenn ich sie um 180° drehe. (Bei Spin 1/2muss man um 720° drehen…)

    @Andreas
    “sowie auf Stringtheorien (in denen Gravitonen geschlossene Strings sind) und die die Loop-Quantengravitation ”
    Da kenne ich mich nicht so gut aus, deswegen werde ich dazu nichts schreiben. Aber über die QFT eines Spin-2-Teiclhens und das,w as man daraus über QG sagen kann (und warum das nicht funktioniert) wird hoffentlich demnächst was kommen.

  8. #8 Alderamin
    13. April 2015

    @Andreas Müller

    Hier könnte man noch einiges dazu sagen.

    Dann sag’ Du doch einfach was dazu. 😉 Bitte, bitte. 😉 😉 Hast ja selbst einen Blog 😉 😉 😉

  9. #9 Realistischer
    14. April 2015

    Diese Raumzeitkrümmung scheint mir doch ein sehr krummes Konzept zu sein. Wenn ein Asteroid an einem Planeten vorbei fliegt und gemäß Newton von der Schwerkraft des Planeten abgelenkt wird, dann bewegt er sich gemäß Raumzeitkrümmung aber geradeaus weiter, nur halt im gekrümmten Raum. Dieses “gekrümmte Geradeaus” wirkt aber beliebig weit – so weit wie der Komet kommt – d.h. der ablenkende Planet kürmt auch den Raum in beliebiger Entfernung noch … aber nicht den ganzen Raum sondern zufällig nur den, wo der Komet gard’ so unterwegs ist. Ich bin ja der Meinung: wenn was so magisch kompliziert ist, dann ist es einfach nur fehlerhaft. Einstein hin,”Konsens” her, wer sich von Vorgaben beeindrucken lässt und verzweifelt versucht zu verstehen was aber sichtlich unverständlich ist, der handelt wie ein religiöser Gläubiger… Also, lasst euch nicht abbringen vom rechten Weg, ihr braven Gefolgsleute!

  10. #10 MartinB
    14. April 2015

    @Realistischer
    “aber nicht den ganzen Raum sondern zufällig nur den, wo der Komet gard’ so unterwegs ist. ”
    Blödsinn – der ganze Raum wird gekrümmt, da wo der Komet nicht ist, merkt er nur nichts davon.
    “was aber sichtlich unverständlich ist,”
    Genereller Tipp: “Für mich unverständlich” ist nicht dasselbe wie “Für alle Menschen unverständlich”.

  11. #11 Andreas Müller
    14. April 2015

    @Alderamin

    Leider fehlt mir die Zeit einen Artikel in meinem Blog zu schreiben. Das Graviton ist schon ein Thema, bei dem man sehr weit ausholen muss und viele Grundlagen aus Relativitätstheorie und Quantenphysik erklären muss. Man müsste etwas zu Renormierbarkeit und Feynman-Diagrammen sagen etc. Hinzu kommt alles, was aktuell in den Ansätzen zu Quantengravitationstheorien steckt.

    Ich hatte mal etwas zum Graviton in meinem Web-Lexikon geschrieben: https://www.wissenschaft-online.de/astrowissen/lexdt_g05.html#g

    Das bleibt für einen Laien sicherlich unverständlich, wie ich zugeben muss und ist sehr knapp. Es macht Sinn, von dort die Links auf Eichtheorie, Stringtheorie und Loop-Quantengravitation zu folgen.

    Der Eintrag “Graviton” in der dt. Wikipedia ist mal ein Einstieg, aber recht mager. Gut gefallen hat mir das Pendant in der engl. Wikipedia: https://en.wikipedia.org/wiki/Graviton

    Beste Grüße, Andreas

  12. #12 MartinB
    14. April 2015

    @Andreas
    Danke für den Link, da werd ich sicher noch mal schauen.
    Dass mit dem massiven Graviton habe ich nicht verstanden – wenn man das erwartet, wieso schreibst du dann
    “Mit diesen Ergebnissen ist jedoch die verschwindende Ruhemasse noch nicht ausgeschlossen! ”
    als wäre das irgendwie erstaunlich?

  13. #13 Andreas Müller
    14. April 2015

    Lieber Martin

    Erwarten würde man eine Ruhemasse von null für die Gravitonen (also wie bei Photonen). Das würde auch dazu passen, dass sich das klassische Analog zu den Gravitonen, nämlich die Gravitationswellen, mit Lichtgeschwindigkeit bewegen. Das wiederum ist ja ein Ergebnis der Linearisierung von Einsteins Feldgleichung der ART.

    Experimentell ist die Maximalmasse eines Gravitons zwar zu einem extrem kleinen Wert bestimmt worden, aber das ist noch kein überzeugender Beleg für die Ruhemasse null – nur ein starkes Indiz.

    Übrigens, wenn es darum geht die Methodik von Yang-Mills-Theorien (QCD) und dem Higgs-Feld auf die Gravitation zu übertragen, dann ist es lohnend sich anzuschauen, wie man eine Eichtheorie der Gravitation bauen kann. Dazu:
    https://en.wikipedia.org/wiki/Gauge_gravitation_theory

    (leider auch hartes Brot…)

    Beste Grüße,
    Andreas

  14. #14 MartinB
    14. April 2015

    @Andreas
    Ja, so habe ich es auch gedacht – der Satz mit dem Ausrufezeichen las sich angesichts dessen aber etwas verwirrend, ich hätte eher erwartet, dass du sagen willst, eine nicht verschwindende Ruhemasse sei nicht ausgeschlossen.

  15. #15 rolak
    14. April 2015

    Raumzeitkrümmel

    🙂 thx, MX.

  16. #16 Realistischer
    14. April 2015

    @MartinB
    “Blödsinn – der ganze Raum wird gekrümmt, da wo der Komet nicht ist, merkt er nur nichts davon.”
    Der Komet merkt nichts davon — und alle anderen Objekte, die dort sind, merken auch nichts davon – es sei denn die wären auch alle am Planeten vorbei gekommen und entsprechend abgelenkt worden.
    Die Raumzeitkrümmung ist ein in sich widersprüchliches Konzept das nur funktionieren würde wenn jedes Objekt seine eigene Raumzeit hätte. Was in Folge die Frage aufwerfen würde, wessen Raumzeitkrümmungs-Gravitonen denn Geltung hätten.
    Ergo kann ein Mensch die Raumzeitkrümmung wirklich verstehen, ausgenommen jene die verstehen dass das ein grober Unfug ist. Man braucht ja nur die Erklärungs*versuche* zu lesen um zu sehen dass keine Nachvollziehbarkeit gegeben ist.

  17. #17 Andreas Müller
    14. April 2015

    @Realistischer

    Also, das möchte ich als jemand, der sich intensiv mit der (Speziellen und Allgemeinen) Relativitätstheorie beschäftigt nicht unwidersprochen stehen lassen.

    Lassen wir bitte die Gravitonen beiseite und reden wir nur vom Konzept der gekrümmten Raumzeit.

    Natürlich spürt ein Testteilchen (damit ist ein Teilchen vernachlässigbarer Masse gemeint) die gekrümmte Raumzeit einer anderen, größeren Masse, z.B. eines Planeten oder der Sonne. Einsteins Allgemeine Relativitätstheorie beschreibt in Gestalt der Feldgleichung alle diese Gravitationsphänomene.
    So beschreibt die Schwarzschild-Raumzeit (eine ganz spezielle Lösung der Feldgleichung) relativistisch die Gravitation einer Punktmasse. Die Krümmung der Raumzeit nimmt ab, je mehr man sich von der Punktmasse entfernt – in Übereinstimmung mit dem Abstandsquadratgesetz im klassischen Bild der Newtonischen Gravitation. Der Relativist sagt das nur anders in differentialgeometrischer Redeweise: eine Raumzeit ist asymptotisch flach.

    Die Bewegung eines Testteilchen oder auch eines Lichtteilchens folgt aus der Berechnung der Geodätengleichung für die Schwarzschild-Raumzeit. Auch diese Methode ist die konsequente Verallgemeinerung der Newton-Gleichung, wenn man Gravitation differentialgeometrisch als gekrümmte Raumzeit beschreibt. Dazu übernahm Einstein die mathematischen Erkenntnisse der Differentialgeometer (z.B. Levi-Civita-Zusammenhang, Christoffel-Symbol).

    Die Berechnungen mit Einsteins Allgemeiner Relativitätstheorie stimmen im Grenzfall schwacher Gravitation mit den Newtonschen Ergebnissen überein.
    Das historisch erste Phänomen, wo Newtons Theorie versagte, aber Einsteins Theorie die Natur richtig und in glänzender Übereinstimmung mit der Beobachtung beschrieb, war die besondere Bahnbewegung des innersten Planeten Merkur (Periheldrehung). Danach folgte die ebenso glänzend passende Vorhersage der Lichtablenkung an der Sonnenmasse, bestätigt mit der Sonnenfinsternis von 1919.
    Ich könnte viele weitere erfolgreiche Tests nennen. Die Beweislast, die für das Konzept der gekrümmten Raumzeit spricht, ist einfach erdrückend.

    Einstein war damals bei der Veröffentlichung der Allgemeinen Relativitätstheorie (sie wird im November 2015 genau 100 Jahre alt) natürlich ebenfalls auf Widerstände gestoßen. Die Relativitätstheorie birgt einige Phänomene, die uns vollkommen ungewohnt, ja, sogar unglaublich erscheinen: Relativität von Zeit und Länge, ein neues Additionstheorem für Geschwindigkeit, Lichtablenkung an Massen, rotierender bzw. expandierender Raum etc.

    So unglaublich uns eine physikalische Theorie erschienen mag: Am Ende des Tages muss sie sich an der Natur messen lassen. Beschreibt die Theorie beobachtete Phänomene qualitativ UND quantitativ richtig? Macht Sie Vorhersagen für neue Phänomene, die auch beobachtet werden (können)? Beinhaltet die Theorie im Grenzfall althergebrachte, bewährte Theorien? Besteht die Theorie immer und immer wieder den Test mit Experimenten bzw. mit Beobachtungen?

    Diese Fragen müssen wir ALLE für die Relativitätstheorie mit Ja beantworten. Sie lässt sich seit Jahrzehnten immer wieder testen und besteht glänzend die Tests. Zur Beschreibung der Gravitation und der unbelebten Natur bei hohen Geschwindigkeiten (vergleichbar der Lichtgeschwindigkeit) ist die Relativitätstheorie einfach das Beste, was wir haben. Im Sinne der Popperschen Wissenschaftstheorie ist es eine bewährte Theorie.

    Selbstverständlich ist damit nicht ausgeschlossen, dass es eine der Relativitätstheorie übergeordnete Theorie gibt. Sie könnte dem quantenhaften Charakter der Natur Rechnung tragen und auch die Gravitonen beinhalten. Aber so weit ist die moderne Forschung noch nicht. Es gibt moderne Ansätze, um über die Gravitation zu quantisieren, aber diese Theorien haben sich im Gegensatz zur Relativitätstheorie nicht bewährt. Entweder sie bestehen die anstehenden Tests oder wir müssen akzeptieren, dass die Gravitation unquantisiert bleibt.

    Beste Grüße,
    Andreas

  18. #18 MartinB
    14. April 2015

    @Realistischer
    “Die Raumzeitkrümmung ist ein in sich widersprüchliches Konzept”
    Nein. Dass du es nicht verstehst, ist schade, aber hör bitte auf, hier dein Unverständnis so zu präsentieren, als würdest du von tatsachen reden.

  19. #19 Herr Senf
    14. April 2015

    Ich sehe das Problem eher im alltagssprachlichen Verständnis von Fachbegriffen.
    Wenn dort “krumm” steht, will man “krumm” sehen oder sich vorstellen.
    Geht schlecht, weil wir im letzten Moment gerade sehen, die “scheinbare” Richtung.
    Krümmung ist ein “Mathe-Konzept”, das in der Kosmologie ausgerechnet werden muß,
    aus direkten Beobachtungen, deren “Verzerrung” wir erkennen müssen.
    Wir haben noch eine Baustelle in Raumzeit III hhttps://scienceblogs.de/hier-wohnen-drachen/2015/03/21/ist-die-raumzeit-gekruemmt-teil-iii-die-zeit/#comment-23193
    Da Dr. Müller hier mitmacht, vielleicht hat er Rat aus erster Hand?
    Grüße Senf

  20. #20 Realistischer
    14. April 2015

    @MartinB
    Ich habe aufgehört, unverständliches Zeug als “Wissen” zu akzeptieren. Entweder etwas ist nachvollziehbar, oder nicht. Und zwar *mir* nachvollziehbar – denn andernfalls ist es kein Wissen sondern nur ein Glaube. Dass ihr mit euren Erklärungen scheitert – und zwar nicht an mir sondern an euch selber, wie oben im Artikel auch nachzulesen ist -, ist mir Beweis, dass ihr es selbst auch nicht wirklich verstanden habt.

  21. #21 Andreas Müller
    14. April 2015

    @Realistischer
    Das, was ich hier gesagt habe, ist alles nachvollziehbar und auch aufgeschrieben worden. Ich mache nur die Erfahrung, dass nicht jeder bereit ist, Zeit und Mühe zu investieren, um es selbst zu verstehen.

    Das muss ja auch nicht jeder. Ich finde es nur recht schwach, hier fälschlich zu behaupten, dass das Konzept der Raumzeit widersprüchlich sei, ohne das belegen zu können und ohne den Versuch zu machen, es selbst besser zu verstehen.

    Beste Grüße,
    Andreas

  22. #22 Krypto
    15. April 2015

    @Andreas, Martin:
    Don´t feed the troll 😉
    Der Typ hat genau wie A. Unzicker eine Art Schutzschild, welcher alles, was er nicht versteht, automatisch als falsch definiert und abprallen lässt.
    Das ist äußerst praktisch für´s Selbstwertgefühl, weil man zugeben muss, etwas nicht zu verstehen.

  23. #23 Krypto
    15. April 2015

    @myself: “nicht zugeben muss”
    blöde doppelte Verneinung 🙂

  24. #24 MartinB
    15. April 2015

    @Krypto
    Meist sage ich Leuten auf meinem Blog zumindest, wenn ich sie nicht mehr ernst nehme und wenn ihre Kommentare unerwünscht sind. “Don’t feed the troll” ist auch nicht unbedingt immer ne gute Politik (hier allerdings schon, da hast du recht).

  25. #25 Benjamin Packisch
    15. April 2015

    @Andreas Müller:
    Als Leser Ihre Blogs wusste ich gar nicht um ihre intensive Beschäftigung mit SRT und ART. Das freut mich

    Was ich zu dem schönen und dankenswert aufrichtigen Artikel von MartinB gerne hinzufügen möchte, ist die Warnung mathematische Konzepte mit der Realität zu sehr zu identifizieren.
    Mathematische Konzepte sind wunderbar, um (empirische) Tests zu definieren und auf Selbstkonsistenz zu prüfen – wie es bei SRT und ART ja häufig und meistens redlich&erfolgreich geschieht. Daraus jedoch formale Alternativlosigkeit zu folgern ist logisch irrig und es führt immer wieder zu häßlichen Eskalationen in Diskussionen.
    Und das gilt für jeglich Theorie. Ontische (also das Sein betreffende) Aussagen sind weder Ziel noch leistbar in kausal-empirischen Theorien.
    Ontisch sind Theorien also niemals beweisbar, wohl aber empirisch und pragmatisch, d.h. die (technische) Arbeit mit den physikalischen Phänomenen betreffend.
    Seins-Aussagen sind trotzdem möglich, aber sie bleiben immer der mathematisch-logischen Relationssphäre zugeordnet.
    Der Bezug einer mathematischen Form zur Physik unterliegt aber prinzipiell der Deutungsfreiheit, auch und vor allem in letzter Konsequenz.

    Diesen Punkt sauber zu differenzieren erspart viel Elend.

  26. #26 MartinB
    15. April 2015

    @Benjamin
    Dazu habe ich mich ja (unter anderem, siehe den tag Wissenschaftsphilosophie) hier detailliert geäußert:
    https://scienceblogs.de/hier-wohnen-drachen/2011/08/15/ist-die-natur-mathematisch/
    https://scienceblogs.de/hier-wohnen-drachen/2010/08/31/kann-die-physik-die-welt-erklaren/

  27. #27 Benjamin Packisch
    15. April 2015

    @MartinB
    Vielen Dank für die Verweise auf die detaillierte Diskussion. Sie enthalten viele wesentliche Punkte, aber ich habe den Eindruck, dass dennoch ein fundamentaler Punkt der vollen Aufklärung harrt: Ich zitiere aus “Ist die Natur mathematisch?”: “Die Kategorisierungen, die die Basis für das Zählen und Messen bilden, erweisen sich nämlich bei näherer Überprüfung als der realen Welt angemessen.”
    Das stimmt, ABER: Diese Angemessenheit liegt an der- oder demjenigen, der das Zählen oder Messen betreibt. Und das gilt eben nicht nur für die von Ihnen m.E. zu Unrecht so sehr geschmähte Goethe-Farbtheorie*, sondern für alle Theorien. “Angemessen” ist die mathematische Weltbeschreibung also nur relativ zu einer spezifschen Interaktionspraxis, nämlich derjenigen der apparativen Einflussnahme und dessen linearer Optimierung. Erkennbar wird diese häufig stillscheigende Voraussetzung im Widerwillen gegen alternative Formalismen, sogar wenn sie numerisch analog sind.
    Ein provokantes Beispiel dafür:
    Wie wäre es, wenn Sie die Goethe’sche Farbtheorie einmal im Lichte quantenoptischer Interferenz betrachteten? Also statt einer metaphysischen (oder gefälliger: proto-physischen) Wellenfunktion eine Finsterniswelle annähmen, durch deren Intereferenz Farben entstehen?
    Ich garantiere Ihnen: selbst wenn Sie perfekte Identität mit quantenoptischer Numerik erzielten würde man Ihnen nicht glauben. Oder sind sie anderer Meinung ;-)?

  28. #28 Benjamin Packisch
    15. April 2015

    EDIT: hinter “Finsterniswelle” bitte eingefügt denken:
    “…eine Finsterniswelle annähmen, deren Intereferenz mit einer Lichtwelle Farben erzeugt? “

  29. #29 MartinB
    15. April 2015

    @Benjamin
    ““Angemessen” ist die mathematische Weltbeschreibung also nur relativ zu einer spezifschen Interaktionspraxis, ”
    Die Tauglichkeit der Praxis erweist sich dann in der korrekten Vorhersage von Experimenten.

    Und ich glaube kaum, dass man mit einer Finsterniswelle dieselben Vorhersagen machen kann wie mit den Maxwell-Gleichungen (ansonsten wäre die Finsterniswelle eine zulässige, weil äquivalente, Beschreibung).

    Für einen eher kritischen Blick auf das Problem empfehle ich noch das hier:
    https://scienceblogs.de/hier-wohnen-drachen/2010/12/10/physik-und-geist-dialog-uber-zwei-weltsysteme/

  30. #30 Benjamin Packisch
    15. April 2015

    Vorhersagen betreffen die Selbstkonsistenz eines Annahmensystems, nicht den Kontakt verschiedender Systeme oder gar die Allgemeingültigkeit eines Annahmensystems. Out of the box geht es so noch nicht. leider.

    Sehr ehrlich von Ihnen aber finde ich es, dass sie schreiben, nicht an die Möglichkeit einer Beschreibung qua Finsterniswelle “glauben”. Darf ich das als willentliche Ablehnung dieser Möglichkeit ohne Prüfung deuten?

    Vielen Dank für Ihren Dialog außerdem. Auch wenn ich finde, dass der Schamane ein zu harmloser Gegner für Rogant ist. Gegen Wissenschaftler sollte man Logiker antreten lassen, dann erst wird es wirklich berührend bzw. spannend. Warum z.B. können Zwecke nicht gesetzlich geordnet sein?
    Vielleicht ist das hier eine Anregung, über den Tellerrand der Metaphysik-Entsagung zu schauen ;-):
    https://www.heise.de/tp/artikel/43/43546/1.html ?

  31. #31 MartinB
    15. April 2015

    @Benjamin
    “Darf ich das als willentliche Ablehnung dieser Möglichkeit ohne Prüfung deuten? ”
    Nein, das darfst du so deuten, dass ich es für extrem unwahrscheinlich halte, dass man die gesamte komplexität der Optik mit einem vollkommen anderen Beschreibungssystem genau so korrekt darstellen könnte. “Glaube” schreibe ich, weil ich es nicht nachgewiesen habe.

    Mit dem Dialog kann ich schon nach kurzer zeit nichts anfangen – Sätze wie “Doch haftet jeder echten geistigen Stärkung zugleich auch ein für mich unverzichtbarer Wahrheitsanspruch an.” lösen bei mir nur ein Achselzucken aus, weil ich keine Ahnung habe, was mir die Autorin damit sagen will. (Von Trivialitäten wie “Mein Glaube ist wahr, weil er richtig ist” ganz abgesehen…)

  32. #32 Realistischer
    15. April 2015

    @Andreas Müller
    Ich habe es ja bereits erklärt. Versuchen SIE doch mal’ das Beispiel des abgelenkten Kometen mittels Raumkrümmung darzustellen … und berücksichten SIE dabei dass der abgelenkte Komet sich nicht allein im Raum bewegt sondern im selben Raum auch andere, nicht so wie der Komet abgelenkte Objekte gegeben sind … und versuchen SIE das konsistent darzustellen und zu erklären ohne Widersprüche und Lücken. Dazu bräuchten SIE aber ein Verständnis das darüber hinaus geht, von dieser oder jener Quelle ein Zitat herbei zu holen. Ich wette, SIE schaffen das nicht.

  33. #33 MartinB
    15. April 2015

    @Realistischer
    Die Rechnung kannst du in jedem Buch zur ART nachlesen.

  34. #34 Andreas Müller
    15. April 2015

    @Realistischer
    Mein Kommentar #17 enthielt genau diese Erklärung. Hatten Sie den gelesen? In Kurz- und hoffentlich verständlicher Form: Die Bewegung eines Kometen unter dem Einfluss eines nahen Planeten kann man mit der Schwarzschild-Lösung beschreiben. Der Komet als massebehaftetes Teilchen bewegt sich auf einer zeitartigen Geodäte. Welche das ist, berechnet man durch Lösung der entsprechenden relativistischen Bewegungsgleichung. Das ist die Geodätengleichung. Man muss für die Berechnung die Raumzeit festlegen. Das ist die Schwarzschild-Metrik.

    Diese Berechnung ist ein Zentralkraftproblem, weil die Masse des Planeten viel größer ist, als die des Kometen. Die Schwarzschild-Raumzeit ist dann eine Hintergrund-Metrik, die selbst durch den kleinen Kometen nicht verformt wird. Natürlich krümmt auch der Komet die Raumzeit, aber das kann vernachlässigt werden, falls seine Masse viel kleiner ist.

    Wenn Sie zwei Körper vergleichbarer Massen betrachten wird es komplizierter, aber es kann immer noch mit der Einsteinschen Feldgleichung gelöst werden. Dann muss man sie numerisch lösen und kann nicht auf eine analytische Lösung der Feldgleichung, wie die Schwarzschild-Lösung eine ist, zurückgreifen.

    Beste Grüße,
    Andreas Müller

  35. #35 Adent
    16. April 2015

    @Realistischer

    Entweder etwas ist nachvollziehbar, oder nicht. Und zwar *mir* nachvollziehbar – denn andernfalls ist es kein Wissen sondern nur ein Glaube.

    Ernsthaft jetzt?
    Es gibt also nur dann Wissen, wenn Sie es verstehen?
    Nunja, das ist wirklich Realsatire, vielleicht denken Sie darüber noch einmal nach und formulieren es ein bischen anders.

  36. #36 MartinB
    16. April 2015

    @Adent
    Protagoras + Solipsismus = Realistischer

  37. #37 Realistischer
    16. April 2015

    @Adent
    *Mein* Wissen muss *mir* nachvollziehbar sein. Aber wenn Sie einfach alles *glauben* von dem andere sagen dass es ein “Wissen” sei, dann sind Sie ein Gläubiger. Auch dann, wenn Sie meinen dass Sie all das “Wissen” an das Sie glauben nun auch selber wissen. Den Unterschied verstehen Sie aber offensichtlich nicht – das erklärt eigentlich alles.

    @Andreas Müller
    Also wird der Komet vom Planeten ***abgelenkt*** und setzt nachher seine Bewegung in der abgelenkten Richtung fort. Dass Sie an diese fortgesetzte Bewegung ausserhalb des Einflussbereichs des Planeten garnicht gedacht haben, zeigt, dass Sie mein Beispiel nicht verstanden haben.
    Also nochmal: der Komet hat eine Bewegungsrichtung, nähert sich damit dem Planeten an, und wird ***abgelenkt***, was sich insbesondere darin zeigt dass die weitere Bewegung – auch nach Verlassen des Einflussbereichs des Planeten – in der abgelenkten Richtung erfolgt.
    Ohne die Ablenkung ist das nicht verstehbar, ausser wenn man den Betrachtungsfokus so weit eingrenzt dass es nicht mehr relevant ist. Abstraktion als Reduktion auf das, was leicht berechenbar ist – man könnte das auch Dummheit nennen.

  38. #38 Adent
    16. April 2015

    @Realistischer
    Ja genau verdrehen Sie nur weiter Aussagen von mir, was anderes können Sie offensichtlich nicht.
    Ich glaube nicht an Wissen, das ist eine dreiste Unterstellung von Ihnen, mehr nicht. Sie haben sich zunächst mal falsch ausgedrückt und behauptet:

    Entweder etwas ist nachvollziehbar, oder nicht. Und zwar *mir* nachvollziehbar – denn andernfalls ist es kein Wissen sondern nur ein Glaube.

    Steht so da oben, wirklich, haben Sie geschrieben, dann schnell den Rückzieher machen durch:

    *Mein* Wissen muss *mir* nachvollziehbar

    sein.
    und dann mir falsche Aussagen unterstellen, das ist genau ihre Art zu diskutieren, daher gehaben Sie sich wohl in ihrer verqueren Realität.

  39. #39 Benjamin Packisch
    16. April 2015

    @MartinB
    Vielen Dank wiederum für Ihre Ehrlichkeit. Wieviel nur könnten wir gewinnen, wenn es gelänge durch Oberfläche sprachlicher Gewohnheit zu dringen! Zum Beispiel darin, die Verwandtschaft von Aussagen wie
    -“Ich halte es für extrem unwahrscheinlich, dass XY.”
    _”Ich will, dass XY.”
    -“Ich glaube, dass XY.”
    zu erkennen!
    Es sind m.E. wirklich nur Nuancen, die hier so bedauerliche Gräben aufwerfen.
    Sie als Simulationsexperte (ich übrigens auch), dürften doch bestens um die Vieldeutigkeit von Numerik und formaler Beschreibung wissen!

    Ob ich Farbe als Wellenlänge beschreibe, die sich aus Maxwell’schen und quantenmechanischen Interferenzen aufbaut, oder als komplementäre Verkopplung zweier Strahlqualitäten, ist doch letztlich Geschmacksache – wenn man sich von den verbreiteten Üblichkeiten nicht irritieren lässt.

    Wohl mag es relevant auf der Ebene der Kommunikation sein.
    Aber worin sehen Sie den Nutzen, gerade hier “bei den Drachen” so sehr auf die Alleingültigkeit der Physik für ernsthaften Naturbeschreibungen zu halten?
    Auch ART und Graviton sind formal-numerische Wahlmöglichkeiten, aber keine alternativlosen Universalien.

    Wie gesagt, auf der didaktischen Ebene mag die Mono-Deutung hilfreich sein, aber die physikalische Wissenschaft lebt doch gerade davon, keinen Deutungsdogmatismus zu betreiben, und schon gar nicht in der Numerik.

    Ich freue mich über jedes Argument in dieser Sache – pro und contra physikalische Deutungsfreiheit!

  40. #40 MartinB
    16. April 2015

    @Benjamin
    “Ob ich Farbe als Wellenlänge beschreibe, die sich aus Maxwell’schen und quantenmechanischen Interferenzen aufbaut, oder als komplementäre Verkopplung zweier Strahlqualitäten, ist doch letztlich Geschmacksache “Beides ist falsch. “Farbe” ist ein psycho-physiologisches Phänomen, kein physikalisches. (Ich weiß, war nur ein beispiel…)

    “Aber worin sehen Sie den Nutzen, gerade hier “bei den Drachen” so sehr auf die Alleingültigkeit der Physik für ernsthaften Naturbeschreibungen zu halten?
    Auch ART und Graviton sind formal-numerische Wahlmöglichkeiten, aber keine alternativlosen Universalien. ”
    Ich kenne keine Alternativen mit vergleichbarer Erklärungs- und Vorhersagekraft – wie anders als mit der ART soll man Schwerkraft beschreiben?

  41. #41 Benjamin Packisch
    16. April 2015

    @MartinB:
    Was ist denn nun ihre Antwort auf die Frage, außer, dass sie Farbe als unphysikalisch voraussetzen und keine Alternativen zu ART und Graviton kennen?
    Soll ich daraus ableiten, dass Sie, wenn Sie Alternativen kennen würden, sie auch beachten würden? Ich tue das mal vorläufig:
    Ich bemerke dabei eine “salvatorische Klausel” in ihrer Antwort: Sie erwarten von Alternativen “vergleichbare Erklärungs- und Vorhersagekraft”. Daher müssen wir erstmal klären, wie weitreichend Sie diese “Kraft” auffassen. Denn nur bzgl. numerischer Aussagen kann es wohl nicht sein, denn dass es numerische Alternativen gibt, steht ja außer Frage (1+1 = 3-1 = 2 = etc.).
    Wir müssen uns also bzgl. der Zielsetzung einer guten Theorie verständigen, über die mess- und prognosekonforme Numerik hinaus, korrekt?
    Welche Aspekte halten Sie da für wesentlich und welche für unbedingt vermeidbar?
    Und um beim Thema zu bleiben gerne mit Bezug zu Graviton und ART, i.O.?

  42. #42 MartinB
    16. April 2015

    @Benjamin
    “Was ist denn nun ihre Antwort auf die Frage”
    Ich verliere etwas den faden – auf welche konkrete Frage?

    Das mit der Numerik verstehe ich nicht – bei physikalischen Theorien geht es um die Vorhersage konkreter Experimente – da ist es egal, in welchen Einheiten oder mit welcher mathematisch äquivalenten Zahlenkombination ich die aufschreibe.

  43. #43 Benjamin Packisch
    16. April 2015

    A. Die konkrete Frage: Worin sehen Sie den Nutzen, gerade hier “bei den Drachen” so sehr auf die Alleingültigkeit der Physik für ernsthaften Naturbeschreibungen zu halten?

    B.Die Causa Numerik: Eine Messung ergibt den Wert x in irgendeinem Einheitensystem , unter Voraussetzung einer Menge von Experimentsparametern Px. Eine Theorie ist eine Abbildung T : Px->x. Und dieses T ist keineswegs eindeutig oder bijektiv. Schlimmer noch: Es gibt kein deutungsinvariantes Kalkül, die Px zu spezifizieren und abzugrenzen.
    M:E. folgt: Gründe für Theorie-Monismus können nicht innerhalb der Theorie liegen. Wo also dann? Konventionen? Wille/Glaube!? xy?
    Freue mich über jegliche Hinweise!

  44. #44 MartinB
    16. April 2015

    @Benjamin
    Machen wir es konkret: Welche andere (nach dem ersten Satz auch noch nichtphysikalische?) Beschreibung der Gravitation außer der ART leistet dieselben Vorhersagen bezüglich z.B. Periheldrehung, Lichtablenkung, der Periodenänderung von Neutronensternen usw?

  45. #45 Realistischer
    16. April 2015

    @Adent
    Sie regen sich sehr künstlich auf. *Mein* Wissen muss für *mich* nachvollziehbar sein. Etwas, das *mir* nicht nachvollziehbar ist, ist für *mich* kein Wissen. Das gilt für Sie auch, nur dass Sie das nicht zugeben können, denn dann würde sich zeigen dass sie eben doch ein Wissenschaftsgläubiger sind.
    Am Besten zeigt sich das Phänomen bei den Quantenphysikern, die sagen sogar selber dass sie es nicht verstehen. Wissen nennen’s das trotzdem. Das ist zwar pervers, aber sichtlich allen egal. Weil alle daran glauben. Nennen tun sie es Wissenschfaft, Wissenschaftsgläubigen genügt das – ihr merkt den Unterschied ohnehin nicht.

  46. #46 Benjamin Packisch
    16. April 2015

    @MartinB:
    Es gibt unbeschränkt viele andere, sogar wenn man nur die physikalischen betrachtet! Wenn ich die hier bislang unspezifizierten Voraussetzungen allerdings eng genug wähle, dann gibt es nur noch eine, aber das ist dann eben Funktion wilkürlicher Px und nicht logisch eindeutig.
    Das Problem der genannten astronomischen Beispiele ist die Vielzahl impliziter Vorannahmen, die keineswegs eindeutig ist, wie dargelegt.
    Wir müssen uns einigen auf die jeweils zugrundeliegenden Eingabedaten und den gewünschten Output, sonst bleibt das alles sehr im Beliebigen.
    Zur Lichtablenkung: Denken Sie, dass Sterne eine Atmosphäre haben, also einen Brechungsindex? Und Galaxien mit und ohne Leuchtkraft?
    Und kennen Sie den Ansatz, die Merkur-Periheldrehung per endlicher Schwerkraftsausbreitung zu berechnen?

  47. #47 Adent
    16. April 2015

    @Realistischer

    Wissenschaftsgläubigen genügt das – ihr merkt den Unterschied ohnehin nicht.

    Können Sie mal mit den Beleidigungen aufhören und Argumente bringen? Sie kennen mich überhaupt nicht und unterstellen mir Wissenschaftsgläubigkeit.
    Gut, das geht auch anders herum Sie sind schlicht ein Crank ohne Basis.

  48. #48 MartinB
    16. April 2015

    @Benjamin
    “Es gibt unbeschränkt viele andere”
    Name ten, wie man so sagt. Und natürlich nicht eine Spezialerklärung für jedes Phänomen, sondern eine umfassende Erklärung der Phänomene. (und nein, Lichtablenkung in Atmosphären erklärt z.B. keine Gravitationslinseneffekte.)

  49. #49 Benjamin Packisch
    16. April 2015

    @MartinB:
    “Ein umfassende Erklärung der Phänomene” fordern Sie. Also keine präzise Numerik, sondern …?
    Der Gravitationslinseneffekt wird natürlich nicht durch Lichtbrechung erklärt -aber es ist nunmal spekulativ, eine Gravitationslinse anzunehmen. Tut man das einmal, gibt es natürlich keine Alternativen mehr, auch nicht bezüglich der Frage, was denn dort eigentlich gravitiert. Ich könnte derweil auch gekrümmte Raumzeit für eine optische Lupe annehmen übrigens, numerisch überaus exakt mit gewaltigen DGLs dazu.
    Also noch einmal meine Frage: Was ist für Sie der Nutzen einer Theorie über die experimentskonforme Zahlenproduktion hinaus?

  50. #50 MartinB
    16. April 2015

    @benjamin
    ““Ein umfassende Erklärung der Phänomene” fordern Sie. Also keine präzise Numerik, sondern …?”
    Eine Erklärung, die die vielen Phänomene gemeinsam erklärt, nicht eine Spezialerklärung für jedes Phänomen.

    “Ich könnte derweil auch gekrümmte Raumzeit für eine optische Lupe annehmen übrigens, numerisch überaus exakt mit gewaltigen DGLs dazu.”
    Wie das konsistent und mit der korreklen Wellenlängenabhängigkeit geht (ohne dass die Lupe dabei durch die gewaltige Raumzeitkrümmung zersplittert oder man die Krümmungseffekte anderweitig beobachten kann, und so, dass man auch die Lichtablenkung im Prisma, wo das Licht ja gerade Wege geht, versteht) würde ich wirklich sehr gern sehen, das dürfen Sie gern mal vorrechnen.

    “Was ist für Sie der Nutzen einer Theorie über die experimentskonforme Zahlenproduktion hinaus?”
    Ich halte nicht soo viel von unkonkreten Diskussionen im luftleeren Raum, wie Sie merken.

  51. #51 Benjamin Packisch
    16. April 2015

    @MartinB:
    “Ich halte nicht soo viel von unkonkreten Diskussionen im luftleeren Raum, wie Sie merken.”
    Dafür weichen Sie meinen Fragen aber sehr kreativ aus ;-).
    Ich will es mal zu ihrem Besten/Konkretesten deuten:
    Die Menge an Phänomenen, für die EINE Theorie korrekte Zahlen produziert, ist Ihr, verzeihung DAS Theoriequalitätsmaß.

    Nun bleibt hier allerdings der schale Eindruck zurück, dass diese EINE Theorie nur so gut in Ihrem Maß sein kann, wie die EINE Deutungspraxis dieser Theorie. Und da erkenne ich zirkelschließende Sophistik. Eine Formel macht nun mal keine physische Realität – außer unter der Annahme, dass es nur EINE Deutung gäbe. Diese Annahme ist aber, wie gezeigt, falsch.

    Zu meinem Lupenbeispiel: Um das redlich und im Sinne meiner Aussage zu tun, müssen Sie mir allerdings zugestehen, ähnlich spekulativ wie im Falle der Gravitationslinsen vorgehen zu dürfen. Erlauben Sie das? Ich denke in diesem Fall, kommen Sie auch selbst drauf, oder? Stichwort: Ablenkung geschieht nur an Raumkrümmungsgradienten.

  52. #52 MartinB
    16. April 2015

    @Benjamin
    “Ich denke in diesem Fall, kommen Sie auch selbst drauf, oder? ”
    Nein, ich habe keine Idee, wie das konsistent geehn soll. Zeigen Sie mal.

  53. #53 Benjamin Packisch
    16. April 2015

    Sie antworten bedauernswerterweise recht selektiv – nun gut, ihre Wahl. Ich bin daher gezwungen Annahmen über Sie zu treffen, z.B. dass Sie physikalische Deutungsfreiheit nun ebenfalls als logisch notwendig akzeptieren.

    Zur Lupe:
    Wir suchen nun also eine Geodätengleichung für einen Lichtstrahl, so dass die Lösung den Lichtstrahlwie ein Strahl durch eine optische Linse beschreibt. D.h. wir suchen eine Kräfteverteilung einerseits (=Energie-Impuls-Tensor), die durch eine spezifische Metrik andererseits beschrieben wird (Raumkrümmungstensor/Ricci-Tensor oder Einsteintensor).
    Einverstanden soweit?
    Als nächstes werden wir die Sache mit einigen Annahmen vereinfachen – wie üblich.
    1. Die Metrik ist jeweils euklidisch in konstantem Material. Das heißt, der Strahl verläuft kräftefrei außer an Materialgrenzen, und das heißt T wird dort 0.
    2. An den Phasengrenzen brauchen wir eine konstante Krümmung in einer Dimension und ansonsten wieder euklidische Verhältnisse. Das heißt, wir brauchen nur die Phasengrenze zu betrachten.
    3. In den (ausgedehnten) Phasengrenzen brauchen wir näherungsweise jeweils eine Hyperbel als Ergebnis. Wir brauchen also als Ziel eine Differentialgleichung als Geodätengleichung die in etwa eine Hyperbel als Ergebnis bringt. Keine unlösbare Aufgabe, denke ich.

    Soweit erstmal. Einwände? Anregungen?
    Im Anschluss können wir über diese Gleichung nachdenken und sie physikalisch interpretieren.

  54. #54 MartinB
    16. April 2015

    @Benjamin
    Vorsicht. Wenn Sie den EIT als Quelle für die Raumkrümmung heranziehen wollen, dann müssen Sie erklären, warum die Massendichte und die optische Dcihte im Material nicht übereinstimmen – durch geringes Zufügen von Phosphor z.B. kann man den Brechnungsindex von Glas deutlich ändern – die Dichte ändert sich aber nicht.

    Und wenn es nur die Wege sein sollen, die beeinflusst werden, dann müssen Sie natürlich auch erklären, warum dann unterschiedliche Farben unterschiedlich gebrochen werden.

    “1. Die Metrik ist jeweils euklidisch in konstantem Material. Das heißt, der Strahl verläuft kräftefrei außer an Materialgrenzen, und das heißt T wird dort 0. ”
    Das dürfte es ziemlich knifflig machenm, zu erklären, wie ein Prisma funktioniert, oder?

    Und wenn T überall Null ist außer an Materialgrenzen, dann kann T nicht der EIT sein.

    Außerdem verstehe ich nicht, wie wir von Raumkrümmung sprechen können, wenn da nur singulär etwas an der Grenzfläche passiert.

    “An den Phasengrenzen brauchen wir eine konstante Krümmung in einer Dimension und ansonsten wieder euklidische Verhältnisse.”
    Konstante Krümmung in einer Dimension verstehe ich nicht – eine normale Linse ist ja in zwei Dimensionen gekrümmt.

    “In den (ausgedehnten) Phasengrenzen brauchen wir näherungsweise jeweils eine Hyperbel als Ergebnis.”
    Verstehe ich auch nicht. Das wird doch auch davon abhängen, was für einen Linsentyp wir haben. Und wenn dort Ihr Quellterm T (was immer der ist) Null ist, weil dort alles euklidisch ist, dann breitet sich Licht doch im Inneren geradlinig aus, oder nicht?

    Ich sehe immer noch nicht mal ansatzweise, wie das funktionieren soll.

    PS:
    ” Ich könnte derweil auch gekrümmte Raumzeit für eine optische Lupe annehmen übrigens, numerisch überaus exakt mit gewaltigen DGLs dazu.”
    Wie sieht denn nun die numerisch überaus exakte gewaltige DGL aus? Das Statement klang ja schon so, als ob Sie die schon fertig haben…

  55. #55 Realistischer
    17. April 2015

    @Adent
    Ich kenne nur Ihre Äusserungen, denen entnehme ich etwas. Kann alles nicht so gemeint gewesen sein, kann alles ein Missverständnis gewesen sein, oder auch nicht. Weis man nicht.
    Sicher ist aber eins: nur weil ich was nicht verstehe das andere vorgeben, heisst dass noch lange nicht dass ich dumm bin. Es kann genauso gut daran liegen dass das, was die anderen da vorgeben, an sich unverständlich ist.
    Ziemlich sicher bin ich mir darin, dass Sie jemand sind der per Vorurteil der Wissenschaft immer recht gibt und mir immer unrecht. Zumindest ist mir kein gegenteiliger Fall bekannt.
    Und wie Andreas Müller freundlicher Weise erklärt hat, stösst as Modellierungskonstrukt Raumkrümmung schon beim zweitleichtesten Anwendungsfall an seine Grenzen – ich bleibe daher aus gutem Grund dabei, diesem Konstrukt seine Befähigung abzusprechen, das Szenario mit dem abgelenkten Kometen zu erklären, denn wie ich ja von Anfang an sagte, der Komet ist nicht allein im Raum, da sind noch andere Objekte auf die er treffen wird, … und weil sich das nicht mehr berechnen lässt, hätte ich die vorgeschlagene Wette gewonnen, auf die Andreas Müller in weiser Voraussicht nicht eingegangen ist. Und ihr Vorurteil dass die Wissenschaft immer recht hat und ich immer unrecht weil ein Crank ohne Basis… nun ja, das war sicher auch nur ein Missverständnis und nicht so gemeint bzw. wäre es ggf. genau umgekehrt, denn ich habe die besseren Argumente, ihr geht nur nicht darauf ein wenn’s euch nicht passt.

  56. #56 MartinB
    17. April 2015

    @realistischer
    Ich bitte noch einmal, damit aufzuhören, hier Unsinn reinzuschreiben. Von “das Modell stößt an seine Grenzen” kann nicht die Rede sein, nur weil man die Gleichungen nicht analytisch lösen kann. Das von Ihnen geliebte Newton-Gesetz stößt beim dort zweitleichtesten Fall, dem Drei-Körper-problem, auch an seine Grenzen und kann nur numerisch gelöst werden. (Auch dort kann man die Bahn eines Kometen, die von zwei anderen Körpern beeinflusst wird, nur numerisch berechnen.)
    Ob ein physikalisches Modell analytisch oder numerisch gelöst werden kann oder muss, sagt nichts über seine Leistungsfähigkeit aus.

  57. #57 Artur57
    Mannheim
    17. April 2015

    Warum eigentlich folgt die Existenz von Gravitationswellen aus der Endlichkeit der Lichtgeschwindigkeit? Ich fand die Begründung immer etwas schwammig, aber es bietet sich heute eine Erklärung an, die zu Einsteins Zeiten noch nicht möglich war.

    Nehmen wir an, zwei Schwarze Löcher rasen frontal aufeinander zu mit nennenswerten Bruchteilen der Lichtgeschwindigkeit. Die Endlichkeit der Gravitationausbreitung hat nun den Effekt, dass sich die Probanden nicht mit der Kraft anziehen, die ihrem tatsächlichen Ort entspricht, sondern einer vergangenen Konstellation. Die Kraft wäre bei instantaner Ausbreitung größer.

    Das hat aber zur Konsequenz, dass wir Probleme mit der Energieerhaltung bekommen. Denn auf diese Weise wird nicht die gesamte potentielle Energie in kinetische Energie umgesetzt. Es bleibt also ein Restbetrag, der nun in Form von Gravitationswellen freigesetzt wird. Also eine Notwendigkeit, um der Energieerhaltung Genüge zu tun.

    Will ich natürlich nicht behaupten, sondern nur fragen, ob das ein brauchbarer Ansatz wäre.

  58. #58 Benjamin Packisch
    17. April 2015

    @MartinB:
    Mir drängt sich der Eindruck auf, Sie wollen gar nicht eine neue Perspektive auf die ART erlangen. Na gut, vielleicht irre ich in Ihrer Motivation, also will auf ein gutwilliges Neues:
    Die von mir vorgeschlagene und nun von Ihnen geforderte Aufgabe hat folgendes Ziel, reduziert auf den analytischen Knackpunkt:
    Aufstellung einer Gleichung mit Bezug zum ART-Formalismus, die Strahlablenkung an einer Dichtegrenze beschreibt. Die kann singulär sein, muss aber nicht. Sinnvollerweise beschreiben wir sie aber ausgedehnt, um das realere Bild einer Hyperbelform zu erhalten (und nicht einfach ein Knick im Strahl). An Prismen geht das ganz genauso ein wellenlängenabhängiger Parameter zur Skalierung der Hyperbel ist nun kein Problem.
    Verstehen Sie jetzt das Prinzip?

    Zur Ihrer Vorsichtsmahnung: Unser Vergleichsobjekt ist die Gravitationslinse, über die wir direkt nur Strahlungsinformation haben, sowie eine Reihe von Annahmen und den aus den Annahmen folgenden Größen. Wir dürfen also für die Linsenanalogie durchaus auch Annahmen treffen, insbesondere über die Qualität, die die Krümmung verursacht. Aber solche Annahmen sind der astronomischen Rechnung üblicherweise nachgeordnet – wenn die Numerik stimmt.
    Denken Sie z.B. auch an Dirac und das Positron.

    P.S.: Das mit der gewaltigen DGL war ein Hinweis auf die völlig reale und leider auch nicht unübliche Möglichkeit, etwas analytisch stark aufzublähen. In einem Forum wie diesem scheint mir da redlich komplexe Analytik gebotener, s.o. . Etwas komplizierter machen als es ist, ist ja nun nicht gerade eine Kunst… im übrigen habe ich nichts vorbereitet, das ist Neuland.

  59. #59 Stefan K.
    17. April 2015

    Hallo! Ist hier zwar absolut off-topis, aber der letzte Dino-Artikel ist schon länger her, deshalb schreib ichs hier in der Hoffnung, dass mein Post weniger leicht übersehen wird (bitte gerne verschieben,falls offtopic stört):
    Nachdem ich in einem anderen Forum die Aussage:
    “An gefiederte Dinosaurier werde ich mich nie gewöhnen. Liebe Wissenschaftler, könnt ihr bitte herausfinden, dass die Dinos doch wie in Jurassic Park ausgesehen haben. Das würde mein Weltbild wieder in Ordnung bringen, danke…”
    gelesen habe und er mir aus der Seele spricht, hab ich mir gedacht, ich frag einen Experten, und da kam mir natürlich Martin Bäker in den Sinn. Also, wie stehn die Cahncen auf neue Erkenntnisse, die Federn widerlegen? Vermutlich schlecht fürchte ich 😉

  60. #60 MartinB
    17. April 2015

    @Benjamin
    “Aufstellung einer Gleichung mit Bezug zum ART-Formalismus, die Strahlablenkung an einer Dichtegrenze beschreibt. “Nein, das ist nicht die Forderung. Die Forderung ist:
    Aufstellen einer in sich un dmit dem rest der Physik konsistenten Theorie, die die Lichtablenkung an einer Linse in irgendeiner Weise auf “Raumkrümmung” zurückführt.

    Dabei dann bitte eben auch beachten, dass der gerkümmte Raum auch die Packung der Atome beeinflussen müsste, dass natürlich auch Schallwellen in gleicher (oder wegen der anderen Packung der Atome anderer, aber berechenbarer) Weise beeinflusst werden, und dass der Effekt wellenlängenabhängig sein muss und dass die Raumkrümmung natürlich keinen Einfluss auf umliegende Dinge wie die Halterung der Linse haben kann. Denn das ist das, was Sie implizieren, wenn Sie sagen:
    “Ich könnte derweil auch gekrümmte Raumzeit für eine optische Lupe annehmen übrigens, numerisch überaus exakt mit gewaltigen DGLs dazu.”
    Denn Eine gekrümmte Raumzeit darf ja nicht bloß das Licht beeinflussen.

    @StefanK
    Die Chancen stehen seehr seeehr schlecht – wobei große Dinos wie z.B. T. rex vermutlich nicht gefiedert waren (ähnlich wie auch Elefanten keinen dicken Pelz tragen), da gbt es auch ein paar Hautabdrücke. Aber es gibt inzwischen so viele Dinos mit Federn, dass die Frage wohl als geklärt gelten muss. (Von einigen abstrusen Abweichlern abgesehen…)

  61. #61 Benjamin Packisch
    17. April 2015

    @MartinB:
    Sie fordern ja immer mehr! Und immer, wenn ich auf einen Ihrer Einwände eingehe, fordern Sie gleich noch mehr – jetzt also auch noch Schallwellen…! Die gekrümmte Raumzeit darf auch Schallwellen beeinflussen, auch vielleicht noch die Raumtemperatur, auch den Luftdruck in 1 und 100m Entfernung der Linse, und auch noch die Phosphor-Dotierung und den Preis der Linse ;-)? Bei der Gravitationslinse ist da die Zusatzannahmenbereitschaft doch etwas, sagen wir “kompromissbereiter”, nicht wahr?
    Vielleicht ahnen Sie jetzt, was ich mit gewaltigen DGLs meine und warum ich sie nicht besonders zielführend finde, um nicht zu sagen: sophistisch. Natürlich ist das möglich, analytisch wie auch numerisch, aber was soll das bringen? Barocke Fülle? Ihr Weltbild retten, weil sie dann in den Details endlich den Fehler entdecken, auf den sie die ganze Zeit schon lauern!?
    In so einem Forum würde ich mich lieber auf das Wesentliche bescheiden und möglichst nur das akut zugänglich verwenden, Occams Razor, Sie wissen schon.

    Mein Punkt ist und war der:
    Theoretische Selbstkonsistenz != Realitätstreue, AUßER man erzwingt es, indem man implizit ein straffes, womöglich realitätsverzerrendes Voraussetzungssystem setzt, das nur der “Eingeweihte” kennt. Diese Tendenz habe ich hier und im obigen Artikel beobachtet, und sie ist m.E. potentiell ungut für das Verständnis, und deshalb weise ich hier daraufhin.

    Wenn Sie damit etwas anfangen mögen, freut mich das, ansonsten walten Sie bitte gern Ihres Amtes wie gehabt!
    MfG
    BP

  62. #62 MartinB
    17. April 2015

    @benjamin
    “Sie fordern ja immer mehr! ”
    Nein, ich fordere eine in sich konsistente Theorie, nicht bloß ne Gleichung.

    “Die gekrümmte Raumzeit darf auch Schallwellen beeinflussen, auch vielleicht noch die Raumtemperatur, auch den Luftdruck in 1 und 100m Entfernung der Linse, und auch noch die Phosphor-Dotierung und den Preis der Linse ;-)? ”
    Das müssen Sie aus Ihrer Theorie ableiten – so funktioniert das in der Physik. Physik ist nicht das Aufstellen von DGLs.

    “Bei der Gravitationslinse ist da die Zusatzannahmenbereitschaft doch etwas, sagen wir “kompromissbereiter”, nicht wahr?”
    Es geht nicht um Zusatzannahmen, es geht darum, das Theorien mit allem, was wir wissen, konsistent sein müssen. Eine Theorie, die sagt “Die Raumzeit wird beeinflusst, aber nicht für Schallwellen” ist eben keine konsistente Theorie der Raumzeit, es sei denn, sie erklärt, warum die Schallwellen nicht beeinflusst werden.

    “Mein Punkt ist und war der:”
    Nein, Sie haben was anderes behauptet, nämlich dass es ein leichtes sei, alternative Erklärungen aufzustellen und dass es solche Erklärungen für die durch die ART beschriebenen Phänomene gäbe. Auf konkrete nachfrage sind Sie dann auf die Linse ausgewichen, aber auch da steckte anscheinend nichts hinter der Behauptung, Sie könnten eine alternative Beschreibung liefern – und eine physikalische Beschreibung ist eine Theorie, nicht bloß eine DGL, bei der man lieber nicht hinterfragt, wie sie zum rest der Physik passt.

    “In so einem Forum würde ich mich lieber auf das Wesentliche bescheiden”
    Soll heißen möglichst im abstrakten Raum diskutieren, aber nicht über konkrete Theorien oder Phänomene. Dazu habe ich wenig Lust – genau wie Physik selbst klappt auch Wissenschaftsphilosophie besser, wenn man sich was Konkretes anguckt.

    “Theoretische Selbstkonsistenz != Realitätstreue”
    Das ist trivial – auch Pseudowissenschaft kann selbstkonsistent sein.
    Gute Theorien müssen schon beides können.

    ” straffes, womöglich realitätsverzerrendes Voraussetzungssystem setz”
    Sagen Sie doch konkret am Beispiel der ART, was dieses System sein soll (und welche Alternativen sich bei einem anderen Voraussetzungssystem ergeben würden).

  63. #63 Benjamin Packisch
    17. April 2015

    @MartinB.
    “Nein, ich fordere eine in sich konsistente Theorie, nicht bloß ne Gleichung. ..Es geht nicht um Zusatzannahmen, es geht darum, das Theorien mit allem, was wir wissen, konsistent sein müssen.”
    Ja, aber das geht nur unter der Voraussetzung, dass ein einheitliches System zur Akzeptanz von (physikalischem) Wissen besteht. Sie blenden völlig aus, dass dies nur relativ zu einer frei wählbaren Aufmerksamkeitsspanne geschehen kann, d.h. welche Phänomene unmittelbar zu beschreiben sind. Diesen Kontext ändern sie hier gerade immer wieder, und genau das ist der sophistisch missbrauchbare Aspekt, auf den ich Sie hinweisen wollte. “Alle” Phänomen kann keine konkret brauchbare Theorie beschreiben (z.B pro Atom ein Atom zur Beschreibung).

    Sie schreiben, ich wiche aus? Weil ich Ihnen nicht einfach so recht gebe!? Wollen Sie konkret ein Beispiel in endlicher Zeit eruieren, also eine höchst konkrete Linse mit einer wie üblich abstrakten Theorie (mein Angebot von einigen, wenn sie genau lesen), oder wollen Sie mir nachweisen, dass ich besser meinen Schnabel gehalten hätte?
    Ersteres haben wir ja gerade erst angefangen, zweiteres wäre schon im Gange und m.E. nicht weiter zitierwürdig. Ihre Wahl!

    Zu Ihrer letzten Frage:
    “Sagen Sie doch konkret am Beispiel der ART, was dieses System sein soll (und welche Alternativen sich bei einem anderen Voraussetzungssystem ergeben würden).”
    Poincaré’scher Konventionalismus ist das Stichwort. Ob man Kräfte oder eine Geometrie definiert, ist per Konvention wählbar, und nicht beweisbar. Das bedeutet, dass wir in Sachen Geometrie in der ART-Physik sehr viel mehr Spielraum haben, als er üblicherweise genutzt wird. Das führt dazu, dass dieser Freiraum nicht zu besserer Wissenschaft führt, sondern in sinnlosen Grabenkämpfen verschlissen wird, bevor man etwas Brauchbares, nicht bloß insulär Nachvollziehbares entwickelt hat. Der Alternativen gibt es viele, aber es ist nicht zielführend, irgendein fertiges System hinzuschleudern, wenn Sie noch nicht mal mit den Vorannahmen einverstanden sind. Sowas nützt keinem- auch wenn das im Crank-Sektor sehr verbreitet ist.

  64. #64 MartinB
    17. April 2015

    @benjamin
    “Ja, aber das geht nur unter der Voraussetzung, dass ein einheitliches System zur Akzeptanz von (physikalischem) Wissen besteht. ”
    Sorry, das ist mir zu abstrakt und abgehoben – die Physik ist pragmatischer als solches theoretisches Überlegen ohne Erdung.

    “Wollen Sie konkret ein Beispiel in endlicher Zeit eruieren, also eine höchst konkrete Linse mit einer wie üblich abstrakten Theorie”
    ?? Wollen sie für jede Linse ne eigene Theorie aufstellen oder wie ist der Satz zu verstehen?

    “Ob man Kräfte oder eine Geometrie definiert, ist per Konvention wählbar, und nicht beweisbar. ”
    Dazu habe ich ja gerade einen Artikel geschrieben, der genau das aussagt. Ja, es gibt unterschiedliche Möglichkeiten, die Gleichungen der ART zu interpretieren. Das sagt nicht viel über die Theorie, deren Vorhersagen dieselben bleiben (evtl. abgesehen von Dingen wie Topologieänderungen, die aber hypothetisch sind), sondern mehr darüber, was unsere Anschauung mit Gleichungen macht.

    “sondern in sinnlosen Grabenkämpfen verschlissen wird”
    Wer soll denn da Grabenkämpfe führen?

    “Der Alternativen gibt es viele, ”
    Wie oben schon gesagt: Name ten.

    “aber es ist nicht zielführend, irgendein fertiges System hinzuschleudern”,
    Schleudern Sie ruhig – hic rhodos, hic salta.

    “wenn Sie noch nicht mal mit den Vorannahmen einverstanden sind. ”
    Da Sie immer noch nicht konkret sagen, welche das sein sollen, wird das ganze langsam ermüdend.

    Ich bringe es mal auf den punkt: Sie erzählen hier von vielen Alternativen zur ART, nennen aber konkret keine einzige.
    Sie erzählen mir, Sie könnten problemlos eine auf Raumkrümmung basierende physikalische Theorie für die Lichtbrechung präsentieren (mit einer angeblich toll komplexen DGL) – aber auf Nachfrage kommt nichts außer Ausflüchten.

    Entweder Sie werden mal konkret, oder wir können es lassen; vages Gerede von “Vorannahmen” und “Konventionen” ist mir zu wenig, um darauf Zeit zu ver(sch)wenden.

  65. #65 Realistischer
    18. April 2015

    @MartinB
    Ihr beschäftigt euch mit esoterischen Konzepten die noch nie jemand gesehen oder gemessen hat, aber ich schreibe Unsinn? Alles klar.

  66. #66 erik der wikikinger . . . ..
    19. April 2015

    @realistischer
    . . . .. welche der 4 elementarkräfte sind destruktiv, welche sind konstruktiv? die ordnung aller dinge (kurz zusammengefasst: ∑ aller wissenschaftlichen, unwissenschaftlichen konzepte) schliesst keine der von dir beobachteten seiten aus . . . ..
    Alle Dinge im Universum können auf der Basis von Physik und Mathematik beschrieben werden. Alle Dinge in EINEN Determinismus zu stecken, . . . .. das ist die Herausforderung . . . ..
    Dies gelingt nur, wenn Gegensätzliches – Miteinander wirken kann. Zum Beispiel setze ich der Kategorie “RICHTIG” (eigenes richtiges denken) deren NEGATION entgegen. Dieses Prinzip findest du (und wir alle anderen natürlich auch) in allen Dingen im Universum: EINS steht immer ALLEM gegenüber, beide bilden gemeinsam ALLES . . . ..
    Die Gleichung 1+1=2 hat nur eine Richtige Lösung und alle anderen, unendlich vielen Aussagen (z.B. 3; 4; 5,6 . . . ∞) sind FALSCH: . . . .. RICHTIG gibt es nur einmal – deren NEGATION, das FALSCH steht dem ∞mal minus 1 entgegen. . . . . .. exp( i * π)= -1 steht dem +1 gegenüber und müssen zusammen =0 ergeben . . . .. Warum müssen die Gegensätzlichen Einsen – EINS-SEIN und zusammen Null ergeben? Wegen einer Ordnung aller Dinge . . . .. man könnte meinen beide EINSEN bilden eine Quantenzustand, so wie ein Photon oder Graviton . . . ..
    . . . .. alles was wir auf diesem Blog erleben ist Physik und Mathematik, weil alles im Leben auf Physik und Mathematik reduziert werden kann. . . . .. [Ph||Ma] bilden auch einen Quantenzustand? . . . .. das muss dann doch näher erläutert werden . . . ..
    Danke für deine Zeit und Inspiration.

  67. […] es am Ende hakt. In gewisser Weise ist dieser Text die Fortsetzung (oder auch das Komplement) zum vorigen Artikel über Gravitonen, ich versuche aber mal, ihn einigermaßen so zu schreiben, dass man ihn auch für sich lesen kann. […]

  68. #68 Benjamin Packisch
    20. April 2015

    @MartinB:
    ” .. Ja, es gibt unterschiedliche Möglichkeiten, die Gleichungen der ART zu interpretieren. …”
    Na eben! Und mein Punkt ist, dass es auch unterschliedliche Möglichkeiten gibt, u.a. die ART physikalisch zu verwenden.

    Die Linse ist ein Beispiel, und Sie müssen zugeben, dass es für einen Physiker keine unlösbare Aufgabe ist: Als Ansatz eine Hyperbel y = +/-sqrt(x²/a² -1) annehmen und dafür eine DGL mit Kompatibilität zum ART-Formalismus aufstellen, und diese dann als möglichen EIT bzw. Masseverteilung zu interpretieren.
    Das ist eine schöne Aufgabe für einen Übungszettel zur Verdeutlichung das Unterschieds zwischen Formel und physikalischer Interpretation, sowie zum Prinzip des ART-Ansatzes.

    Ich hoffe, dass ist Ihnen jetzt konkret genug!

    Im Idealfall regt es dazu an, die logische Ebene hinter der mathematischen Modellbildung zu erkennen: Nämlich die Freiheit in der Objektwahl, die rein nach der jeweiligen Absicht getroffen werden kann ohne Einbußen in der logischen Konsistenz.
    Dass es gute Gründe für Konventionen in dieser Wahl gibt, liegt auf der Hand. Aber meine Hoffnung ist es, dass die Einsicht in die konkrete Wahlfreiheit dabei ein wenig den “biblischen Zorn” aus der Causa Physik-Grundsatz-Diskussion nehmen kann und dass Gleichungen wie u.a.”RT-Kritiker=inkompetenter Narr” eben nicht unbedingt wissenschaftlich sind (auch wenn sie manchmal oder sogar oft stimmen…).

    Ein allzu frommer Wunsch :-)?

  69. #69 MartinB
    20. April 2015

    @Benjamin
    “diese dann als möglichen EIT bzw. Masseverteilung zu interpretieren.”
    Und diese Masseverteilung hat aber keine der sonstige Eigenschaften von Masse, wie etwa Trägheit oder Erzeugung eines Schwerefelds…
    Sorry, aber ich glaube, Sie haben nicht wirklich verstanden, wie eine physikalische Theorie funktioniert – das Aufstellen einer DGL und Hineininterpretieren von irgendetwas in diese DGL ist es nicht.

    “Nämlich die Freiheit in der Objektwahl, die rein nach der jeweiligen Absicht getroffen werden kann ohne Einbußen in der logischen Konsistenz.”
    Nein, wenn ich das, was die Linse mit Licht tut, auf einen EIT zurückführe, dann muss dieser EIT eben auch alle sonstigen Eigenschaften haben, die einem EIT zukommen, sonst habe ich einfach nur das falsche Wort gewählt.

    ” dass Gleichungen wie u.a.”RT-Kritiker=inkompetenter Narr” eben nicht unbedingt wissenschaftlich sind ”
    Ach daher weht der Wind…
    Hätt’ ich mir ja denken können.

  70. #70 Benjamin Packisch
    20. April 2015

    @MartinB
    “Sorry, aber ich glaube, Sie haben nicht wirklich verstanden, wie eine physikalische Theorie funktioniert – das Aufstellen einer DGL und Hineininterpretieren von irgendetwas in diese DGL ist es nicht.”
    Wenigstens glauben Sie das nur ;-). Vielleicht erinnern sich bei Gelegenheit doch wieder an die Sache mit Dirac und dem Positron.

    “Nein, wenn ich das, was die Linse mit Licht tut, auf einen EIT zurückführe, dann muss dieser EIT eben auch alle sonstigen Eigenschaften haben, die einem EIT zukommen ..”
    Also warum sträuben Sie sich denn gar so sehr…? Der EIT ist doch keine heilige Kuh, die man nur auf kosmischer Weide grasen lassen darf ;-)! Es geht doch hier um das Prinzip dahinter. Oder konkreter: Welche Eigenschaften gehen Ihnen denn ab im Linsenbeispiel?

    ” ” dass Gleichungen wie u.a.”RT-Kritiker=inkompetenter Narr” eben nicht unbedingt wissenschaftlich sind ”
    Ach daher weht der Wind… ”
    Hätt’ ich mir ja denken können.”
    Was hätten Sie sich denken können?

  71. #71 MartinB
    20. April 2015

    “Vielleicht erinnern sich bei Gelegenheit doch wieder an die Sache mit Dirac und dem Positron.”
    Ihnen ist schon bewusst, dass die Dirac-Interpretation mit der Dirac-See falsch war?

    “Der EIT ist doch keine heilige Kuh, die man nur auf kosmischer Weide grasen lassen darf ”
    Nein, aber der EIT fasst bestimmte physikalische Größen wie Energien udn Impulse zusammen. Man kann nicht irgendeine Gleichung aufstellen und dann eine Größe darin willkürlich EIT nennen und glauben, jetzt hätte man ein sinnvolles Modell.

    “Was hätten Sie sich denken können?”
    Das es letztlich um “Kritik” an der RT geht.

  72. #72 Benjamin Packisch
    20. April 2015

    @MartinB:
    Ihnen ist sicher auch bewußt, dass man das Positron überhaupt nur gesucht bzw. gefunden hat aufgrund von Diracs Berechnungen – insofern verstehen Sie schon. Seine Deutungen, nun ja, Interpretationen ändern sich eben von Zeit zu Zeit – sogar bei gleicher Numerik. Oder meinen Sie, dass die Lehrmeinung an dieser Stelle irreführend ist?

    Energien und Impulse sind deweil doch ziemlich konkrete Konzepte, die man bei einem Lichtstrahl auch anwenden darf, und zwar wie beschrieben am besten in Grenzschichtnähe bei ansonsten euklidischen Verhältnisse – eben analog dem Lichtweg durch eine Gravitationslinse mutatis mutandis.

    Was meinen Sie mit “Kritik” an der RT? Die RTen sind m.E. eine wunderbare Sache, mathematisch wie physikalisch – glücklicherweise geht es in der Physik ja nicht um den Glauben an eine Theorie oder deren Sinn, sondern um Brauchbarkeit. Oder sehen Sie das anders? Sind sie nachher ein “Deutungsdogmatiker”? Das wiederum sollte ich mir vielleicht allmählich denken…

  73. #73 mesut2012
    20. April 2015

    Mal eine Frage als Laie: bei den drei Grundkräften außer der Gravitation hat man das klassische Feld quantisiert (im Rahmen der Quantenmechnik mittels Austauschteilchen beschrieben) und versucht nun, diese Beschreibung auf die Gravitation auszudehnen. Gab es auch bereits mal Versuche, das Feld dieser drei Kräfte zu “geometrisieren”, wie es Einstein mit dem klassischen Gravitationsfeld getan hat und vielleicht auf diese Weise zu einer einheitlichen Beschreibung zu gelangen?

  74. #74 MartinB
    20. April 2015

    @Benjamin
    Mir wird’s jetzt wieder zu unkonkret…

    “Energien und Impulse sind deweil doch ziemlich konkrete Konzepte, die man bei einem Lichtstrahl auch anwenden darf,”
    Aber nicht, um die Lichtbrechung zu erklären – denn dazu bräuchte man sehr große raumkrümmungen…

    @mesut
    “Gab es auch bereits mal Versuche, das Feld dieser drei Kräfte zu “geometrisieren”, wie es Einstein mit dem klassischen Gravitationsfeld getan hat und vielleicht auf diese Weise zu einer einheitlichen Beschreibung zu gelangen?”
    Oh ja, diverse sogar. Einstein selbst hat das lange versucht, auch einige andere. Im Moment sieht es aber nicht so aus, als ob eine “einfache” Geometrisierung funktioniert. Tatsächlich ist aber die formale Ähnlichkeit zwischen der ART und den anderen Kräften relativ groß – das Stichwort dazu ist “Eichtheorie”; dazu habe ich vor langer Zeit mal was gebloggt:
    https://scienceblogs.de/hier-wohnen-drachen/2011/03/09/sind-elementarteilchen-symmetrisch/

  75. #75 Benjamin Packisch
    20. April 2015

    @MartinB.
    “Aber nicht, um die Lichtbrechung zu erklären – denn dazu bräuchte man sehr große raumkrümmungen…”
    Ja, und…? Warum sollte das nicht lokal, d.h. genau in der Grenzschicht des Materialwechsels möglich sein…?

  76. #76 Niels
    20. April 2015

    @Benjamin Packisch
    Kannst du mal kurz zusammenfassen, worum es dir eigentlich konkret geht?

    Und mein Punkt ist, dass es auch unterschliedliche Möglichkeiten gibt, u.a. die ART physikalisch zu verwenden.

    Nein? Das würde doch bedeuten, dass man für einen Vorgang mit Hilfe der ART mindestens zwei unterschiedliche Vorhersagen treffen könnte. Wobei aber natürlich in der Natur nur ein Zustand eintreffen kann. Die ART wäre dann nicht mehr widerspruchsfrei und damit als Theorie erledigt.

    Im Idealfall regt es dazu an, die logische Ebene hinter der mathematischen Modellbildung zu erkennen: Nämlich die Freiheit in der Objektwahl, die rein nach der jeweiligen Absicht getroffen werden kann ohne Einbußen in der logischen Konsistenz.

    Hm?
    Mal ganz von Anfang an. In der Physik betrachten wir ein System, ermitteln Messgrößen zum Zeitpunkt t1 und sagen dann mit Hilfe eines mathematischen Formalismus voraus, welche Werte diese Messgrößen zum Zeitpunkt t2 annehmen.
    Welche Freiheit in der Objektwahl gibt es dabei?

  77. #77 Benjamin Packisch
    20. April 2015

    @Niels:
    Es ging und geht mir um die m.E. etwas zu schwache Betonung der Wahlfreiheit in der Abbildung Mathematik-Physik, hier im Aritkel zum Graviton. Die Rolle des modellbildenden Beobachters und seiner Absicht wird dabei z.B. unterschlagen. Graviton und ART z.B. folgen m.E. unterschiedlichen Absichten.

    “Das würde doch bedeuten, dass man für einen Vorgang mit Hilfe der ART mindestens zwei unterschiedliche Vorhersagen treffen könnte.”
    Möglich, aber ich wollte auf zwei andere Dinge hinaus: einerseits, dass man den gleichen Vorgang – feste Modellierungskonzepte vorausgesetzt – auf viele Arten formal darstellen kann; andererseits, dass man den gleichen Formalismus auf unterschiedliche Phänomene anwenden kann.
    Für die ART bedeutet das: sie ist physikalisch nur so widerspruchfrei wie deren Interpretationspraxis – und je abstrakter eine Theorie, desto heikler.

    Die Freiheit in der Objektwahl besteht darin, dass man die Messgröße und dessen Kontext frei bestimmen kann, d.h. die Bestimmung der Abgrenzung idealer Größen und deren Störungen. Im Grunde ist es der Kern der Aussagen Kants, dass die a priori Wahl unserer Art der Anschauung die Beobachtung beeinflusst.

  78. #78 MartinB
    20. April 2015

    @Benjamin
    ” dass man den gleichen Vorgang – feste Modellierungskonzepte vorausgesetzt – auf viele Arten formal darstellen kann…dass man den gleichen Formalismus auf unterschiedliche Phänomene anwenden kann.”
    Das bleibt ja beides zu belegen – das Linsenbeispiel war ja ein grandioser fehlschlag.

    Nein, man kan nich tmal eeben so tun, als würde Lichtablenkung in einer Linse durch Raumkrümmung hervorgerufen – jedenfalls nicht ohne dabei die Begriffe der Physik vollkommen durcheinanderzukegeln.
    “der Kern der Aussagen Kants, dass die a priori Wahl unserer Art der Anschauung die Beobachtung beeinflusst.”
    Von einer “Wahl” hat Kant im Hinblick auf die a priori-Erkenntnisse nicht geredet, im Gegenteil – dass wir dort nicht wählen können, sondern diese vorgegeben sind, führt ja gerade zu den Antinomien der reinen Vernunft.

  79. #79 Niels
    20. April 2015

    @Benjamin Packisch

    Graviton und ART z.B. folgen m.E. unterschiedlichen Absichten.

    Nämlich welchen?

    einerseits, dass man den gleichen Vorgang – feste Modellierungskonzepte vorausgesetzt – auf viele Arten formal darstellen kann; andererseits, dass man den gleichen Formalismus auf unterschiedliche Phänomene anwenden kann.

    Na ja, die klassische Mechanik kann man mathematisch äquivalent entweder mit Hilfe der newtonschen Gesetze oder mit dem Hamilton-Jacobi-Formalismus und dem Prinzip der kleinsten (stationären) Wirkung behandeln.
    Dieses Prinzip und dieser Formalismus haben sich dann auch für andere Bereiche der Physik als überaus fruchtbar erwiesen.

    Für die ART bedeutet das: sie ist physikalisch nur so widerspruchfrei wie deren Interpretationspraxis

    Wo kommt denn diese Folgerung her?
    Ich sehe da keinen Zusammenhang und halte diese Aussage außerdem für falsch.
    Wenn die Mathematik funktioniert und widerspruchsfrei ist, spielt die Interpretationspraxis doch überhaupt keine Rolle.
    So können wir zum Beispiel wunderbar Computer, Laser und lauter andere tolle Dinge bauen, obwohl wir nicht den Schimmer einer Ahnung haben, welche Interpretation der Quantenmachanik die richtige ist (bzw. welche der Wahrheit am nächsten kommt.)

    Ich glaube du verwechselst physikalische Widerspruchsfreiheit damit, wie sehr eine physikalischen Theorie (für dich?) ontologisch zufriedenstellenden Schlussfolgerungen liefern kann.

    je abstrakter eine Theorie, desto heikler.

    (Wie bestimmst du den Abstraktionsgrad einer Theorie? Ist etwa das Entropiekonzept abstrakter als das Konzept der physikalischen Größe der Wirkung, ein Skalarfeld abstrakter als ein Vektorfeld,…?)

    Die Freiheit in der Objektwahl besteht darin, dass man die Messgröße und dessen Kontext frei bestimmen kann

    Hm, könntest du das mal konkret an einem Beispiel verdeutlichen? Das wage ich nämlich zu bezweifeln.

  80. #80 sffd
    20. April 2015

    Gravitonen sind nichts anderes als geschlossene Strings 😉

  81. #81 sffd
    20. April 2015

    Benjamin meint vielleicht die Wahl einer Eichtheorie nach dem Prinzip des logischen Positivismus. Unterschiedliche Eichungen sind voreinander empirisch und logisch/mathematisch nicht unterscheidbar und daher gleich und das Framework, der Rahmen darum ist verschieden. Die Eichung ist bedeutungslos, im gleichen Sinne wie es in der Sprache Fragen wie z.B. welchen Geschmackt hat das Grundgesetzt gibt die bedeutungslos sind weil sie einfach ein Artefakt unserer Sprache sind. Und damit hat er recht. Dieses Prinzip ist das einzige das in der Physik nach dem Wiener Kreis gültig ist. Das wusste auch schon Albert Einstein mit seinem Äquivalenzprinzip und der falschen Aethertheorie. Gravitonen sind theoretisch, aber nur mit der falschen Wissenschaftstheorie von Karl Popper nicht real. Es gibt nicht nur Falsifikation sondern auch Verifikation eben durch logische/mathematische Analyse. Dieses Prinzip hat seit Jahrhunderten zu Fortschritten in der Physik geführt und ist mit Sicherheit richtig. Seinen Höhepunkt das hat Prinzip in der S-Matrix Theorie in der alles bedeutungslos ist ausser Teilchen die aus dem unendlichen kommen, interagieren und wieder in das unendliche gehen. Und kombiniert mit der Reggae Theorie ergibt dies die Stringtheorie. Deswegen gilt auch nicht je abstrakter desto heikler, wenn die Theorie konsistent ist, dann ist sie richtig.

  82. #82 sffd
    20. April 2015

    Um das genauer zu verstehen empfehle ich Literatur über die beiden Gruppen in den 1960-70. Auf der einen Seite Chew/Mandelstam/Gribov mit der S-Matrix Theorie, die nur auf dem logischen Positivismus bzw. konsistentzargumenten besteht. Und die Feldtheorie Gruppe bestehend aus Weinberg/Glashow/Polyakov.

  83. #83 MartinB
    21. April 2015

    @sffd
    “in der alles bedeutungslos ist ausser Teilchen die aus dem unendlichen kommen, interagieren und wieder in das unendliche gehen”
    Gehässig gesagt hat das ja nicht so viel mit unserem Universum zu tun, wo die Teilchen meistens nicht aus dem Unendlichen kommen…

    “wenn die Theorie konsistent ist, dann ist sie richtig.”
    Das halte ich für eine sehr gewagte Behauptung, wenn wir über physikalische Theorien sprechen – es würde ja implizieren, dass es nur eine einzige denkbare Physik geben kann, und das ist eine verdammt starke Behauptung, für die ich überhaupt keinen Beleg sehe. (Außer der Hoffnung einiger Stringtheoretikerinnen, dass sie das als Argument nehmen können, um sich davor zu drücken, mal konkrete Zahlen zu produzieren (ja, das war ein bisschen gehässig, gebe ich zu)).

  84. #84 EnnoA
    Hamburg
    21. April 2015

    @MartinB
    “Wie so oft in letzter Zeit stehen die Dinge, die ich hier erklärt habe, so in keinem Buch (warum, weiß ich nicht).”
    Die Frage ist leicht zu beantworten: Weil Du es noch nicht geschrieben hast! Es gibt offensichtlich sonst niemanden, die das so gut hinkriegt. Warum nicht die Blogeinträge überarbeiten und ein Buch daraus machen?

  85. #85 MartinB
    21. April 2015

    @EnnoA
    “Warum nicht die Blogeinträge überarbeiten und ein Buch daraus machen?”
    Das ist ein gedanke, mit dem ich immer wieder spiele. Das problem ist erstens die Zeit, zweitens, dass ich, wenn ich z.B. die QFT-Serie zum Buch machen würde, im Nachhinein alles nochmal komplett neu machen würde, drittens dass es entweder ein gigantisches Monster wird (weil alles irgendwie mit allem zusammenhängt, siehe die Querverweise) oder ich müsste die Themen sehr genau zuschneiden, und viertens, dass ich nen Verleger bräuchte, wenn das Buch wirklich ne Reichweite bekommen soll, die über die typische Blogleserinnenzahl hinausgeht.
    Was nicht heißen soll, dass ich es nicht trotzdem irgendwann mache – eine mir nahestehende Person sagte mal auf meine Bemerkung “Eigentlich müsste ich mal nen Buch dazu schreiben” “Oh, ich bin sicher, dass du das irgendwann machst”…
    Aber meine Frage zielt eher auf die Physikdidaktik ab – Leute, die QFT/ART-Bücher schreiben, werden diese Dinge doch einfach wissen (hoffe ich zumindest) – warum schreiben die immer nur Formeln hin, statt die auch mal ausführlich zu erklären und zu interpretieren?

  86. #86 Benjamin Packisch
    21. April 2015

    @MartinB.
    “dass man den gleichen Formalismus auf unterschiedliche Phänomene anwenden kann.”
    Das bleibt ja beides zu belegen – das Linsenbeispiel war ja ein grandioser fehlschlag.

    Also kommen Sie: Dass Formalismen nicht zwingend oder 1:1 an Phänomene gebunden ist, ist doch nun wirklich nicht das Ei des Kolumbus..und dass ihnen das Linsenbeispiel nicht schmecken mag, kann ich auch nicht ändern. Formal ist es möglich, und man kann folglich auch numerisch etwas ausrechnen. Wenn Sie einen konkreten physikalischen Einwand haben, nennen Sie ihn mir gern, dann diskutieren wir ihn.

    Und wenn schon Kant, dann richtig, z.B. hier als Einstieg: Kopernikanische Wende des Denkens
    Es geht dabei um den Einfluss des logischen Bezugspunkts auf die Beobachtung – auch wenn Kants Position da nur ziemlich selten konsequent aufgegriffen wird. Die RTen sträuben sich ebenfalls davor, auch wenn sie in die richtige Richtung gehen.

    P.S.: Ihre Bedenken bzgl. der String-Theorie teile ich übrigens voll und ganz. Selbstkonsistenz ist keine Garantie für physikalische Brauchbarkeit.

  87. #87 MartinB
    21. April 2015

    @Benjamin
    ” Formal ist es möglich”
    Nein, nicht ohne die Bedeutung der Begriffe “Energie” und “Impuls” – die ja nun mal in den EIT eingehen – zu verändern. Falls Sie drauf beharren, können Sie die Erklärung ja mal ausarbeiten, statt immer nur zu behaupten, dass es geht.

    “Wenn Sie einen konkreten physikalischen Einwand haben”
    Den kann ich erst haben, wenn Sie eine konkrete physikalische Theorie präsentieren, statt immer nur von einer tollen DGL zu erzählen…

  88. #88 Benjamin Packisch
    21. April 2015

    @Niels
    “Graviton und ART z.B. folgen m.E. unterschiedlichen Absichten.”
    Nämlich welchen?

    Das Graviton zielt ab auf eine ab initio Erkläung der Gravitation, d.h. eine lokale “bottom-top” Erklärung, und die ART zielt auf eine globale Erklärung, d.h. ausgehend vom Ganzen top-bottom – m.E. ein sehr sinnvoller Ansatz, aber eben nicht ganz leicht zu interpretieren.

    Auf ihre anderen Fragen will ich en bloc antworten (ART vs. Interpretationspraxis, Technik als “Beweis” für Theorien, Freiheit der Objektwahl in der Empirie etc.):
    Der Knackpunkt liegt darin, dass eine Theorie nur dann logisch streng im Experiment falsifizierbar ist, wenn absolute Klarheit bzgl. sämtlicher Einflussfaktoren besteht. Wie jeder Experimentator weiß, ist das niemals der Fall. Man ist hier prinzipiell auf mehr oder weniger wilkürliche Messkonventionen angewiesen, und auch auf leider nicht immer völlig bewußte Hypothesen. Alle konkreten physikalischen Theorien enthalten hypothetische Fiktionen (z.B. das Bohr-Atommodell, oder das quantenmechanische Wahrscheinlichkeitsfluid), die durch beständige Wiederholungen aber nicht selten einen schleichenden Realitätsanspruch erlangen.

    Laser, GPS etc. werden nun häufig als Theorie-Beweise angegeben, weil dies Geräte “ja funktionieren”. Wenn man nun mit Technikern spricht oder selber einer ist, der solche Geräte realisiert, dann kann man feststellen, dass technische Optimierung erschreckend unabhängig von der zugrundeliegenden Theorie ist. Beispiel GPS: Es gibt dort einfach einen Korrekturterm, der die Genauigkeit um ca. 50cm erhöht. Ob der als ART, SRT, “Ätherdrift”, “Sagnac-Korrektur” o.ä. bezeichnet wird, hat keine praktische Relevanz, es ist einfach ein Term von dutzenden, den man numerisch praktisch beliebig anders darstellen könnte, es aber traditionell nicht tut.

    Was allerdings technisch ziemlich relevant ist, ist Anschaulichkeit (oder wie Sie es etwas abschätzig nennen, ob eine Theorie “ontologisch zufriedenstellend” ist). Die ist natürlich nicht immer gegeben, aber sie hilft in der technischen Entwicklung ungemein, hat aber viel mit Erfahrung zu tun.

    Erfahrung ist es auch, die den Fokusunterschied zwischen Experimentatoren und Theoretikern erklärt. Sie ist nicht formalisierbar, sondern im Idealfall Teil der Konventionen, nach denen man misst und arbeitet. Wenn man die aber nicht regelmäßig kritisch hinterfragt und evtl. umwirft, dann kann man relativ weit daneben liegen, ohne es gleich zu merken.

    Es mir durchaus bewußt, dass diese Darstellung nicht der üblichen entspricht – genau darum weise ich hier daraufhin. Quellen dazu sind schwierig anzugeben, weil sie selten 100% akzeptabel sind, oder auch ungut konnotiert.

  89. #89 Benjamin Packisch
    21. April 2015

    @MartinB:
    Warum denn so heftig? Ich will Ihnen doch nichts Böses! Ein bisschen Polemik muss schon sein 😉
    Bloß wenn Ihnen das Prinzip schon so stark widerstrebt, dann ist es mit einem sinnvollen Dialog einfach schwierig – ähnlich wie mit Ihrem Schamanen und Rogant. SIe kriegen Ihre DGL, ich verspreche es Ihnen, unter zwei Bedingungen: wir klären ihre Einwände gegen euklidische, d.h. ART-mäßig ungekrümmte und kräftefreie Verhältnisse außerhalb der Grenzfläche haben und Sie gedulden sich bis zum Wochenende. Einverstanden? Wenn Sie gegen ersteres keine Einwände haben, geht es vielleicht etwas schneller.

  90. #90 MartinB
    21. April 2015

    @Benjamin
    “Warum denn so heftig?…. Ein bisschen Polemik muss schon sein”
    Und wieder ein Ironiemeter explodiert.

    ” SIe kriegen Ihre DGL”
    Ich will keine bloße DGL – Sie haben eine DGL versprochen, die auf der basis des EIT interpretiert werden kann.

    “unter zwei Bedingungen”
    Darf ich daraus schließen, dass sie die Theorie der Lichtablenkung an Linsen durch Raumzeitkrümmung in Wahrheit noch gar nicht ausgearbeitet haben?

    ” wir klären ihre Einwände gegen euklidische, d.h. ART-mäßig ungekrümmte und kräftefreie Verhältnisse außerhalb der Grenzfläche haben”
    Wenn Sie einen EIT heranziehen wollen, dann wird der wohl kaum an der Grenzfläche mit einer delta-Distribution beschrieben sein – sonst ist es nicht der EIT.

    Nochmal: Wenn Sie sagen, Sie können eine Theorie der Lichtablenkung durch Raumzeitkrümmung mithilfe des EIT aufstellen, dann muss diese Theorie folgendes leisten:
    1. Die lichtablenkung erklären (und zwar korrekt abhängig von der Lichtwellenlänge), und auch für andere Glaskörper wie Prismen (wir wollen ja nicht für jedes einzelphänomen ne komplett neue Erklärung)
    2. Als “Quellterm” für die Lichtablenkung den EIT verwenden – und das muss der handelsübliche EIT sein, nicht irgendein mathematisches Gebild mit anderen Eigenschaften, das Sie nur so nennen
    3. Die theorie muss erklären, warum wir die Raumzeitkrümmungseffekte nur beid er Lichtablenkung beobachten und nicht z.B. bei der Schallausbreitung. Und natürlich, welchen Einfluss z.B. geänderte Maßstäbe durch die Raumkrümmung z.B. auf die Dichte unserer Linse haben, oder auf Objekte, die sich direkt neben der Linse befinden.

    Wenn Sie das konsistent liefern können, dann können wir weiterreden – die DGL, die sich dann aus der Theorie ergibt, ist dabei wirklich der kleinste Teil des ganzen.

    (Sie sollten allerdings überlegen, die Theorie dann lieber bei Phys Rev Lett o.ä. einzureichen statt sie im Internet zu posten…)

  91. #91 EnnoA
    Hamburg
    21. April 2015

    Den Leuten, die die Formeln hinschreiben können, fehlen in der Regel die didaktischen Fähigkeiten und Leuten mit didaktischen Fähigkeiten fehlt oft das Fachwissen.
    Ich denke, es gibt aber ein großes Bedürfnis, dass jemand die abstrakte Formelsprache in anschauliche Vorstellungsbilder umsetzt.
    Erst dann hat man die Physik verstanden, und das gilt auch für die Physiker selbst. Wenn diese also nicht in der Lage sind, ihre Formeln ausführlich zu erklären und zu interpretieren, dann fehlt möglicherweise sogar das tiefergehende Verständnis.

  92. #92 Benjamin Packisch
    21. April 2015

    @MartinB.
    “Warum denn so heftig?…. Ein bisschen Polemik muss schon sein”
    Und wieder ein Ironiemeter explodiert.

    Bei wem?
    Sie wollen wohl eine DGL, und wenn ich das hier gegen Ihren Widerstand schaffe, dann ist PRL ja wohl nur noch Formsache 😉
    Nur vorab: Natürlich nähere ich mich diesem Thema natürlich nicht komplett “jungfräulich”, sonst würde ich mich wohl kaum “zu den Drachen” wagen, “sportlich” würde ich es nennen.

    Zur Sache:
    0. Ausdehnung: Warum darf Ihrer Meinung der EIT nicht lokal begrenzt sein, genauer: mit endlicher Reichweite, aber nicht singulär?
    1. Wellenlänge: wie gesagt, parametrisch unkritisch.
    2. Handelsüblicher EIT: Ab wieviel Eingängen != 0 ist er für sie denn “handelsüblich”?
    3. Raumkrümmung “für alle”: Schall wird doch wohl recht erheblich beeinflusst am Übergang von Luft zu Glas z.B. Und die Dichte ändert sich eben an der Grenzfläche – warum wird in der üblichen Optik ja nicht mal gefragt.Diesen Punkt halte ich daher für nachgeordnet.

  93. #93 MartinB
    21. April 2015

    @Benjamin
    “Bei wem?”
    Bei mir – im selben Absatz zu sagen “Warum so heftig” und “ein wenig Polemik muss sein” ist schon etwas witzig, um es vorsichtig zu sagen…

    “Ausdehnung: Warum darf Ihrer Meinung der EIT nicht lokal begrenzt sein, genauer: mit endlicher Reichweite, aber nicht singulär?”
    Weil der EIT definitionsgemäß z.B. als 00-Komponente die Energie/massendichte enthält, die wird in einer Linse wohl laum auf die Grenzfläche beschränkt sein.

    1. Ne, nicht “parametrisch unkritisch” – eine Theorie muss die Wellenlängenabhängigkeit auf ein physikalisches Phänomen zurückführen, nicht bloß nen fiddle factor einbauen.

    “2. Handelsüblicher EIT:”
    Der EIt ist handelsüblich, wenn die 00-Komponente die Massen/Energiedichte ist und die anderen Komponenten entsprechend die Energie- und Impulsströme/Spannungen. So ist er definiert.

    “Schall wird doch wohl recht erheblich beeinflusst am Übergang von Luft zu Glas z.B. ”
    Ja, aber nicht so wie Licht. Wenn die Lichtablenkung bloß durch Raumkrümmung kommt, dann krümmt sich der Raum ja auch für den Schall und auch für z.B. alpha-teilchen, die ich durch die Linse schicke.

    So, aber wir brauchen nicht weiter zu diskutieren – Sie präsentieren/veröffentlichen Ihre Theorie, dann reden wir weiter.

    @EnnoA
    “Wenn diese also nicht in der Lage sind, ihre Formeln ausführlich zu erklären und zu interpretieren, dann fehlt möglicherweise sogar das tiefergehende Verständnis.”
    Manchmal fürchte ich das auch, aber vielleicht hat auch jede ihre eigene Vorstellung, was anschaulich ist und wieviel Formeln noch o.k. sind.

  94. #94 sffd
    21. April 2015

    @Martin: Was meinen Sie mit “nur eine einzige Physik”? Logisch kann es nur eine einzige Physik geben. Wann wurden z.B. Verletzungen der Prinzipien der modernen Physik wie unitarität oder Lorentz Invariantz gemessen? Noch nie. Und so bleibt es auch. Die Physik ist eine so “erwachsene” Wissenschaft das man sie mit der Biologie und Evolutionstheorie, in diesem Sinn Stringtheorie vergleichen kann. Und sie verstehen die S Matrix Theorie nicht. Sie lokal in einer flachen Raumzeit das gleiche wie eine Feldtheorie. In Sinne des logischen Positivismus, mit desen Hilfe man mit Computern die wichtigsten bedeutungsvollen Daten berechnet kommt man zu diesem Schluss. Der Rest wird wegrationalisiert.

  95. #95 Benjamin Packisch
    21. April 2015

    @MartinB.
    ist schon etwas witzig, um es vorsichtig zu sagen…
    Na bitte, eine gewisse Humorspanne gibt es doch auch bei Ihnen ;-).
    Jetzt bin ich am Zug. Zwecks Punkt 0 (“Ausdehnung”) warne ich Sie nur vor. Die Energie/Massendichte in 00 ist für das Linsenbeispiel sehr schön nutzbar in Sachen “Selbstenergie”. Diesen Terminus diabolicus kennen Sie ja sicher ebenfalls…

    Bis bald
    BP.

    @sffd:
    Das noch “nie” irgendetwas gemessen wurde, war noch NIE ein Beweis für oder gegen etwas. Sowas kann immer an der Messmethodik/-absicht /-apparatur liegen.

  96. #96 MartinB
    21. April 2015

    @sffd
    “Logisch kann es nur eine einzige Physik geben.”
    Warum sollte das Universum eindeutig sein? Warum sollte ein anderes Universum logisch unmöglich sein?
    “Sie lokal in einer flachen Raumzeit das gleiche wie eine Feldtheorie. ”
    Vielleicht kam meine Bemerkung etwas zu heftig rüber – ich fand einfach die Aussage, eine Theorie, die sich angeblich auf Wechselwirkungen von Dingen, die aus dem Unendlichen kommen, beschränkt, könne unser Universum beschreiben, unglücklich formuliert. (Dass das in guter Näherung sinnvoll ist, bestreite ich ja gar nicht.)

    ” Die Physik ist eine so “erwachsene” Wissenschaft das man sie mit der Biologie und Evolutionstheorie, in diesem Sinn Stringtheorie vergleichen kann”
    Was kann ich womit vergleichen? Die Stringtheorie mit der Evolutionstheorie? Das halte ich für arg übertrieben, angesichts der Tatsache, dass es für die ET überwältigend gute experimentelle und beobachtete Evidenz gibt.
    Klar hat die Stringtheorie viele theoretische Ergebnisse vorzuweisen, aber eine experimentelle Bestätigung steht ja noch in jeder Hinsicht aus (und wenn am CERN nicht bald ein paar SuSy-Teilchen auftauchen, dann wird es wohl knifflig).

    “In Sinne des logischen Positivismus, mit desen Hilfe man mit Computern die wichtigsten bedeutungsvollen Daten berechnet kommt man zu diesem Schluss.”
    Und den Satz habe ich schlicht nicht verstanden

  97. #97 sffd
    21. April 2015

    Jungs. Im logischen Positivismus, der seit Jahrhunderten (Anfang bei Ernst Mach und Wiener Zirkel später) bestätigt ist (auch A.E. war ein Fan davon), haben Dinge in der Mathematik und Logik, die gleiche Bedeutung wie empirische Experimente. Nur weil unserer Sprache die Eigenschaft der dikreten Unendlichkeit hat, bedeutet das nicht das alles eine Bedeutung hat das man so rausplappert, jedenfalls nicht in der Physik. Und das wird im Formalismus der logischen Positivismus herausgefiltert, am Ende ergibt das die S-Matrix Theorie die mit Reggae Theorie zur moderne Stringtheorie wird bei der Strings als schwarze Löcher interpretiert werden mit extrem vielen Zuständen, damit bekommt man nämlich das Problem wie man in einer nicht definierten Raumzeit Messungen auf kleinen Skalen hinbekommt gelöst (die alte Inkonsistenz zwischen QM und ART), eben durch Analyse mit logischen Positivismus. Die nächste Frage ist dann. Was würde es wohl bedeuten wenn es eine andere Theorie gäbe die genau das gleiche wie die Stringtheorie kann? Das ergibt keinen Sinn, was soll das sein? Es ist unmöglich das noch eine Theorie äquivalent zu zillionen Zuständen sein kann. Die Wahrscheinlichkeit ist zu gering.

    @Martin: Ich hab auch nicht gesagt das andere Universen unmöglich sind. Aber eine andere Physik ist unmöglich. Wir leben nicht bei Alice im Wunderland wo alles möglich ist. Die ST ist nur konsistent in einem bestimmten Rahmen. So wie z.B. eine Glasscheibe auf verschiedene weisen auf dem boden zerspringen kann, aber bei jeder art und weise eben die gleichen Naturgesetze gelten.
    Und die ST braucht auch keine Supersymmetrie. Der natürlich Ort von strings liegt auf praktisch unmessbaren Skalen. Und jede Theorie die das leisten kann was die ST leistet hat das selbe Problem. Das lustige ist ja das man immer sagt, ja die ST hat in 40 Jahren noch keine überprüfbaren (im Popper Wahnsinn) Vorhersagen gemacht. Aber seit 40 Jahren hat es auch noch keine andere Theorie geschafft eine Formel für die Streuamplitude von Gravitonen zu liefern, in der Stringtheorie nämlich die Virasoro-Shapiro Amplitude. So eine Formel ist ja wohl das mindeste das eine Theorie der QG haben sollte. Trotzdem konnte das bis jetzt keine ausser der ST. Und aus der Erfahrung von fast einem Jahrhundert Physik weiß man das es keinen Sinn macht die alten Prinzipien weg zuwerfen, jeder der das versucht hat in all den Jahren hatte irgendwann eine inkonsistente Theorie. So funktioniert auch Wissenschaft nicht, man wirft nicht gefestigte Erkenntnisse der letzten Jahrzehnte weg, das passiert so gut wie nie. Man baut auf alten Erkenntnissen auf. (On the shoulders of giants). Und man sieht auch keine wissenschaftlichen Arbeiten in denen das Argument “Seit 40 Jahren keine Ergebnisse” ernst genommen wird, das gilt nur bei populären Stammtisch Diskussionen.

  98. #98 sffd
    21. April 2015

    Weitere Beispiele für Erfolge des logischen Positivismus:

    Quanten Mechanik
    Äthertheorie (Verbannung, er ist unmessbar, also nicht existent)
    Äquivalenzprinzip
    (Es ist unmöglich zwischen den Effekten einer Beschleunigung und den Effekten der Gravitation zu unterscheiden, also ist beides gleichwertig, so wie jede Aussage über den Geschmackt des Grundgesetztes gleichwertig bedeutungslos ist).
    Stringtheorie(wie oben gesagt)
    Holographisches Prinzip
    (Welches das Prinzip der Quantengravitation ist, es ist analog zu dem Äquivalenzprinzip in der ART)

    USW.

  99. #99 sffd
    21. April 2015

    Und es gibt viele gute QG Theorie. Zum Beispiel QG in 2D und 3D. Dort ist sie komplett verstanden.

  100. #100 MartinB
    22. April 2015

    @sffd
    ” Im logischen Positivismus, der seit Jahrhunderten (Anfang bei Ernst Mach und Wiener Zirkel später) bestätigt ist ”
    ??? Das halte ich für ne ziemlich starke Aussage. Ist ja nicht so, als wäre logischer Positivismus die einzige Denkmöglichkeit.
    Und gerade die Qm mit all ihren Problemen (ist die Wellenfunktion deiner Ansicht nach existent?) als Beispiel dafür heranzuziehen, scheint mir noch gewagter.

    “Und das wird im Formalismus der logischen Positivismus herausgefiltert, am Ende ergibt das die S-Matrix Theorie die mit Reggae Theorie zur moderne Stringtheorie wird ”
    Huh? Der logische Positivismus führt direkt zur Stringtheorie – oder ich verstehe den Satz falsch…

    ” Was würde es wohl bedeuten wenn es eine andere Theorie gäbe die genau das gleiche wie die Stringtheorie kann?”
    Mir wäre eine Theorie lieber, die mehr kann – zum Beispiel experimentell überprüfbare Vorhersagen…

    ” Das ergibt keinen Sinn, was soll das sein? Es ist unmöglich das noch eine Theorie äquivalent zu zillionen Zuständen sein kann. Die Wahrscheinlichkeit ist zu gering.”
    Ist es nun unmöglich oder ist die Wahrscheinlichkeit zu gering?
    Und die Tatsache, dass die Stringtheorie Zillionen Möglichkeiten auswirft, halte ich nicht gerade für eine Stärke der Theorie.

    “Aber eine andere Physik ist unmöglich.”
    Warum soll das so sein? Nur weil sie in unserem Universum nicht gegeben ist? Oder weil man sagt: eine unserer 10 hoch 500 Varianten wird schon passen?

    “Und die ST braucht auch keine Supersymmetrie.”
    Ich dachte, alle gängigen Stringtheorien würden Superstrings verwenden?

    ” Aber seit 40 Jahren hat es auch noch keine andere Theorie geschafft eine Formel für die Streuamplitude von Gravitonen zu liefern,”
    Nö. Aber die ST sagt ne Menge Dinge vorher, die wir eben nicht beobachtet haben – diverse Teilchen und Extra-Dimensionen zum Beispiel. Oder man nimmt die 10-hoch-500-Variante, dann sagt die Theorie gar nichts mehr vorher, weil alles geht.

    Sorry, wenn es etwas sarkatsisch rüberkommt – aber deine Behauptungen klingen ja fast so, als wäre jeder, der keine ST macht, von vornherein auf dem Holzweg, das halte ich für ziemlich absurd.

  101. #101 sffd
    22. April 2015

    @Martin: 10^500 ist übrigens keine große Zahl, es ist eine kombinatorische Zahl. Ein Produkt aus 500 Dingen die man auf 10 verschiedene Arten anordnen kann. Oder zum Beispiel 500 Zahlen experimenteller Daten. Diese Zahl wurde in den 60iger von Feldtheorie befürwortern (S-Matrix Gegner) zu Propagandazwecken so geschrieben damit sie groß aussieht (gab auch stringpropaganda).

    Zur QM, les mal das Buch von Paul Dirac zur QM, die wahre Natur der QM ist durch Analyse mit logischem Positivismus nicht erfahrbar, darum sind auch alle Interpretationen gleichwertig, es ist nur der Wechsel des “frameworks”. Wenn man z.B. die Wahrscheinlichkeitsinterpretation akzeptiert dann fragt man normalerweise nicht danach ob die Wellenfunktion existiert oder nicht, man akzeptiert es einfach und hinterfragt es nicht weil die Frage bedeutungslos ist. Eben wie die Frage welche Farbe die Eifersucht hat, lediglich ein Artefakt unserer Sprache, durch die Eigenschaft der diskreten Unendlichkeit können wir eben unwissenschaftliche Sätze konstruieren die in der Physik nicht helfen, diese Frage nach der Natur der QM sind einfach bedeutungslose Symbole aneinandergereiht.
    Das Problem mit den vielen Möglichkeiten in der ST kommt auch daher das man nicht wie in der QFT viele Parameter hat die man einstellen muss (noch viel mehr Möglichkeiten als in der ST), sondern weil es in der ST keine Parameter mehr gibt, man hat nur Lösungen. Jede Theorie die die ST ersetzen soll hat solche Probleme. Aber ich warte jetzt erst mal auf die Ergebnisse beim LHC, wird sehr spannend was da passiert in den nächsten Monaten, er läuft ja schon seit ein paar Wochen.

  102. #102 sffd
    22. April 2015

    Ahso, ja die gängisten sind natürlich supersymmetrisch. Aber es gibt auch ST ohne Supersymmetrie.

  103. #103 MartinB
    22. April 2015

    @sffd
    “Wenn man z.B. die Wahrscheinlichkeitsinterpretation akzeptiert dann fragt man normalerweise nicht danach ob die Wellenfunktion existiert oder nicht, man akzeptiert es einfach und hinterfragt es nicht weil die Frage bedeutungslos ist. ”
    Was genau meinst du mit “man akzeptiert es einfach”? Was akzeptiert man? Dass man mit Objekten hantiert, deren ontologischer Status vollkommen unklar ist?
    Wahrscheinlich gibt es verschiedene Spielarten des Positivismus, aber ich hatte de Ideen von Mach immer genau so verstanden, dass Theorien solche Objekte wie Wellenfunktionen, die selbst keiner Messung zugänglich sind, nicht enthalten sollten.

  104. #104 MartinB
    22. April 2015

    Achso noch ne ganz doofe Frage:
    “Aber es gibt auch ST ohne Supersymmetrie.”
    Fallen die auch unter die 10 hoch 500 möglichen Universen?

  105. #105 Adent
    22. April 2015

    @Martin B
    Sffd war neulich schon bei Florian zu Gast und hat dort ähnliches verbreitet, am Ende der Diskussion (nein eigentlich schon am Anfang) waren alle zu blöd zu kapieren, dass es die Weltformel längst gibt, nämlich in Form der Stringtheorie (wobei nicht genau klar wurde welche genau er von den vielen meint 😉 ).
    Es hat wenig Zweck mit dem zu diskutieren, da du einfach zu dumm bist, ich schon sowieso, da ich kein Physiker bin. Seltsamerweise fand ich aber die Argumente von Florian und anderen einleuchtender (wahrscheinlich weil ich zu dumm bin).

  106. #106 Benjamin Packisch
    Licht-Refraktion an einer Linse im ART-Formalismus
    22. April 2015

    Es ging doch schneller. Denn:
    Hier (VL-Skript Prof. Main, Stuttgart) [1] Kap. 6.1.6 ist die Sache weitgehend durchgerechnet.

    1. Zur formalen Zulässigkeit des ART-Formalismus an einer optischen Linse

    Mit [1] ist zunächst klargestellt, dass es formal möglich ist, Lichtbeugung im Rahmen der ART zu beschreiben, und zwar auch in der Form eines Brechungsindexes.

    Zu klären ist für die Belange hier Folgendes: Wie lässt sich die (relativistische) Streuung an einem sphärischen G-Potential auf Streuung an einer ebenen Dichte grenze übertragen?

    Der Einfachheit betrachten wir nur den dynamisch interessanten Teil, und das ist die einfache Lichtbrechung an einer ebenen Dichtegrenze, d.h. dem scharfen Übergang von Vakuum zu Materie.
    Die prinzipielle Lösung ist die Annahme eines ebenen Potentials, das sich nur in der z-Richtung senkrecht zur Grenze hin ändert. Wenn z dem r im Beispiel entspricht, dann ist diese Änderung qualitativ identisch zu den Graustufen in Abb. 6.6 [1] für eine Richtung zum Zentrum hin.
    Abseits der Grenze nehmen wir weiter konstantes Potential an, d.h. in der Materie ebenfalls.

    Eine exakte Lösung ist dann eine quantitative Anpassung der Analytik in [1] sein. Sie ist symmetrisch sogar auch für den Austritt des Lichtstrahls aus der Linse ins Vakuum anwendbar, und auch für kontinuierlich gekrümmte Grenzflächen.
    Aus meiner Sicht gibt es kein formales bzw. qualitatives Argument, dass eine solche Anpassung verbieten würde. Die Raumzeit ist außerhalb des Potentialgradienten flach, d.h. euklidisch und damit aus Sicht des Photons kräftefrei.
    Als Einstieg in die präzisere Analytik würde ich das Potential als erf(z) annehmen, bzw. eine geglättete Heaviside-Funktion.
    Physikalisch entscheidend ist, dass die Ablenkung des Lichtstrahls proportional zum Potentialgradienten ist – aber nicht zum Absolutwert des Potentials.
    Soviel zur mathematischen Form, die man natürlich noch weiter verfeinern könnte, aber hier soll mal der Hinweis auf die Gleichungen (6.85) & (6.82) in [1] sowie die Vorarbeiten dort genügen.

    2. Zur physikalischen Plausibilität des Ansatzes

    Mit welchen Argumenten können wir eine solche Potentialverteilung physikalisch begründen? Und was spricht dagegen?
    Zunächst zu den Gegen-Argumenten.
    Es spricht gegen die Linsen-Modellierung per ART
    1) dass eine optische Linse von der Größenskala etwas völlig anderes ist, als das, was man üblicherweise mit der ART berechnet, z.B. kosmische Gravitationslinsen, und folglich eine Übertragung in die Anschaulichkeit verfehlt sei;
    2) dass man geometrische Optik üblicherweise nicht relativistisch berechnet, sondern phänomenologisch, und daher gravitative Einflüsse nicht berücksichtigt, vom Energieverlust an Photonen in vertikaler Richtung einmal abgesehen;
    3) dass man üblicherweise annimmt, dass Raumkrümmmung sehr stark sein muss (oder die Längenskala sehr groß), um ART-Effekte messen zu können, und wenn man sie misst, dass dann nicht isoliert ein Phänomen wie die Lichtausbreitungsrichtung betroffen ist,
    sondern eine ganze Bandbreite von atomaren, mechanischen und elektromagnetischen Begleiteffekten anzutreffen seien.

    Für die Modellierung einer optischen Linse analog einer Gravitationslinse spricht demgegenüber:
    1) Die ART ist formal eine geometrisch-phänomenologische Theorie, d.h. sie trifft keine kausale Aussage über die Gravitation selbst, sondern nur über dessen Identität mit der Raumgeometrie und die daraus folgenden Null-Beschleunigungsorte. Es gibt folglich keinen physikalisch zwingenden Grund, diesen Ansatz auf eine Skala zu beschränken. Wohlgemerkt ist die Beschreibung der Gravitation auch sonst primär phänomenologisch (wenn man mal vom Higgs-Teilchen absieht), d.h. wir haben lediglich ein Kräftepostulat aufgestellt, wie in der Gravitation üblich. Auch Newtons Himmelsmechanik
    ist bei genauer Betrachtung eine Umformulierung der Geometrie des 3. Kepler’schen Gesetzes.
    2) Die geometrische Optik ist ebenso geometrisch-phänomenologisch, d.h. auch sie macht keine kausale Aussagen über die Ursache der Lichtbrechung, geschweige denn dessen kausale Relation mit der Material-Lichtgeschwindigkeit.
    3) Mit welchem (physikalischen) Recht und wieviel Sinn darf man also die Phänomenologie der Optik mit der der ART verknüpfen?
    Warum sollte das Schwerepotential vom Vakuum ins Linsenmaterial markant zunehmen – und nicht die üblichen elektromagnetisch-molekularen Effekte für die Brechung verantwortlich sein?
    Die Frage ist, ob wir Sekundäreffekte folgern müssen, die von den bekannten Lichtbrechungserscheinungen abweichen, wenn wir so eine Potentialverteilung annehmen.
    Grundsätzlich ist es Tatsache, dass in unmittelbarer Grenzflächennähe markante Effekt auftreten. Üblicherweise interpretiert man sie aber elektromagnetisch-molekular und nicht geometrisch oder gravitativ.

    M.E. spricht aber nur die physikalische Gewohnheit gegen eine Uminterpretierung. MartinB legt hier Wert auf die Beachtung der Wellenlängenabhängigkeit dabei. Ich will sie also diskutieren.

    Was man dazu braucht, ist einen Schweregradienten in der Ausdehnung der Lichtwellenlänge, sowie eine materialabhängige Schwerefeldmagnitude.
    Ersteres erklärt zwanglos, dass für kurze Wellenlängen die Wechselwirkungsintensität stärker ist als für lange (weil mehr Wechselwirkung pro vollem Wellenzug). Es ist der gleiche Ansatz wie bei den sonst üblichen Streumechanismen (Mie etc.).
    Zweiteres benötigt die Forderung, dass das Schwerepotential in einem Material z.B. dichteabhängig ist – auch das m.E. kein Hexenwerk und nicht “verboten” als Ansatz.

    Ergo: Wenn wir eine axiomatische Kühnheit ähnlich der der Physik der 1920er gestatten, dann ist eine optische Linse durchaus mit dem Formalismus der ART beschreibbar.

    Ob das mehrheitsfähig, praktikabel oder sexy genug für PRL o.ä. ist bezweifle ich zwar, aber es ist logisch und physikalisch möglich. Möge es den Lesern hier – unabhängig von ihrer Meinung – Erkenntnis ermöglichen. Wenn jemand damit reich und berühmt wird freut es mich ebenso, noch mehr aber wenn es sachdienliche Kritik gibt.

    Viele Grüße
    BP

    P.S.: Vielen Dank an MartinB für das beständige Interesse an der Sache und den Raum hier im Forum!

    P.P.S.: Noch eine latent häretische Geometrie-Bemerkung als Anhang, zum Zentralkörperproblem: Die Lösungs-Ellipsen lassen sich drehimpulskonform als Überlagerung zweier gekoppelter und je gleichmäßig schneller Kreisbewegungen parametrisieren – z.B. eines “Epizykel” auf einem “Deferentenkreis” – aber das ist ein anderes Mode-Thema 😉

  107. #108 Niels
    22. April 2015

    @Benjamin Packisch
    Ist das hier eigentlich von dir?

    https://www.kritik-relativitaetstheorie.de/2013/05/offizielles-fazit-loser-stecker-zu-den-neutrino-experimenten-am-cern-ist-fragwurdig/

    https://www.kritik-relativitaetstheorie.de/Anhaenge/Untersuchung-NeutrinoAnomlieBeiOPERA_Packisch2012b.pdf

    Der Knackpunkt liegt darin, dass eine Theorie nur dann logisch streng im Experiment falsifizierbar ist, wenn absolute Klarheit bzgl. sämtlicher Einflussfaktoren besteht. Wie jeder Experimentator weiß, ist das niemals der Fall.

    Deiner Meinung nach kann man physikalische Theorien also prinzipiell nicht falsifizieren?

    Wenn man nun mit Technikern spricht oder selber einer ist, der solche Geräte realisiert, dann kann man feststellen, dass technische Optimierung erschreckend unabhängig von der zugrundeliegenden Theorie ist.

    Das ist ziemlich sicher falsch. Man hätte zum Beispiel niemals Halbleitertransistoren unabhängig von Quantenmechanik und SRT “realisieren” können, und damit wäre die gesamte heutige Elektronik unmöglich.

    Ob der als ART, SRT, “Ätherdrift”, “Sagnac-Korrektur” o.ä. bezeichnet wird, hat keine praktische Relevanz, es ist einfach ein Term von dutzenden, den man numerisch praktisch beliebig anders darstellen könnte, es aber traditionell nicht tut.

    Versteh ich nicht. Der Korrekturfaktor ist natürlich ein Zahlenwert. Was soll da “numerisch praktisch beliebig darstellbar” bedeuten?

    Der Witz ist doch natürlich, dass die ART diesen Zahlenwert richtig vorhersagt.

    @sffd
    Bist du sdgdfsg von hier ab #35?
    https://scienceblogs.de/astrodicticum-simplex/2015/02/11/neue-quantenmechanische-gleichungen-schaffen-singularitaeten-ab-gab-es-doch-keinen-urknall/

  108. #109 Niels
    22. April 2015

    @Benjamin Packisch
    #106 sehe ich erst jetzt.

    Sorry, aber so einen Quatsch hab ich schon lange nicht mehr gelesen.

  109. #110 MartinB
    22. April 2015

    @Benjamin
    Phew, für einen Moment dachte ich, ein echter Physikprof würde behaupten, man könne Linsen mit der ART beschreiben, aber ich sehe, dass der eigentliche Text von dir ist.

    “Die prinzipielle Lösung ist die Annahme eines ebenen Potentials,”
    Was soll das für ein potential an der Grenzfläche der Linse sein? ich dachte, das sollte mit dem EIT gehen?

    ” Die Raumzeit ist außerhalb des Potentialgradienten flach, d.h. euklidisch und damit aus Sicht des Photons kräftefrei.”
    Ich verstehe gar nichts. Die RZ soll an der Grenzfläche der Linse nicht flach sein, innerhalb der Linse aber schon? Was soll dass mit dem Formalismus der ART zu tun haben?

    “Was man dazu braucht, ist einen Schweregradienten in der Ausdehnung der Lichtwellenlänge, sowie eine materialabhängige Schwerefeldmagnitude.”
    Wie soll das gehen? Wenn ich rotes Licht draufstrahle, ändert sich plötzlich die Raumkrümmung an der Linse? Und wenn ich weißes Licht strahle, was dann? Jedes Photon bekommt seine eigene RZ oder wie?

    Die Textwand erfüllt leider nicht mal ansatzweise die geforderten Kriterien. Kurz gesagt schließe ich mich Niels Kommentar an.

    @Adent
    Danke für den Hinweis; manchmal frage ich mich schon, was Stringtheoretikerinnen so rauchen…

  110. #111 MartinB
    22. April 2015

    @Niels
    Danke für den Link auf die Diskussion bei Florian. Wow.

  111. #112 sffd
    22. April 2015

    @Martin: Man akzeptiert einfach das man nie wissen wird was eine Wellenfunktion ist. Shut up and calculate. Jedenfalls in der Wahrsch. int.. In der Everett interpretation kann man viele Sachen hinzufügen und sie interpretieren. Sie ist aber der wahr. int. gleichwertig. Was ontologie usw. angeht, das ist nur bedeutungsloses Gequatsche im log. Positivismus. Der ja auch, anders als bei Mach, konsistente Mathematik und Logik zulässt. Mach bezieht sich nur auf sensorische Erfahrung.

  112. #113 Benjamin Packisch
    22. April 2015

    @Niels
    Ist das hier eigentlich von dir?
    Nicht direkt, auch wenn mein Name draufsteht. ich habe nichts gegen die RTen. Würde es denn Dein Urteil über meine Ausführungen hier berühren, pars pro toto und so? Was kannst Du beisteuern, dass ich Dich besser beurteilen kann, ohne zu lesen was Du hier schreibst ;-)?

    Deiner Meinung nach kann man physikalische Theorien also prinzipiell nicht falsifizieren?
    Hart, aber leider “ziemlich sicher” wahr, in Deinen Worten. Duhem-Quine vs. Popper hilft in dieser Sache. Was bleibt, sind Konventionen. Für quasireligiöse Sicherheiten taugt die Physik wenig, wozu auch.

    Das ist ziemlich sicher falsch. Man hätte zum Beispiel niemals …
    1+1=3-1=68/34=sqrt(4)=… =2. Kannst Du einer 2 ansehen, wie sie entstanden ist? Ich nicht. Ich kann nur vermuten, wie sie “gemeint” sein könnte.
    Zahlen beweisen daher gar nichts, wenn dann nur die geometrische Idee dahinter, und bei numerischen Wunderwerken wie u.a. dem GPS interessiert nicht mal die, sondern nur die Brauchbarkeit. Das Problem ist zugegebenermaßen, wie man vernünftig die richtige von der falschen Idee unterscheidet. Freund Computernumerik macht das leider sehr, sehr schwerig…

  113. #114 Benjamin Packisch
    22. April 2015

    @MartinB & Niels.
    Also zwingen kann ich niemanden, zu durchdenken, was ich da vorgestellt habe. Aber ich muss festhalten, dass Euer Wille offenbar tiefer liegt als Eure Erkenntnisbereitschaft.
    Nun gut, es bleibt ein Angebot für die Freidenker unter den Lesern hier.

    Noch ein sachlicher Versuch:
    Warum kann man Ihrer Meinung nach ein ebenes Potential analog dem über einer ausgedehnten Kondensatorplatte nicht im Rahmen einer ART-Metrik darstellen?

  114. #115 sffd
    22. April 2015

    Benjamin redet hier z.B. ganz viel bedeutungslosen Unsinn ( wenn es zur harter berechenbarer Physik komm).

  115. #116 sffd
    22. April 2015

    ich bin kein stringtheoretiker. Könnt ihr alles im ncafe nachlesen und in ent. Literatur.

  116. #117 MartinB
    22. April 2015

    @Benjamin
    “Also zwingen kann ich niemanden, zu durchdenken, was ich da vorgestellt habe”
    Als erstes müsstest du es mal selbst durchdenken, was offensichtlich nicht der Fall war.

    “Warum kann man Ihrer Meinung nach ein ebenes Potential analog dem über einer ausgedehnten Kondensatorplatte nicht im Rahmen einer ART-Metrik darstellen?”
    Die Frage verstehe ich nicht. Ist die Frage, ob man ein unendlich ausgedehntes konstantes Schwerefeld in der ART haben kann so wie das E-feld einer unendlichen Leiterplatte konstant ist? Das ist nicht so einfach siehe
    https://arXiv.org/abs/0708.2906v1

  117. #118 Benjamin Packisch
    22. April 2015

    @MartinB
    Vielen Dank für den Link. Dass es analytisch subtil ist bestreite ich nicht, aber schon, dass es unmöglich ist. Gl. (5) und (24) links (“Brane world”) in der Arbeit gehen doch genau in die oben skizzierte Richtung.

    @sffd: Ruhig Blut, mein Guter. Logischer Positivismus ist eine feine Sache, und ich bin sogar Deiner Meinung, dass es nur eine wahre Physik gibt – aber trotzdem sehr, sehr viele Beschreibungsmöglichkeiten.
    Wenn Du ansonsten etwas Konkretes beizutragen hast, bitte gern.

  118. #119 sffd
    22. April 2015

    @Benjamin: Die verschiedenen Beschreibungen die du meinst habe ich oben schon erklärt. Es gibt diese Redundanzen z.B. in Form von Eichtheorien. Das ist nix mysteriöses. Wie ich schon geschrieben habe, hat das mit unserer Sprache zu tun und der Eigenschaft der diskreten Unendlichkeit. Z.b. kann ich sagen “Warum? Warum? Warum? Warum? Warum? Warum?….ect.” irgendwann macht es keinen Sinn mehr logisch und experimentell. Der logische Positivismus ist ein Werkzeug mit dem man diese Redundanzen findet und eliminieren kann um Computermodelle auf das wesentliche zu reduzieren. Und die Physik die man auf einem Computer Modellieren kann, das ist auch die richtige Physik.

  119. #120 sffd
    22. April 2015

    Und nicht irgendwelche Objekte die eine höhere Kardinalität als das Kontinuum haben z.B.

  120. #121 MartinB
    23. April 2015

    @sffd
    Meine Frage bei #104 war übrigens ernst gemeint.

  121. #122 Niels
    23. April 2015

    @Benjamin Packisch

    Nicht direkt, auch wenn mein Name draufsteht.

    Wie passiert einem denn so etwas?

    Würde es denn Dein Urteil über meine Ausführungen hier berühren, pars pro toto und so?

    Wenn jemand nicht an Falsifikation glaubt, aber diese OPERA-Geschichte als Beleg aufführt, dass die SRT falsch sein muss, ist das schon irgendwie seltsam.

    Außerdem bin ich mir nicht besonders sicher, dass ich mit jemand sinnvoll über Aussagen physikalischer Theorien diskutieren kann, der schreibt:

    Leider liefert die kritische Nachuntersuchung der offiziellen Berichterstattung zur NeutrinoAnomalie kein erfreuliches Bild der konventionellen Physik-Landschaft. Die Argumente des Establishments sind löchrig oder absurd und ihr Verhalten gegenüber der Öffentlichkeit grenzt ans Panische. Offenbar sind die OPERA-Ergebnisse politisch derart unter Druck, dass keine anschlussfähigen oder zukunftsweisenden Resultate mehr möglich sind.
    Die Mehrheit in der Physik-Community scheint anderer Meinung zu sein [z.B. 8], allerdings darf man hier wohl Befangenheit unterstellen.

    Als Fazit bleibt, dass die etablierten Forschungsinstitutionen ihrer Aufgabe offensichtlich nicht mehr gerecht werden. Es gibt folglich einen dringenden gesellschaftlichen Bedarf, das konventionelle Physik-Theoriegebäude einer sorgfältigen und grundsätzlichen Prüfung zu unterziehen. An Argumenten mangelt es jedenfalls längst nicht mehr.

    Wenn das dann auch noch ausgerechnet ausschließlich auf der Seite “Kritische Stimmen zur Relativitätstheorie” [link entfernt. MartinB] veröffentlicht wird, kann man sich nämlich ausmalen, wohin die Diskussion wahrscheinlich führt…
    Sorry, Niels, ich habe nen Link entfernt, sonst gibt es hier wieder ne Beschwerde an den Uni-Präsidenten (war zwar eher lustig, muss aber ja nicht sein.)

  122. #123 MartinB
    23. April 2015

    @Niels
    Habe deinen Kommentar editiert – es gab da mal wegen eines ähnlichen Links gleich ne Beschwerde an den Uni-Präsidenten (mit einem sehr netten Telefonat mit der Rechtsabteilung und ohne weitere Konsequenzen, aber man muss den Leuten ja nicht vollkommen unsinnig Arbeit machen…)

  123. #124 Niels
    23. April 2015

    Oh, klar. Tut mir leid, ich bin wirklich überhaupt nicht auf den Gedanken gekommen, dass dir das irgendwie Ärger machen könnte.

  124. #125 Benjamin Packisch
    23. April 2015

    @Niels
    Wenn das dann auch noch ausgerechnet ausschließlich auf der Seite “Kritische Stimmen zur Relativitätstheorie” [link entfernt. MartinB] veröffentlicht wird, kann man sich nämlich ausmalen, wohin die Diskussion wahrscheinlich führt
    (Hervorhebung vom mir). Aha, wohin denn?
    Der Artikel war von einem ehem. Kollegen von mir, der aber seinen Namen nicht preisgeben wollte. Der freut sich bestimmt, dass sein Text hier mal wieder “rezipiert” wird. Wenn Du eine “seriöseres” Publikationsorgan empfehlen kannst, auf das man ungelöscht verweisen kann, kann ich ihm das gern ausrichten ;-).
    Aber wenn gleich von Rechtsabteilung etc. die Rede ist, ohweh – das interessiert mich wirklich nicht, im übrigen auch nicht, ob die RTen makellos sind oder nicht (s.o.). Sie sind eine formale und konventionelle Realität, egal ob das einem gefällt. Was fangen wir damit an, darum geht es.

    Interessant finde ich Deine Vermutung, dass ich nicht “an Falsifikation” glaubte .
    In der Wissenschaft steht uns doch da ein etwas solideres Kriterium zur Verfügung, nicht “wahr”?
    Ich kann Dich nur warnen, den Kontext eines Sachverhaltes nicht mit seinem Wahrheitsgehalt zu identifizieren.

    @sffd
    Und die Physik die man auf einem Computer Modellieren kann, das ist auch die richtige Physik.
    Nein, leider muss ich Dir da widersprechen. Man kann sehr, sehr viel Blödsinn am Computer modellieren, so hübsch der auch sein mag, Physik ist das eher selten (von den Halbleitern darin mal abgesehen).

    @MartinB
    Ich freue mich auf weiterhin über konstruktive Kritik, insbesondere bzgl Ihrer Referenz und meinem Hinweis darauf. Oder sind Sie einverstanden soweit 😉 ?

  125. #126 Niels
    23. April 2015

    @Benjamin Packisch

    Der Artikel war von einem ehem. Kollegen von mir, der aber seinen Namen nicht preisgeben wollte.

    Ach, die RT-“Kritiker” fordern Echtname und kontrollieren den Perso, bevor man etwas veröffentlichen darf? 😉

    Wenn Du eine “seriöseres” Publikationsorgan empfehlen kannst, auf das man ungelöscht verweisen kann[…]

    Das hast du falsch interpretiert. Ich hab auf eine Einschätzung zur Betreiberin dieser Seite verlinkt.

    In der Wissenschaft steht uns doch da ein etwas solideres Kriterium zur Verfügung, nicht “wahr”?

    Es gibt ein wissenschaftliches Kriterium, um zu entscheiden, welche erkenntnistheoretische Position “wahr” ist?
    Welches denn?

    Du vertrittst die Duhem-Quine-These, richtig?
    Damit habe ich mich noch nie besonders tiefgehend auseinandergesetzt. Damit wir über das selbe reden:
    Würdest du der Beschreibung in der englischen Wiki zustimmen?
    Wie sieht es mit der Beschreibung hier aus?
    https://www.claus-beisbart.de/teaching/su2006/realism/underdet.pdf
    (Vertrittst du nach dieser Unterscheidung nur die Duhem-These oder das ganze Paket?)

    Verstehst du darunter etwas anderes und/oder kennst du eine Quelle, die diese These besser zusammenfasst?

    Übrigens bin ich nach der Beschreibung dieser beiden Quellen nicht sicher, dass die These tatsächlich aussagt, dass man die ART als ganzes nicht falsifizieren könne.

  126. #127 Benjamin Packisch
    23. April 2015

    …Publikationsorgan … ungelöscht verweisen kann[…]
    Das hast du falsch interpretiert. Ich hab auf eine Einschätzung zur Betreiberin dieser Seite verlinkt.

    Vielleicht gibt es Dir zur Denken, warum man diese Art “Einschätzung” hier nicht schätzt. Wohl weniger aus juristischer Angst vor Strafe, sondern weil sowas wissenschaftlich nicht weiterhilft. Da besitzt das Gesetz einen ganz guten Sinn.
    Aber bevor ich Deine vielen Fragen beantworte: Wohin führt es denn nun Deiner Meinung nach wahrscheinlich, wenn man sich im Kontext von RT-Kritikern nicht versteckt?
    Was genau passiert dann? Oder was fürchtest Du?

    Nur kurz zu Deinen Fragen:
    Die engl. Wikipedia-Seite zu Duhem-Quine finde ich in diesem Falle nicht so klar wie die deutsche: https://de.wikipedia.org/wiki/Duhem-Quine-These
    Der Beisbart-Artikel ist mir etwas zu wenig auf die physikalische Praxis bezogen und spart die heißen Fragen aus. Was bedeutet das Ganze z.B. für das Peer-Review-System oder die fast immer unselige Debatte zwischen RT-Freunden und -Feinden?
    Was meinst Du dazu, unabhängig von etwaigen Lager-Zugehörigkeiten?

  127. #128 MartinB
    23. April 2015

    @Benjamin
    “warum man diese Art “Einschätzung” hier nicht schätzt. ”
    ich habe den Link entfernt, weil eine gewisse Person nach einer ähnlichen Diskussion hier bei mir, in der der Name dieser Person gefallen ist, eine Beschwerde an den Uni-Präsidenten der TU BS geschrieben hat (die man auch mit wenig mühe im Internet finden kann). Obwohl sich das ganze mit einem einfachen Telefonat mit unserer Rechtsabteilung *für mich* erledigt hatte, haben die Leute dort besseres zu tun, als absurde Beschwerdebriefe zu beantworten – denn dort musste je jemand einen Antwortbrief verfassen.

    Inhaltlich stimme ich Niels vollkommen zu.

    (Und Debatten mit Leuten, die die SRT für Blödsinn halten, sind hier auf dem Blog auch eher unerwünscht.)

  128. #129 Niels
    23. April 2015

    @Benjamin Packisch

    sondern weil sowas wissenschaftlich nicht weiterhilft.

    Na ja, meiner Meinung nach sagt es schon ein bisschen was aus, wenn ausschließlich auf einer bestimmten Webseite publiziert wird. Da ist es zum Beispiel nicht völlig aus der Luft gegriffen, eine gewisse Übereinstimmung mit den Aussagen des Seitenbetreibers zu vermuten. Schließlich ist es ja nicht so, als wäre es heutzutage unmöglich, irgendwo anders eine pdf hochzuladen.

    Was genau passiert dann? Oder was fürchtest Du?

    Eine “Diskussion”, die mir und dem Gegenüber nichts bringt, sondern nur diesen Kommentarbereich zuspamt. (Aber der Spam-Zug ist im Prinzip eh schon abgefahren. 😉 )

    Zum Thema:
    Der deutsche Wiki-Eintrag ist eben sehr kurz. Wie gesagt, mit dieser These habe ich mich noch nie richtig beschäftigt, ein so kurzer Artikel macht eine Einschätzung ziemlich schwierig.
    Im wesentlichen bleiben ja nur diese beiden Sätze übrig:
    Dementsprechend kann einerseits eine Theorie nicht durch einzelne empirische Beobachtungen und Experimente verifiziert oder falsifiziert werden – es stehen immer eine Reihe weiterer Theorien mit zur Debatte. Andererseits haben erkenntnistheoretische Subjekte stets mehrere Möglichkeiten, wenn eine Beobachtung im Widerspruch zu einer bestimmten Theorie steht, diese Theorie so zu verändern, dass sie wieder mit den Beobachtungen übereinstimmt.

    Dafür hätte ich dann gerne ein paar Beispiele oder Belege.
    Spontan würde ich sagen, dass das für physikalische Theorien nicht zwingend zutreffen muss.

    Was bedeutet das Ganze z.B. für das Peer-Review-System oder die fast immer unselige Debatte zwischen RT-Freunden und -Feinden?

    Wieder spontan: Gar nichts?

  129. #130 sffd
    23. April 2015

    @Benjamin: Die richtige Mathematik für die Physik besteht Grundlegend aus einem Solovay Mengentheorie Modell wobei es hier keine “unmessbaren” Mengen gibt, das macht auch jeder Physiker der Modelle auf dem Computer simuliert, er nimmt an jede Menge ist messbar, und das ist auch richtige Physik und nicht irgendwelche Abstrakten Mathematischen Objekte die eine Kardinalität höher oder gleich dem Kontinuum haben. Ich hab jetzt nicht alles gelesen was du geschrieben hast, aber ich denke das eben genau der Formalismus des logischen Positivismus dieses Problem löst, da er äquivalente Beschreibungen eliminiert. Oder geht es dir darum das man nur künstliche Modelle hat? Dann würde ich sagen das es ja klar ist das die Natur ein “chaotisches” Verhalten im Sinne der Komplexitätstheorie hat, wobei geringe Änderungen in den Anfangsbedingungen zu großen Änderungen im System führen können. Aber das heißt nicht das sich konkrete berechenbare Strukturen bilden können. Und das sich jede Theorie zu einer richtigen biegen lässt ist totaler Quatsch. Z.B. bekomme ich keine Quantenmechanik ohne Unitarität. Keine RT ohne konstante Lichtgeschwindigkeit im Vakuum, und und und.

    Man kann natürlich auch Physik machen wie sie vor urzeiten mal vorgeschlagen wurde. Man filmt einfach ein Phänomen und macht dann statistische Aussagen und die Vorgänge. Aber so funktioniert Wissenschaft nicht. Oder willst du etwa so etwas Benjamin?

  130. #131 sffd
    23. April 2015

    Und Holismus ist auch Quatsch. Jeder weiß das es in der Physik immer einen Unterschied zwischen lokalen und globalen Phänomenen gibt, obwohl die Frage ist vielleicht noch ein wenig offen und kontrovers. Aber bis jetzt spricht alles dafür.

  131. #132 sffd
    24. April 2015

    @Martin: Achso, weiß ich nicht, hab mich leider nicht ausreichen mit diesem Modellen beschäftigt. Was auch nicht wirklich einfach ist. Aber auf jedenfall gibt es diese Modelle.

  132. […] der Gravitonen winzig klein. (Mehr über Gravitonen und Quantengravitation findet ihr hier und hier.) Solche winzigen Quanten zu messen, wäre extrem […]

  133. #134 St0n
    10. September 2016

    Moin,
    ist der Kollaps einer Wellenfunktion eigentlich instantan?
    Wenn ich z.B. ein Pollenkorn, das ja falsifizierbar eine de Broglie Wellenlänge hat und der Unschärferelation unterliegt mit Photonen(auch Wellen )beleuchte um den aktuellen Ort festzustellen , geschieht die Welle Photon-Welle Pollenkorn interaktion ohne Zeitverzug?

  134. #135 MartinB
    10. September 2016

    @St0n
    Das isteine knifflige Frage, insbesondere im Zusammenhang mit der SRT siehe die Erläuterungen in diesem Artikel unter 5.
    https://scienceblogs.de/hier-wohnen-drachen/2012/10/10/quantenmechanik-und-realitat/?all=1

  135. #136 JoergS
    Berlin
    13. Dezember 2017

    Hallo erstmal,
    ich bin spät dran mit meinem Kommentar, aber ich hab’ den Blog erst jetzt entdeckt.
    Zunächst danke für die umfassende Erklärung und Herleitung. Das is alles sehr einleuchtend, aber am Ende stellt sich mir die Frage wie denn das Graviton als Austauchteilchen der Schwerkraft diese Aufgabe schafft.
    Nun ich gehe mal davon aus das die Schwerkraft eine Eigenschaft der Raumzeit ist. Genauer eigentlich das Mass der Verzerrung derselben, weil die Schwerkraft ja unmittelbar wirkt und muß, denn eine reine Übertragung durch ein Austauschteilchen (Graviton) ist durch die Lichtgeschwindigkeit begrenzt. Aus der Überlegung heraus könnte es das Graviton als solches nicht geben. Leider muss ich anmerken, das ihre Erklärung sehr schlüssig ist! Wo liegt mein Fehler? Vielleicht könne sie das aufdröseln. Danke und bis später.
    JoergS

  136. #137 MartinB
    13. Dezember 2017

    Ich verstehe das Argument nicht – eine Verzerrung der Raumzeit bewegt sich auch mit Lichtgeschwindigkeit (siehe Gravitationswellen).

  137. #138 AntiHeld
    13. Dezember 2017

    Der Fehler liegt in der Annahme “Schwerkraft ja unmittelbar wirkt”. Tut sie halt nicht

  138. #139 Alderamin
    13. Dezember 2017

    @AntiHeld

    Doch, tut sie. Der gekrümmte Raum befindet sich um die Masse herum und wirkt instantan auf den Weg einer Probemasse. Nicht unmittelbar vermittelt sind nur Änderungen der Schwerkraft (z.B. wenn man die Masse plötzlich beschleunigen würde – gleichförmige Bewegung der Masse ist hingegen relativ gesehen dasselbe wie Stillstand).

  139. #140 MartinB
    13. Dezember 2017

    @Alderamin
    Für die Schwerkraft ist der gekrümmte Raum (nahezu) irrelevant, da spielt in wesentlichen der veränderte Zeitablauf rein (außer nahe am Schwarzen Loch o.ä.).

    Was du mit “gleichförmige Bewegung der Masse ist hingegen relativ gesehen dasselbe wie Stillstand” meinst, ist mir nicht ganz klar. Meinst du, dass man z.B. die Schwarzschild-Lösung einfach lorentz-transformieren kann?

  140. #141 JoergS
    Berlin
    13. Dezember 2017

    Worauf ich hinaus will ist eigentlich, das eben für die Bewegung von Schwerkraftwellen kein Graviton notwendig ist. Die Verzerrung bewegt sich wie eine Welle vorwärts.

  141. #142 Alderamin
    13. Dezember 2017

    @MartinB

    ad 1: Sorry, ich meinte die Raumzeit insgesamt.

    ad 2: viel einfacher:
    Wenn sich etwa die Sonne gleichförmig im Raum bewegt, dann kreist ein Planet nicht etwa um den Ort, wo sich sie vor der Lichtlaufzeit von ihr zum Planeten befunden hatte (könnte man ja naiverweise annehmen, weil der Planet den neuen Ort ja vielleicht nicht “wissen” könnte; wenn ich einen Flieger am Himmel höre, dann höre ich den Schall auch nicht von dem Ort kommen, wo ich den Flieger gerade sehe, sondern von dem, wo er sich befand, als die Schallwellen sich auf den Weg zu mir machten).

    Sondern das ganze Schwerefeld bewegt sich gleichförmig mit der Sonne, der Planet umkreist den Ort, wo die Sonne sich aktuell befindet. Denn es darf kein Unterschied zwischen der Situation bestehen, ob man als mit der Sonne bewegter Beobachter diese in Ruhe sieht (und den Planet um sie kreisen) oder sich relativ zu ihr bewegt (dann muss der Planet die Sonne immer noch umkreisen).

    Ist doch korrekt, oder?

  142. #143 MartinB
    13. Dezember 2017

    @Alderamin
    @1 Wollte nur pingelig sein
    @2 Ja, meines Wissens ist das korrekt, so steht es bei Baez (und so ist es auch bei elektrischen Ladungen, siehe Feynman Lectures).

    @JoergS
    Das Graviton ist in der ART selbst nicht notwendig, das braucht man nur, wenn man die ART quantisieren will, also eine Theorie der Quantengravitation aufstellt.

  143. #144 Niels
    13. Dezember 2017

    Auch dazu gibt es einen Eintrag bei den Physics FAQ:

    Does Gravity Travel at the Speed of Light?
    In the simple newtonian model, gravity propagates instantaneously: the force exerted by a massive object points directly toward that object’s present position. […]

    In general relativity, on the other hand, gravity propagates at the speed of light; that is, the motion of a massive object creates a distortion in the curvature of spacetime that moves outward at light speed. This might seem to contradict the Solar System observations described above, but remember that general relativity is conceptually very different from newtonian gravity, so a direct comparison is not so simple. Strictly speaking, gravity is not a “force” in general relativity, and a description in terms of speed and direction can be tricky. For weak fields, though, one can describe the theory in a sort of newtonian language. In that case, one finds that the “force” in GR is not quite central—it does not point directly towards the source of the gravitational field—and that it depends on velocity as well as position. The net result is that the effect of propagation delay is almost exactly cancelled, and general relativity very nearly reproduces the newtonian result.

    This cancellation may seem less strange if one notes that a similar effect occurs in electromagnetism. If a charged particle is moving at a constant velocity, it exerts a force that points toward its present position, not its retarded position, even though electromagnetic interactions certainly move at the speed of light. Here, as in general relativity, subtleties in the nature of the interaction “conspire” to disguise the effect of propagation delay. It should be emphasized that in both electromagnetism and general relativity, this effect is not put in ad hoc but comes out of the equations. Also, the cancellation is nearly exact only for constant velocities. If a charged particle or a gravitating mass suddenly accelerates, the change in the electric or gravitational field propagates outward at the speed of light.

    Die deutsche Wiki hat dazu einen Eintrag unter
    Aberration (Gravitation).

    Ausführlich und mit Formeln das Ganze dann hier:
    Aberration and the Speed of Gravity

  144. #145 AntiHeld
    13. Dezember 2017

    @Alderamin: Sehe ich nicht so, und deckt sich imho auch nicht mit dem was z. B. der von Niels gepolsterte Eintrag auf der Wikipedia sagt.
    Evtl. reden wir aber auch aneinander vorbei.
    Natürlich wirkt die Krümmung selbst instantan auf die Probemasse, die Krümmung selbst existiert aber am Ort der Probemasse ja nur, weil sie vor einer endlichen Zeit an einem anderen Ort hervorgerufen wurde. Daran ändert sich auch nichts wenn mann ein anderes Bezugssytem wählt.

  145. #146 AntiHeld
    13. Dezember 2017

    @Alderamin
    Nachdem ich nochmal die Englische Version der Wikipedia studiert habe ziehe ich meinen letzten Post zurück…..
    Vielleicht doch nochmal ne Vorlesung zur ART besuchen 🙂

  146. #147 PaulC
    Essen
    11. Januar 2020

    Hallo erstmal und danke für den tollen Artikel (auch wenn ich ein paar Jahre zu spät kommentiere).

    Als ein absoluter Laie, der sich lediglich etwas für das Thema interessiert: Verstehe ich es richtig, dass ein (hypothetisches) Graviton deutlich schwächer ist, als ein Proton? Und das ein Graviton theoretisch mit deutlich weniger Energie erzeugt werden könnte (wenn wir wüssten wie) als ein Photon?
    Weiterhin scheint ein Graviton dem Photon relativ ähnlich zu sein. Es sind auch beides Eich-Bosonen, korrekt?
    Ich habe ebenfalls gelesen, dass es Zerfallsprozesse im Teilchenbeschleuniger gibt, bei denen Photonen emittiert werden. Könnte es demzufolge auch sein, dass bei Zerfallsprozessen von (schweren) Teilchen auch Gravitonen entstehen? Und das wir das nur nicht messen können, da die Energien zu klein sind? Könnte es daher sein, dass wenn wir größere Mengen (einige Gramm statt ein paar Atomen) von den “richtigen” schweren Teilchen hätten, die Gravitonen beim Zerfall emittieren, wir dann beim absichtlich herbeigeführten Zerfall auch ein Schwerkraftfeld sehen könnten?

    Danke und beste Grüße
    PaulC

  147. #148 MartinB
    11. Januar 2020

    @PaulC
    “Verstehe ich es richtig, dass ein (hypothetisches) Graviton deutlich schwächer ist, als ein Proton? ”
    Was meinst du mit schwächer? Die Wechselwirkung mit anderen Elementarteilchen (Kopplung) ist schwächer, das ist richtig.

    “Und das ein Graviton theoretisch mit deutlich weniger Energie erzeugt werden könnte (wenn wir wüssten wie) als ein Photon?”
    Nein, für beide gilt energie = Wirkungsquantum mal Frequenz. Es ist halt nur so, dass Photonen typischereise viel höhere Frequenzen haben als Gravitonen, weil sie durch atomare Prozese erzeugt werden, die sehr viel schneller ablaufen. Photonen mit sehr niedriger Energie haben entsprechend gigantische große Wellenlängen; es kann sie zwar geben, aber sie wären sehr schwer zu detektieren.

    “Könnte es demzufolge auch sein, dass bei Zerfallsprozessen von (schweren) Teilchen auch Gravitonen entstehen? Und das wir das nur nicht messen können, da die Energien zu klein sind? ”
    Ja, aber die Wahrscheinlichkeit für solche Prozesse ist eben extrem klein, weil die Kopplung so schwach ist.

    “Könnte es daher sein, dass wenn wir größere Mengen (einige Gramm statt ein paar Atomen) von den “richtigen” schweren Teilchen hätten, die Gravitonen beim Zerfall emittieren, wir dann beim absichtlich herbeigeführten Zerfall auch ein Schwerkraftfeld sehen könnten?”
    Ich denke nicht, weil auch dann ja die Aussendung von Gravitonen sehr unwahrscheinlich ist. Wenn man Teilchen hätte, die beim zerfall immer Gravitonen aussenden, wäre es evtl anders, aber solche Teilchen würden dann (weil die Kopplung eben so schwach ist) auch nur mit sehr niedriger Rate zerfallen.

  148. #149 Günter Stadler
    Mudau
    18. Mai 2020

    Also, wenn das so so stimmt mit dem Graviton, und den Gravitationswellen, dann ist für mich die Frage, ist der Raum selbst gequantelt und sind dann die Raumquanten selbst die Gravitonen, die Impuls und Energie transportieren?

  149. #150 MartinB
    18. Mai 2020

    @Günter
    Ja, das ist eine Möglichkeit, aber bisher hat niemand es wirklich geschafft, auf der Basis dieser Idee eine fertige Theorie (mathematisch) aufzustellen.
    Siehe auch den Artikel über Quantengravitation oder das letzte Kapitel in meinem Buch zur ART.

  150. #151 Jens
    9. Juli 2021

    Bin jetzt erst auf die Diskussion gestoßen. Und würde gerne etwas zur Aufklärung des Unverständnisses von einem bestimmten Kommentator hier beitragen (bzw. auch das gegenseitige Unverständnis).
    Wissen ist nicht = Verstehen. Ob jetzt die vorhandene Ablenkung des Kometen (für sie, oder jemand anderen) verständlicher ist nach der newtonischen Gravitation oder der einsteinischen Raumzeitkrümmung oder einer anderen nicht bestätigten Theorie (z.B. die Gravitonen), spielt für die gewonnene Erkenntnis = Wissen, keine Rolle. Der Effekt der Ablenkung und damit auch die (dauerhafte) Änderung der Flugbahn bleibt ja bestehen. Nur das Einstein es präziser berechnen könnte als Newton und seine Erkenntnis damit richtig ist.

  151. #152 MartinB
    10. Juli 2021

    @Jens
    Wissen ist nicht synonym mit Verstehen, o.k.. Aber dann würde ich Wissen gerade nicht mit “gewonnene Erkenntnis” gleichsetzen, denn etwas “erkennen” ist in meinen Augen ist mehr als etwas “wissen”. Sonst wäre auch Kants “Dass alle Erkenntnis mit der Erfahrung anfange…” sinnlos. Wissen=Erfahrung, Erkenntnis=Verstehen.

  152. #153 Jens
    20. Juli 2021

    @MartinB
    Ja stimmt, beim Verstehen habe ich den ursächlichen Zusammenhang erkannt = Erkenntnis. Eine Erkenntnis über die Herkunft der Ursache steht dann wiederum aus. Apfel fällt wegen Gravitation, Gravitationswirkung entsteht durch Raumzeitkrümmung*. * Entsteht durch Masse*. * Entsteht aus Energie. Energie entsteht aus Kräften und Wegen. Kräfte bestehen aus? Wege bestehen aus Lagedifferenzen im Raum. Der Raum besteht aus ? Ich bitte um Verständnis für die physikalisch unfachliche oder sehr grobe Sprache.
    Sind Dimensionen eigentlich kontinuierlich?

  153. #154 MartinB
    20. Juli 2021

    @Jens
    Zum Thema erklären siehe hier:
    https://scienceblogs.de/hier-wohnen-drachen/2010/08/31/kann-die-physik-die-welt-erklaren/

    “Sind Dimensionen eigentlich kontinuierlich?”
    Was meinst du genau? Ob Raum und Zeit kontinuierlich sind? Wissen wir nicht, auf sehr kleiner Längen/Zeitskala könnte die Raumzeit auch körnig oder so sein, Stichwort “Quantenschaum”.

  154. #155 Thomas
    Köln
    2. September 2021

    Ich bin eben auf der Suche nach der Gravitation auf diesen Blog gestoßen. Dankeschön für die ausführliche Erläuterung. Zugegeben, mangels Vorbildung verstehe ich einiges nicht so ganz. Trotzdem treibt mich eine Frage um, die ich hier gern loswerden möchte. Und die bezieht sich ebenfalls auf die Frage nach der Kontinuität. Falls das komplett unverständlich oder am Thema vorbei geht, dann sorry dafür.
    Ich weiß aus der klassischen Physik, also der analogen Welt, dass es nicht möglich ist, Sprünge im Energieniveau zu erzeugen (nicht in Nullzeit). Außer in Höhe eines Quants? Für die digitalisierte Welt der Quantenphysik müsste das bedeuten, dass es nicht möglich ist, eine beliebige Anzahl Quanten gleichzeitig zu übertragen. Nur, was heißt das genau: immer nur ein Quant nach dem Anderen? Was bedeutet überhaupt gleichzeitig? In welchem Bezugssystem muss das gelten? Und: braucht es dann nicht zwangsläufig eine Quantelung der Zeit selbst?
    Ich hoffe das ergibt Sinn…

  155. #156 MartinB
    2. September 2021

    @Thomas
    Ich bin nicht sicher, dass ich verstehe, was du meinst.
    In der Qm ist es so, dass Zustände mit einer eindeutigen Energie immer auch stationäre Zustände sind. Man kann Übergänge zwischen denen haben, bei denen eine Wechselwirkung z.B. ein Elktron von einem Niveau in ein anderes anhebt – das geht, weil das Elektron dann sozusagen von t= – unendlich bis zur Wechselwirkung in dem einen Zustand ist und von der Wechselwirkung bis t=+unendlich im anderen.
    Letztlich ist das aber natürlich nur ein emathematische Näherung an das, was wirklich im Universum passiert.

    Die Übergänge zwischen Niveaus snd auch nicht wirklich sprunghaft und instantan, auch das ist nur eine Näherung bzw. veranschaulichung. Insofern darf man die Vorstellung hier meiner Ansicht nach nicht zu weit treiben.

  156. #157 Thomas
    Köln
    7. September 2021

    Hallo Martin, danke für die Antwort. Der Hinweis auf den stationären Zustand klärt mein Verständnisproblem zum Teil.

  157. #158 MartinB
    8. September 2021

    Sehr gut, sonst gern noch spezifischer nachfragen.

  158. #159 MichaelW
    Dresden
    7. November 2022

    Bin in letzter Zeit mal wieder in der Physik unterwegs. Selbst bin ich an der Schnittstelle Elektronik, Software, Signalanalyse mit Interesse an praktischen Lösungen und Wissenschaft. hinzu kam Mikrobiologie und ich wüsste gern “Was ist Lenen”? (hat Schrödinger drüber geschrieben, aber die Frage nicht schlüssig beantwortet.
    Was mir scheint: Die Theorien wie z.B. die ART erlauben genaue Berechnungen und Vorhersagen, aber sie erklären die Welt nicht wirklich. Was ich suche ist ein nachvollziehbares Weltmodell. Ich kann numerisch, aber der Mechaniker in mir möchte halt wissen wie es funktioniert, das “Uhrwerk”.

  159. #160 MartinB
    8. November 2022

    @MichaelW
    Ich fürchte, damit können wir nicht dienen:
    https://scienceblogs.de/hier-wohnen-drachen/2010/08/31/kann-die-physik-die-welt-erklaren/
    Du kannst immer fragen ‘Und was ist der Mechanismus dahinter?’. Ladungen ziehen sich an – Mechanismus dahinter sind elektrische Felder – Mechanismus dahinter sind virtuelle Photonen in der Quantenfeldtheorie. Und was ist der Mechanismus dahinter? Vielleicht das Verhalten von Strings. Aber wie genau funktionieren die Strings? usw.

    Ist in meinen Augen eine Form der Antinomien der reinen Vernunft von Kant.

  160. #161 MichaelW
    Dresden
    8. November 2022

    Ich hab mal über Mustererkennung prom. Insofern ist mir Statistik und Korrelation ziemlich klar. Um die Diskussion hier vll. einzuordnen: Was die Physiker tun, ist Modellierung. Man baut sich ein Modell, also ein Abbild der Realität, im Kopf, auf Papier oder neuerdings im Computer. Man kann Entscheidungen und Vorhersagen nur aufgrund eines Modells treffen, auch wenn es sehr einfach und nicht ganz zutreffend sein mag. Denke man verwechselt Modelle mitunter mit der Realität. Irgendwann stoßen die an eine Grenze, müssen verbessert oder verworfen werden.
    Ein Modell ist eine “Kenntnisbeschreibung”. Die ART z.B. ist ein Modell der Gravitation. Mathematik ist ein Tool zur Modellbildung, muss aber nicht benutzt werden. Siehe auch: das Atommodell. In der Mustererkennung braucht man auch ein Modell dessen was man erkennen möchte. Und alles spielt sich nur in subjektiven Gehirnen ab. Die Natur interessiert sich nicht für Differentialgleichungen, sind nur menschliche Tools zwecks Modellbildung.

  161. #162 MartinB
    9. November 2022

    @MichaelW
    Modellierung – jein. Wir führen ja schon eine exttreme Vielzahl von Phänomenen auf sehr wenige grundlegende Bausteine zurück, das ist sicher mehr als das, was man normalerweise alls ‘Modellierung’ versteht.
    Was die Mathematik angeht:
    https://scienceblogs.de/hier-wohnen-drachen/2011/08/15/ist-die-natur-mathematisch/

  162. #163 MichaelW
    Dresden
    11. November 2022

    Ja ich habe das gelesen, aber finde die Diskussion müßig. Fakt ist, dass der Mensch die Mathematik nach und nach “erfunden” hat. Bisschen “Steinbruchmentalität” war auch dabei. Ein Modell ist m.E. ein von der Realität abgesetztes, meist informationsbasiertes Konstrukt, dass es erlauben soll, zu einem Input X (Frage) einen Output Y (Antwort, Entscheidung) zu liefern. Ein gutes Modell ist da möglichst gut an der Realität dran. Und Einfachheit ist natürlich auch erwünscht. Insofern sehe ich die ART auch als Modell, wenn auch sehr kompakt, sozusagen “kondensiert”. Die Natur funktioniert auch ohne Menschen und die Idee von “Mathematik”. Und ein Schaf hat sich auch Modellwissen aufgebaut, sonst käme es nicht zurecht. Übrigens ein Zahnabdruck ist auch ein Modell. Und ein Haus aus Legosteinen. Es gibt auch Modellierung durch Enumeration. Da hat man eigentlich nichts verstanden, speichert halt alles in Form von Zahlen ab. Neuronale Netze sind Modellspeicher. Die können Modelle durch Anschauung bilden, niemand versteht genau, wie das Modell dann aussieht, aber funktioniert offenbar sehr gut. Ein Chef sagte immer, wenn er keine Lösung fand anhand seiner “Modelle”: Das Leben wird schon eine Lösung finden. Und so war es ja auch in Wirklichkeit vor Mia. Jahren.

  163. #164 MartinB
    13. November 2022

    ” Die Natur funktioniert auch ohne Menschen und die Idee von “Mathematik”.”
    Es bleibt aber – zumindest für mich – ein Rätsel, wie die Natur es schafft, trotzdem mathematisch beschreibbar zu sein. Und die Tatsache, dass es Modelle gibt, belegt ja, dass wir irgendwelche Aspekte der Realität damit auch wiedergeben. (Siehe auch die QM-Diskussion ob die WF epistemisch vs. ontologisch ist). Einfach nur zu sagen “es ist halt ein Modell” ohne zu hinterfragen, wie es sein kann, dass ein solches Modell so perfekt funktioniert, ist mir schlicht zu wenig. Wir wären (Idee von D. Deutsch) nicht zufrieden, wenn wir eine Orakelmaschine hätten, die alles vorhersagt, ohne dass wir eine Idee haben, wie sie es tut – das ist auch kein Modell im eigentlichen Sinn.
    Neuronale Netze sind pragmatisch nützlich, aber verstehen tun wir damit letztlich nichts. Das Erstaunliche ist doch, dass die Natur uns eine andere Beschreibung erlaubt, und das sagt meiner Ansicht nach auch etwas über die Natur aus.

  164. #165 Tarvos
    13. November 2022

    @MichaelW

    Ich glaube jeder versteht deinen Wunsch nach dem Wissen um das “Uhrwerk”.

    Und auf der einen Seite ist Physik genau der richtige Ort um dannach zu suchen auf der anderen Seite wird Dir nichts schmezhafter Vermitteln, dass dein Ziel vermutlich abschliessend nicht zu erreichen sein wird.

    Aber hey, es gibt so viel zu entdecken, so viel zj Stsunrn und sich zu Wundern. Und ab und an macht es klick und man hat das Gefühl das sich Verbindungen ergeben und das kann wirklich erfüllend sein.

    Ich glaube z.B. auch, dass die Natur deterministisch ist (vgl. z.B. DeBroglie-Bohm). Aber ich bin mir auch ziemlich sicher, dass es die Natur nich interessiert was ich glaube und es gibt wirklich keinen Vernünftigen Grund anzunehmen, dass es so ist ( oder auch nicht).

  165. #166 MichaelW
    Dresden
    13. November 2022

    Ja ich verstehe schon dass es erstaunlich ist, dass man Felder so gut beschreiben kann. ich war seinerzeit auch fasziniert von den Maxwellschen Gleichungen. Hatte im Grundstudium Elektrotechnik studiert (Fachstudium Hörakustik, Psychoakustik und Spracherkennung, alles sehr signalbezogen). Fourieranalyse ist mir in Fleisch und Blut übergegangen, dann auch beruflich bedingt. Wavelets gibt es ja auch, so ein wenig quantenmäßig. Ferner 4 Semester Feldtheorie. War zwar sehr interessant, aber man war auch froh als es vorbei war. Hab das nie wieder gebraucht. Bin dann mehr und mehr der Software-Entwicklung verfallen und das mach ich immer noch.
    Bei all der Freude über die so gut passenden Theorien sollte man aber nicht vergessen, was das Original und was die Abbildung ist. Ich fürchte manchmal, manch einer setzt das gleich und stellt sogar das Original in Frage. Aber man darf nicht zu geozentrisch denken und auch nicht zu “homo”zentrisch. Es existiert alles genauso wenn es den Menschen nicht gäbe. Natürlich ist die ART eine geniale Beschreibung der Welt zumindest was Gravitation betrifft. Aber ich muss sagen so richtig befriedigt mich das nicht mit der Raumkrümmung. Oder den ausgetauschten “Gravitonen”. Auf jeden Fall “weiss” ein Körper von der Anwesenheit anderer Körper, hat auch einen Informationsaspekt. Bei Elektrostatik oder Magnetismus ebenso. Es fing wieder an das Interesse als ich mit den Enkeln mit zwei Magneten gespielt habe. Warum wissen die voneinander?

  166. #167 Tarvos
    14. November 2022

    “Es fing wieder a das Interesse als ich mit den Enkeln mit zwei Magneten gespielt habe. Warum wissen die voneinander?”

    Oh jetzt kann ich nicht anders. Dieser Frage hat sich der Nobelpreisgewinner R.P.Feynmann gewidmet…

    Hier zum geniessen:

    https://youtu.be/MO0r930Sn_8

    (ich hoffe der Link ist erlaubt)

  167. #168 MichaelW
    Dresden
    19. November 2022

    Mal zurück zur “alltäglichen” Gravitation. Gravitation ist auf Gegenseitigkeit, also jede Masse wird von einer anderen angezogen und umgekehrt. Da ein Körper aus Atomen besteht, deren Masse im Kern liegt, bilden die Atomkerne die Quelle der Gravitation. Die Gravitationskräfte addieren sich? denn die Wirkung z.B. der Erde reicht recht weit, mindestens zum Mond. Da gibt es doch diesen Punkt (Fläche), wo sich die Anziehungskräfte aufheben.
    Ein Atom z.B. in der Erde ist umgeben von vielen anderen Atomen. Jedes Atom übt eine Gravitationskraft auf alle anderen aus. Kontinuierlich oder oszilliert das aufgrund der Wellenfunktions-Unbestimmtheit?
    Gravitation kann man offenbar nicht abschirmen, elektromagnetische Felder schon. Ob Gravitation z.B. eine sehr dicke Uranplatte durchdringen? Vermutlich addiert sich deren Gravitation nur. Eine Antigravitation zu erfinden wäre super. Wie entsteht eigentlich Zentrifugalkraft?

  168. #169 MartinB
    19. November 2022

    @MichaelW
    Das sind ja sehr viele Fragen auf einmal…
    Reichweite der Gravitation ist unendlich, sie nimmt aber mit der Entfernung ab.
    Wie die Gravitation auf Quantenebene funktioniert, weiß niemand.
    Gravitation kann man nicht abschirmen, weil es anders als bei elektrischen Feldern nur eine Sorte “Ladung” gibt.
    Zentrifugalkraft ist eine Scheinkraft, die man in einem rotierenden Bezugssystem wahrnimmt.

    Alle diese Fragen (und noch viele mehr) beantwortet mein Buch “Isaac” übrigens ausführlich…

  169. #170 MichaelW
    Dresden
    20. November 2022

    Interessant, bitte um genauere bibliographische Angaben.
    Auf jeden Fall kann die Zentrifugalkraft gewaltig sein, die Himmelsmechanik basiert darauf. Aber im kleinen: z.B. ein Fliehkraftregler; setzt die Zentrifugalkraft in eine lineare Kraft um, die dann etwas bewirkt. Scheinkraft? Wahrnimmt?
    Noch eine Frage: “Potentielle Energie”. Ich hebe etwas “hoch” d.h. weg vom Erdmittelpunkt. Ich kann die aufgewendete Energie rückgewinnen, wenn ich den Körper wieder herunterlasse. Angewandt z.B. auch im Pumpspeicherwerk. Wo wird die Energie zwischenzeitlich gespeichert? Kann sich Gravitation “verbrauchen”?
    Mit einem Magneten kann man das auch machen. Ich ziehe ein Eisenteil weg vom Magneten unter Energieeinsatz und diese wird wieder frei wenn ich das Teil wieder zum Magneten hin lasse. Geht auch mit einer Spiralfeder. Wo ist da die Energie – wohl letzten Endes elektromagnetisch erklärbar.

  170. #171 MartinB
    20. November 2022

    @MichaelW
    Rechts oben auf der Blogseite findest du den Link zum Buch:
    https://link.springer.com/book/10.1007/978-3-662-57293-1
    (Auch bestellbar bei jeder Buchhandlung oder Online…)

    Fliehkraft ist eine Scheinkraft, die nimmt man wahr, weil man sich selbst als ruhend annimmt, währned man in Wahrheit beschleunigt wird.

    Gravitationsenergie in der klassischen Physik ist im Gravitationspotential gespeichert. In der ART ist die Sache knifflig, sie ist dort als “Raumkrümmung” gespeichert, aber nicht direkt lokalisierbar.

    Beim Magneten steckt die Energie im Magnetfeld, Felder können Energie speichern. Im Quantenbild sind das virtuelle Photonen, die die Energie speichern.

  171. #172 MichaelW
    Dresden
    21. November 2022

    Beim Suchen nach Gravitation ist mir dieser Titel untergekommen:
    Quantum Energetics Volume 1: Theory of Superunification von Vladimir Leonov
    Man kann es herunterladen (> 760 Seiten). Er meint die “Große Vereinheitlichung” gelöst zu haben durch Einführung einer 5. Kraft ” superstrong electromagnetic interaction” von der bekannten Grundkräfte Spezialfälle seien. Alles beruht letztlich auf der dunklen Materie. Als Elementarteilchen sieht er das “Quanton”. Ist aber alles mathematisch anspruchvoll ausgeführt. Ist der Name bekannt?

  172. #173 MartinB
    22. November 2022

    @MichaelW
    Nein. Soweit ich sehe, gibt es dazu aber keine Veröffentlichungen in Fachzeitschriften (wo Forschungsergebnisse publiziert werden), jedenfalls habe ich bei google scholar keine gefunden.
    Die Wahrscheinlichkeit, dass es sich lohnt, das anzugucken, geht vermutlich gegen Null.

    Tipp, gerade wenn du selbst kein Physiker bist: Halte dich an die etablierte Wissenschaft – du wirst nicht ohne weiteres in der Lage sein, seltsame Privattheorien von echter Physik zu unterscheiden.

  173. #174 MichaelW
    Dresden
    22. November 2022

    Also so ganz unbedarft bin ich nicht, was die Einschätzung von papers angeht. Ich habe ca. 10 Jahre an der Universität wissenschaftlich gearbeitet; später dann privat auf dem Gebiet Zellbiologie, habe ca. 3000 papers gesammelt und durchgesehen bzw. eingeschätzt ob das seriös ist. Mit der “etablierten”. Wissenschaft habe ich das grundsätzliche Problem, dass nur dann etwas neues entstehen wird, wenn man zumindest gedanklich davon löst. Was ist “echte” Physik? Diese Einteilung in die verschieden “Claims” kritisiert man insbesondeer auch in der Medizin als kontraproduktiv. Wenn wir weiterkommen wollen, müssen wir frei denken können. Der Leonov scheint durchaus kein Privatforscher mit skurilen Ideen zu sein. Sein “Weltmodell” hat mich doch angesprochen, es ist das, was ich seit sehr langer Zeit suche und es scheint denkbar. Er wirft ja nicht alles über Bord, z.B. die Raumkrümmung oder die Quantenphysik. Auf jeden Fall ein hochinteressantes Modell.

  174. #175 MartinB
    22. November 2022

    @MichaelW
    Wenn du nicht Physik studiert hast, hast du keine Chance, seriöse von unseriösen papers an Hand ihres Inhalts zu unterscheiden, egal wie viel allgemeine Erfahrung du haben magst. Selbst ich (mit Promotion in E-Teilchenphysik) traue mir nicht zu, immer zu erkennen, ob etwas korrekt ist oder nicht, weil ich nicht mehr ganz aktuell in der Forschung bin.

    “Wenn wir weiterkommen wollen, müssen wir frei denken können. ”
    Können wir alle. Aber “frei denken” hilft nur sehr bedingt, um neue Theorien aufzustellen.

  175. #176 MichaelW
    Dresden
    23. November 2022

    Wissenschaftliche Entwicklung gleicht der Evolution. Wir haben mal evolutionäre Verfahren zur Optimierung von Systemen implementiert. Indem man einige “Individuen” radikal verändert, besteht grundsätzlich die Möglichkeit, neue Gipfel der “Nutzensfunktion” zu erreichen im Vergleich zu Gradientenverfahren der kleinen Schritte. Man gelangt auch so auf einen Gipfel, der aber auch ein lokaler Gipfel sein kann. Und solche “revolutionären” Schritte hat es ja auch in der Physik gegeben. Man hört ja auch seit längerem von einer Krise der Physik, was im evolutionären Modell bedeutet dass man auf einem lokalen Gipfel sitzt und sich über die schöne Aussicht freut – beunruhigend nur, dass man in der Ferne einen höheren Gipfel erkennen kann. Habe mir das Buch Isaac gekauft, bin gespannt. Habe allerdings einige Physikbücher hier, ungelesen. Auch den Leonov. Ich meine, wenn der mehr als 700 Seiten geschrieben hat, viel Mathematik dabei – das macht man nicht so mit links. Ein Evolutionsschritt ist auch “Rekombination”, also bisherige Information geht auch ein. Und da fällt mir das 3. Grundgesezt des dalektischen Materialismus ein: Negation der Negation.

  176. #177 Tarvos
    24. November 2022

    @MichaelW:

    Aber was genau erwartest Du denn jetzt?
    Du möchtest selber keine Physik betreiben.
    Du hast ein Buch gefunden, was Du nicht verstehst aber Dir gefällt der „radikale Ansatz“ ( den Du nicht versteht).
    Die etablierte Physik findest Du blöd, weil es offene Fragen gibt. Und weil numerische Optimierungsverfahren immer die Gefahr beinhalten in lokalen Extrema festzuhängen sollten wir frei denken….
    Du hast (wie Du sagst) ein Problem mit etablierter Physik

    Irgendwie alles wenig handfest. Und (sry) weniger als ich von einem Promovierten Ingenieur erwarte (Troll?)

  177. #178 MichaelW
    Dresden
    25. November 2022

    an Tarvos: Danke für die Komplimente! Keine Sorge ich habe keine Probleme! Ich muss nicht Physiker werden. Ja und mal raus aus den in Stein gemeisselten Denkschablonen – ich kann es mir leisten. Was mich seit längerem beschäftigt ist die Frage: Was ist leben? Papers zu diesem Thema füllen Aktenordner. Ein Herr Del Giudice war da nah dran. Und dieses Leonov-Werk verteh ich schon, hat aber einige Sprünge. Ansonsten scheint es mir so, dass insbesondere die Teilchenphysik ein paar ungeklärte Probleme hat. Und wegen einiger Fragen, die ich gern aufhellen würde, interessiert mich die Struktur des Vakuums. “Dass ich erkenne was die Welt im Innersten zusammenhält…”

  178. #179 Tarvos
    25. November 2022

    Aber was möchtest Du denn dann?
    I just dont get it…

    Du hast ein Faibles für Theorien abseits dessen was etabliert ist aber wenig Lust Physik selber zu betreiben.

    Und Du schmeißt mit vielen „spannenden“ Fragen um Dich (Sinn des Lebens, wie funktionieren Magnete, Struktur des Vakuums (?) )

    Es ist total cool diese Neugier zu haben, aber die Abkürzung die Du suchst gibt es nicht.

    Sry

  179. #180 MichaelW
    Dresden
    26. November 2022

    – Nein ich habe keinen “Fable” für abseitige Sachen, aber “etabliert” ist für mich nicht in Stein gemeisselt. Es gibt keine Autoritätenbeweise. Und an der Physik störte mich schon immer, dass die Struktur des Vakuums nicht thematisiert wird. Das würde der Physik irgendwie den Rückhalt geben. Hatte als Etechniker 4 Semester Feldtheorie, 4 Semester Theoretische Elektrotechnik, 2 Sem Physik. Ich habe nichts gegen Physik, aber ich bin auch ein begeisterter Ingenieur und Algorithmenentwickler, und der Ing. möchte gern “deliverables”.
    – Habe ein großes Labor/Büro… ziemlich voll mit Elektronik/Elektrokram. 20 Jahre GmbH gewesen.
    – Ich interessiere mich nicht für den “Sinn” des Lebens, sondern für die sog. “living matter”. Also warum “lebt” die Zelle, dieses kleine Volumen, Betreibe das Fach jetzt 6-7 Jahre. Medizin muss man dazu nicht studieren. Es ist Physik. M.E. ist es etwas Quantenmechanisches. Die Kohärenz von Wassermolekülen? Es gibt jedenfalls eine ernsthafte Forschungsrichtung “Quantenbiologie” und Schrödinger hat ja dieses Buch geschrieben “Was ist Leben”. Aber beantwortet hat er die Frage auch nicht.
    Na gut der Auslöser war “Anti-Aging/Rejuvenation”, am Ende war ich bei Quantenphysik und Entropie/Negentropie (Wortschöpfung von Schrödinger).

  180. #181 stone1
    27. November 2022

    Bezüglich

    Quanton

    falls tatsächlich ein neues Elementarteilchen entdeckt worden wäre, und zwar schon anno 1996 (aus dem Jahr stammt das Buch/Paper), hätte das bestimmt Wellen geschlagen und der Entdecker hätte es mindestens zu einem Wikipediaeintrag geschafft, sollte man annehmen. Hat er aber nicht. Dafür kann man in die Entwicklung des Quantenmotor Vladimir Leonov investieren.

    So schaut’s aus. Ich würde nicht mal ein Butterbrot oder einen Gebrauchtwagen von diesem Herrn kaufen, geschweige denn ein Physikbuch, selbst wenn es quasi gratis als .pdf downloadbar ist.

    Wohingegen Martin Bäkers auf etablierter und nicht herbeiphantasierter Physik beruhende Buch seinen Preis auf alle Fälle wert ist.

  181. #182 MartinB
    27. November 2022

    @MichaelW
    ” Und an der Physik störte mich schon immer, dass die Struktur des Vakuums nicht thematisiert wird. ”
    Huh? Die ganze Quantenfeldtheorie ist letztlich nichts als eine Theorie des Vakuums, siehe insbesondere die letzten Teile meiner QFT-Serie. Natürlich wird das thematisiert und erforscht (Casimir-Effekt, Vakuum-Fluktuationen, etc.), es ist eben nur schwierig, das absolute Vakuum zu erforschen, weil es sofort keins mehr ist, sobald wir mit ihm Wechselwirken (siehe auch meine diversen anderen Artikel zum Vakuum, davon gibt es hier ja einige).

    “Etwas Quantenmechanisches” ist keine Wissenschaft. Erstens ist die ganze Chemie quantenmechanisch, zweitens ist das viel zu unspezifisch um irgendwie einen Nutzen zu ergeben.

    Es ist im Übrigen interessant, dass ein Großteil der Leute, die hier aufschlagen und erzählen, die Physik sei irgendwie in Schablonen gefangen (oder die selbst eine Alternative Physik entwickeln) E-Technik-Ingenieure (Frauen ausdrücklich nicht mit gemeint) sind. Ich vermute, dass da etwas dahintersteckt, vielleicht der Glaube, nur weil man in der E-technik oft mit “Oh, ich bastle mir mal eine unkonventionelle Lösung” weiterkommt, wäre das in anderen Disziplinen genauso.

  182. #183 MichaelW
    Dresden
    27. November 2022

    Ok da sehe ich ein dass es hier der falsche Platz ist um eine Sache oder Hypothese zu diskutieren, die vom vorgegeben Weg abweicht. Die genannten Vakuum-Effekte sind mir natürlich bekannt, wobei es für den Casimir-Effekt auch eine andere Erklärung gibt. Ich kann so ein Buch wie das von Leonov schon lesen, bin kritisch genug. Das “Quanton” ist kein neues Elementarteilchen dar, sondern den Baustein der Quantisierten Raumezeit dar. Beim Lesen musste ich doch oft schmunzeln, weil er öfter man meinen Anschauungen, was Physik betrifft, sehr nahe kam. Natürlich ist viel Selbstbeweiräucherung dabei…aber man kennt die Umstände nicht.
    Ok, also dann weiter viel Freude an der Physik. Zum Glück reicht nach Leonov die Energie des Colliders nicht aus um einen Big Bang auszulösen.
    Und nichts gegen die Ingenieure: dem Inschenör ist nichts zu schwör.

  183. #184 Karl-Heinz
    Graz
    28. November 2022

    @MichaelW

    Oh, man
    Du glaubst wirklich dass die folgende Behauptung von Vladimir Leonov richtig ist?

    Es sollte mit deiner Ausbildung ein leichtes sein, die untenstehende Behauptung ad absurdum zu führen.

    Unwiderlegbare theoretische und experimentelle Beweise für den Zusammenbruch des Postulats “Die Lichtgeschwindigkeit ist
    Lichtgeschwindigkeit ist eine Konstante‖ wurden erbracht.
    2. Zum ersten Mal erhielten wir experimentell bestätigte Beweise und neue Formeln, die zeigen
    dass ein Gravitationsfeld die Lichtgeschwindigkeit verlangsamt. Die Änderung der Lichtgeschwindigkeit tritt
    nur im radialen Gravitationsfeld in der vertikalen Ebene.
    3. Aufgrund des Einflusses des Gravitationsfeldes der Erde beträgt die Abbremsung der Lichtgeschwindigkeit auf der
    der Erdoberfläche 0,1 m/s. Die Abbremsgeschwindigkeit des Lichts im Gravitationsfeld der Sonne in der Nähe der
    Sonne beträgt 318 m/s. Die Lichtgeschwindigkeit ist auf der Oberfläche eines Schwarzen Lochs zum Stillstand gekommen. Es gibt einen Bereich von
    der variablen Lichtgeschwindigkeit in der Natur vom Maximalwert bis zu Null. Die Physik basiert
    die auf dem Postulat “Die Lichtgeschwindigkeit ist eine Konstante” beruht, muss überarbeitet werden.
    4. Diese Entdeckungen wurden möglich durch die neue Quantentheorie der Gravitation, die Theorie der
    Supervereinigung und dem grundlegend neuen dynamischen Interferometer, das für den Nachweis von
    Änderungen der Lichtgeschwindigkeit in einem Gravitationsfeld. Die Experimente können in jedem
    universitären Physiklabor wiederholt werden.
    5. Diese Entdeckungen verändern unser Verständnis der Natur der Schwerkraft und des Elektromagnetismus radikal.
    Elektromagnetismus.
    Dieser Beitrag mit Ergänzungen wurde von Vladimir Leonov am 29. Dezember 2021 verfasst.

  184. #185 MartinB
    28. November 2022

    @Katl-Heinz
    OMG, so einen Unsinn schreibt der?
    Wobei man ja fairerweise sagenmuss, dass Sätze wie “Wenn Gravitations im Spiel ist, ändert sich die Lichtgeschwindigkeit” auch in seriösen Büchern stehen – wenn man halt nicht sauber zwischen lokalen und globalen Koordinatensystem unterscheidet, dann kommt eben Blödsinn heraus.

  185. #186 Karl-Heinz
    Graz
    28. November 2022

    @MartinB
    Ja, die berühmte Shapiro-Verzögerung.

    Bei MichaelW habe ich das Gefühl, er will die Relativitätstheorie überspringen und sich gleich dem für ihn zukunftsträchtigen Vladimir-Leonov-Modell zuwenden. Ein schwere Fehler. Er merkt nicht, was für einen Blödsinn Vladimir-Leonov verzapft.

  186. #187 stone1
    28. November 2022

    Vor allem könnte man sich fragen, warum die Physiklabors nicht längst voller begeisterter Studenten und Professoren sind, die diese angebliche Änderung der Lichtgeschwindigkeit nachweisen, da ja laut obigem Zitat

    Die Experimente können in jedem
    universitären Physiklabor wiederholt werden.

    Schon sehr seltsam, bestimmt hält da die ganze ver- bzw. eingeschworene Physikergemeinde den Deckel drauf. 😉

  187. #188 MichaelW
    Dresden
    28. November 2022

    Warum sollte ich mich mit den genannten Aussagen beschäftigen? Ich sag ja garnicht dass der Leonov immer recht hat. Sehe den neben vielen anderen als Ideengeber. Was mich aber interessiert, warum ist die Vakuum-Geschwindigkeit so hoch wie sie ist? Sie wird ja materialabhängig kleiner? Woraus besteht das Vakuum, dass sie halt nur bzw. gerade den bekannten Wert hat? Es gab jüngst Meldungen, dass man Licht langsamer laufen lies. Angeblich hat man dem Licht eine Korkenzieher-Struktur aufgeprägt, so dass der Weg länger war. Woraus besteht das “Medium” Vakuum?

  188. #189 Karl-Heinz
    Graz
    28. November 2022

    @MichaelW

    ε0*µ0*c²=1

    Das Korkenzieher-Licht ist sehr interessant. Das stammt aber nicht von
    Vladimir-Leonov.

  189. #190 MichaelW
    Dresden
    29. November 2022

    Nein mein Blickwinkel ist etwas weiter, der Leonov war mir jüngst aufgefallen. Suche nur ab und zu nach bestimmten Schlüsselworten, schließlich krieg ich das nicht bezahlt ebenso wie die Mikrobiologie, die sich ja mit der Physik trifft, wenn es ganz klein wird. Die Korkenzieher-Lichtstrahlen wurden wohl auf den Phillipinen hergestellt, es wurde dann später reproduziert. Wie man das macht? Stand in Scinexx denk ich.

  190. #191 Christian
    Wien
    8. Februar 2023

    Bei allen Wellen sind Teilchen die Energie- und Impulsüberträger. Aus quantenphysikalischer Sicht wären es bei Gravitationswellen eben die Gravitonen. Welche Teilchen wären das aber aus Sicht der relativistischen Physik? Bei einer quantisierten Relativitätstheorie wären es eben Raumzeitquanten. Raumzeitquanten wären aber nicht unbedingt dasselbe wie Gravitonen. Denn nachdem Kaluza die Maxwell-Gleichungen aus den um eine Raumrichtung erweiterten Einsteinschen Feldgleichungen ableiten konnte, könnten uns schwingende Raumzeitquanten einmal als Photonen und einmal als Gravitonen erscheinen.
    Jetzt basiert aber auch Kaluzas Theorie auf der Klassischen Physik, womit auch der Elektromagnetismus ein nichtquantisiertes geometrisches Phänomen wäre, das keiner Photonen bedarf. Wie würde also gemäß Kaluza die Energie- und Impulsübertragung durch Licht funktionieren? Wenn es darauf eine Antwort gibt, dann gäbe es wohl auch einen Analogieschluss hin zur Gravitation.

  191. #192 Karl-Heinz
    Graz
    9. Februar 2023

    @Christian

    Ich habe die KI befragt und die meint folgendes. 🙂

    Die obige Aussage ist unvollständig und nicht ganz richtig. Kaluza hat tatsächlich die Idee einer Erweiterung der Einsteinschen Feldgleichungen auf fünf Dimensionen vorgestellt, wobei eine der Dimensionen als “versteckte” Dimension interpretiert wurde. Seine Arbeit hat zur Entwicklung der Kaluza-Klein-Theorie beigetragen, die eine Vereinigung von Gravitation und Elektromagnetismus vorschlägt.

    Allerdings ist es nicht der Fall, dass schwingende Raumzeitquanten in der Kaluza-Klein-Theorie als Photonen und Gravitonen erscheinen. In der Kaluza-Klein-Theorie werden Photonen und Gravitonen auf unterschiedliche Weise aus den Schwingungen der Raumzeit interpretiert, jedoch sind sie keine quantenmechanischen Teilchen. Die moderne Physik beschreibt Photonen als quantenmechanische Teilchen, die elektromagnetische Wellen tragen, und Gravitonen als hypothetische Teilchen, die für die Übertragung der Gravitation verantwortlich sind.

  192. #193 Christian
    Wien
    9. Februar 2023

    Servus Karl-Heinz! Das ist tatsächlich ein KI-Beitrag? Gar nicht schlecht! Aber: Ja freilich werden bei Kaluza Photonen und Gravitonen auf unterschiedliche Weise aus den Schwingungen der Raumzeit interpretiert, dass sie aber keine quantenmechanischen Teilchen sind, ist klar, weil die Relativitätstheorie und die daraus abgeleitete Kaluza-Theorie ja sonst keine Theorien der Klassischen Physik mehr wären. Ich habe ja daher auch von einer quantisierten Relativitätstheorie geschrieben.

    Was ich aber gerne wissen würde, ist, wie sich Einstein (und Kaluza) vorgestellt haben, dass durch Schwingungen der Raumzeit Impuls und Energie übertragen werden sollen? Denn wie ich bei meinem gestrigen Beitrag eingangs erwähnt habe, sind bei allen Wellen Teilchen die Energie- und Impulsüberträger. Aus dem bloßen Wellencharakter von Licht lässt sich doch nicht eine Energie- und Impulsübertragung ableiten, oder?

    Hat sich Einstein überhaupt Gedanken gemacht, was denn die Raumzeit sein und wie eine Verwellung der Raumzeit funktionieren soll?

  193. #194 Karl-Heinz
    Graz
    9. Februar 2023

    Servus Christian

    Mein neuer Slogan ist: Wenn mich jemand versteht, dann kann es nur eine KI sein. 😉

    Frage: Warum steckt in der Krümmung der Raumzeit eine Energie?

    Antwort von der KI: In der allgemeinen Relativitätstheorie wird die Raumzeit als ein vierdimensionales Gebilde betrachtet, dessen Krümmung durch die Anwesenheit von Materie und Energie bestimmt wird. Dies bedeutet, dass die Bewegung von Materie und Energie die Krümmung der Raumzeit verändern kann.

    Die Idee, dass die Krümmung der Raumzeit mit Energie verbunden ist, kann man aus der Formulierung der Energie-Impuls-Tensor herleiten. In der Physik beschreibt der Energie-Impuls-Tensor die Energie und den Impuls, die in einem bestimmten Bereich der Raumzeit vorhanden sind. Die Krümmung der Raumzeit kann als eine Art Energie-Dichte interpretiert werden, die in einem bestimmten Bereich der Raumzeit vorhanden ist.

    Diese Idee, dass die Krümmung der Raumzeit mit Energie verbunden ist, ist eines der grundlegenden Konzepte der allgemeinen Relativitätstheorie und hat weitreichende Auswirkungen auf unsere Vorstellung von der Natur der Raumzeit und der Gravitation.

    Frage: Wie kann eine elektromagnetisches Feld in diesem Zusammenhang betrachtet werden?

    Antwort von der KI: In der klassischen Elektrodynamik beschreibt das elektromagnetische Feld die Wechselwirkung zwischen elektrischen und magnetischen Momenten. In der allgemeinen Relativitätstheorie wird das elektromagnetische Feld jedoch als Teil des vierdimensionalen Raumzeitgebildes betrachtet.

    In diesem Kontext kann das elektromagnetische Feld als eine Art Krümmung der Raumzeit interpretiert werden, die durch die Anwesenheit von elektrischen und magnetischen Momenten verursacht wird. Diese Krümmung kann ihrerseits die Bewegung von Ladungen beeinflussen und kann auch von Bewegungen von Ladungen verändert werden.

    Es ist wichtig zu beachten, dass das elektromagnetische Feld und die Gravitation zwei unterschiedliche Arten von Krümmungen der Raumzeit sind, die jeweils durch unterschiedliche Arten von Energie und Impuls verursacht werden. Trotzdem können sie miteinander wechselwirken, wie es beispielsweise bei der Bahnberechnung von Planeten im Sonnensystem der Fall ist.

    Frage: Und warum benötigt Herr Kaluza 5 Dimensionen?

    Antwort von der KI: Theodor Kaluza benötigte 5 Dimensionen, um die elektromagnetische Kraft und die Gravitation in einer einzigen Theorie zu vereinen. Im Jahr 1921 veröffentlichte er eine Arbeit, in der er zeigte, dass man die Maxwell-Gleichungen, die die elektromagnetische Kraft beschreiben, aus den um eine Dimension erweiterten Einsteinschen Feldgleichungen ableiten kann.

    In diesem Ansatz wurde eine zusätzliche Dimension eingeführt, die so konstruiert war, dass sie in den üblichen vier Dimensionen (drei räumliche und eine Zeitdimension) nicht beobachtbar war. Diese zusätzliche Dimension wurde als geschlossene Kurve (eine Art von Kreis) betrachtet, die so klein war, dass sie nicht von uns beobachtet werden konnte.

    Kaluza argumentierte, dass die Maxwell-Gleichungen in diesem fünfdimensionalen Raumzeitkontext als eine Art Schwingung in der zusätzlichen Dimension interpretiert werden könnten. Diese Schwingungen konnten dann als elektromagnetische Wellen in unseren üblichen vier Dimensionen interpretiert werden.

    Dieser Ansatz war ein bedeutender Durchbruch in der Verbindung von Elektrodynamik und Gravitation und hatte einen großen Einfluss auf die weitere Entwicklung der Theorien über die Einheit der Kräfte. Heute ist er jedoch weitgehend durch die Stringtheorie und andere moderne Konzepte abgelöst worden, die eine höhere Dimensionenanzahl als notwendig erachten.

    PS: Aber ich denke, dass MartinB sich da sicher besser auskennt.

  194. #195 Karl-Heinz
    Graz
    10. Februar 2023

    @Christian

    Wie würde also gemäß Kaluza die Energie- und Impulsübertragung durch Licht funktionieren? Wenn es darauf eine Antwort gibt, dann gäbe es wohl auch einen Analogieschluss hin zur Gravitation.

    Ja … dann … mal gucken…oh, was ist den das?

    Ich wollte anschließend von der KI wissen, ob die drei Raumdimensionen und die eine Zeitdimension in der Stringtheorie kompakt sind oder nicht. Die KI meinte dann, dass in einigen Versionen der Stringtheorie, die beobachtbaren drei Raumdimensionen und die Zeitdimension als emergente Phänomene auftreten.

    Natürlich wollte ich dann wissen, was den genau ein emergentes Phänomen sein soll.

    Antwort der KI:

    Ein emergentes Phänomen ist ein Phänomen, das aus den Interaktionen vieler einfacher Teilchen oder Systeme auf einer kleineren Skala entsteht und auf einer größeren Skala eine eigene, zusammenhängende Struktur und Eigenschaften hat. Es ist nicht direkt aus den Eigenschaften der einzelnen Teilchen oder Systeme ableitbar, sondern entsteht aus ihrer Wechselwirkung.

    Ein Beispiel für ein emergentes Phänomen ist das Verhalten einer Flüssigkeit. Eine Flüssigkeit besteht aus vielen Teilchen, aber die Eigenschaften einer Flüssigkeit, wie ihre Fähigkeit zur Fließbewegung und ihre Oberflächenspannung, sind nicht direkt aus den Eigenschaften der Teilchen abzuleiten. Stattdessen entstehen diese Eigenschaften aus den Wechselwirkungen zwischen den Teilchen.

    PS: Ich denke, ich habe wieder was dazu gelernt. 🙂

    PS2: In vielen Versionen der Stringtheorie treten drei Raumdimensionen und eine Zeitdimension auf, aber es ist unklar, ob sie kompakt oder nicht-kompakt sind.

  195. #196 MartinB
    10. Februar 2023

    @Karl-Heinz
    Ist das echt ne Antwort von ChatGPT? (Falls ja, wäre mir die erste Antwort irgendwann vielleicht mal einen Blogpost wert…)

    Jedenfalls ist die erste Antwort falsch. Ja, in der ART steckt Energie in der Krümmung der Raumzeit, aber diese ist nicht ohne weiteres lokalisierbar. (Das ist kein echtes problem innerhalb der Theorie, wohl aber für unsere Anschauuung bzw. unsere Vorstellung, was Energie ist.) Das muss so sein, weil man lokal an jedem Raumzeitpunkt immer ein lokal flaches Bezugssystem finden kann (Äquivalenzprinzip), d.h. man kann für eine Umgebung jedes Raumzeitpunkts immer ein Bezugssystem finden, in dem die Raumzeit eben lokal ungekrümmt ist.

    Wenn sich in Zukunft Leute auuf solche KIs verlassen, z.B. Studis beim Lernen, wird es gruselig, wenn da solche Halbwahrheiten verbreitet werden.

    Zu Kaluza-Klein kann ich nicht viel sagen, da kenne ich mich nicht aus – aber soweit ich mich erinnere, ist es in Penroses Road To Reality ganz gut erklärt, ist aber schon 12 Jahre her, dass ich das gelesen habe.

  196. #197 Karl-Heinz
    Graz
    10. Februar 2023

    @MartinB

    Zumindest ist die ChatGPT schlagfertig und irgendwie, da sie auch Fehler macht, menschlich. 😉

    Meine Frage an ChatGPT: Ich sprach von einer lokalen Umgebung. Ist da nicht alles flach? Wie kommst du dann auf eine Krümmungsenergie?

    Ihre Anwort: Es tut mir leid, ich habe meine Antwort missverständlich formuliert. In einer lokal begrenzten Umgebung, wie z.B. im Umfeld eines Tischs auf der Erde, ist die Krümmung des Raum-Zeit-Gefüges tatsächlich sehr gering. In diesem Fall wäre die Krümmungsenergie so gering, dass sie vernachlässigbar ist und die Umgebung als flach betrachtet werden kann.

    Eine lokale Krümmung des Raum-Zeit-Gefüges ist in der Tat nur in Bereichen von großen Massen und Energien signifikant, wie z.B. in der Nähe von schwarzen Löchern oder bei der Kollision von Neutronensternen, wie es in den Gravitationswellenbeobachtungen nachgewiesen wurde. In solchen Fällen kann die Krümmungsenergie signifikant sein und direkt beobachtet werden, indem man die Auswirkungen auf die Bewegung von Teilchen in der Umgebung betrachtet.

    Meine Frage an ChatGPT:Kann eine lokale Umgebung überhaupt gekrümmt sein? Ist da nicht alles Flach, wenn die Umgebung wirklich sehr klein ist?

    Ihre Anwort:
    Ja, Sie haben absolut recht. In einer sehr kleinen lokalen Umgebung kann man die Raumzeit als flach betrachten, und es ist korrekt, dass die Krümmung des Raum-Zeit-Gefüges in einer solchen Umgebung vernachlässigbar ist. Die Krümmung des Raum-Zeit-Gefüges wird erst bei größeren Skalen signifikant und spielt eine wichtige Rolle in der Allgemeinen Relativitätstheorie.

  197. #198 Christian
    Wien
    10. Februar 2023

    Meinen Gruß auch an den Martin, der zum Dialog zwischen Karl-Heinz und mir dazugekommen ist!

    Was mich interessiert, ist, wie sich Einstein (und Kaluza) vorgestellt haben, dass durch eine Wellenbewegung der Raumzeit Impuls und Energie übertragen werden sollen? Denn wie ich bei meinem obigen Beitrag #191 eingangs erwähnt habe, sind bei allen Wellen Teilchen die Energie- und Impulsüberträger. Aus dem bloßen Wellencharakter von Licht lässt sich doch keine Energie- und Impulsübertragung ableiten, oder?

    Hat sich Einstein überhaupt Gedanken gemacht, was denn die Raumzeit sein und wie eine Verwellung der Raumzeit funktionieren soll? Wäre die Relativitätstheorie eine quantisierte Theorie, dann wäre die Raumzeit eben aus Raumzeitquanten aufgebaut, und eine Verwellung der Raumzeit (als Gravitationswellen nach Einstein bzw. als Gravitationswellen und elektromagnetischen Wellen nach Kaluza) wäre mitsamt Impuls- und Energieübertragung im Prinzip eine genauso einfache Sache wie etwa die Beschreibung von Wasserwellen.

  198. #199 Karl-Heinz
    Graz
    10. Februar 2023

    @Christian

    Aus dem bloßen Wellencharakter von Licht lässt sich doch keine Energie- und Impulsübertragung ableiten, oder?

    Doch … 😉

    Impulsdichte des elektromagnetischen Feldes g
    g = S / c2
    S … Poynting-Vektor S = E × H
    E, H: elektrische bzw. magnetische Feldstärke
    c: Lichtgeschwindigkeit

    Die Energiedichte des elektromagnetischen Feldes im Vakuum ist
    w_em=1/2 ε_0(E^2+c^2B^2)
    bzw.
    w_em=1/2(ED+BH)

  199. #200 Christian
    Wien
    10. Februar 2023

    Einerseits: Guter Hinweis, Karl-Heinz.
    Andererseits: Lässt sich der Compton-Effekt alternativ auch ausschließlich über die Welleneigenschaft von elektromagnetischer Strahlung deuten. Ich meine, dass dem nicht so ist.

  200. #201 Karl-Heinz
    Graz
    10. Februar 2023

    @Christian

    Andererseits: Lässt sich der Compton-Effekt alternativ auch ausschließlich über die Welleneigenschaft von elektromagnetischer Strahlung deuten. Ich meine, dass dem nicht so ist.

    Das würde ich verneinen. Ich kann den Compton-Effekt nur verstehen, wenn man die Photonen als Teilchen auffasst, das Energie und Impuls auf das Elektron überträgt.

    Vielleicht sagst du mir, worauf du genau hinaus willst.
    Ich tue mich dann leichter. 😉

  201. #202 MartinB
    10. Februar 2023

    @Karl-Heinz
    Die neue Antwort ist ja eher schlimmer. In einer lokal begrenzten Umgebung ist die Krümmung immer vernachlässigbar, das ist genau der Kern des Äq-Prinzips. Das gilt im Bereich eines Schwarzen Lochs genau wie auf der Erdoberfläche (beim SL ist die Umgebung halt nur kleiner). Das könnte man als Pingelig abtun, aber nicht, wenn es um die Frage der lokalisierung der Gravitationsenergie in der Raumzeit angeht – egal auf welchen Punkt ChatGPT zeigt, an dem die Energie gespeichert sein soll, ich finde immer ein Bezugssystem, indem die RZ hier perfekt flach ist. Und da über das, was am selben Punkt zur selben zeit passiert, alle Beobachterinnen Einig sein müssen, kann man so zeigen, dass man die Energie des Gravitationsfledes nicht lokalisieren kann. Ist verrückt, ist aber so.

    In derzweiten Antwort sagt er das dann ja auch selbst,a ber den Rückschluss auf die Energie bekommt er dann eben nicht hin.

    @Christian
    Zu Kaluza kann ich nichts sagen – aber in der ART folgt ein Teilchen nun mal einer Geodäte, und wenn die Raumzeit gekrümmt ist, ändert sich dabei der Impuls und die Energie (in einem bestimmten Bezugssystem gemessen).

    “Aus dem bloßen Wellencharakter von Licht lässt sich doch keine Energie- und Impulsübertragung ableiten, oder?”
    Doch klar. In der Maxwell-Theorie ist die Energiedichte E²+c²B², der Impulsübertrag auf geladene Teilchen lässt sich da auch über die Lorentzkraft leicht nachvollziehen.

    “Hat sich Einstein überhaupt Gedanken gemacht, was denn die Raumzeit sein und wie eine Verwellung der Raumzeit funktionieren soll? ”
    Was meinst du mit funktionieren? Die Raumzeit ist gekrümmt (wenn man diese Interpretation der ART verwendet), man kann das mathematisch beschreiben und damit nahezu perfekte Vorhersagen treffen. “Funktionieren” impliziert ja, dass du dir ein weiteres Modell vorstellst, das darunter liegt, aber das führt am Ende auch zu einem unendlichen Regress, darüber habe ich hier auch schon ab und zu mal gebloggt. Irgendwann ist Schluss mit “Erklären” und man kann nur noch beschreiben, was ist. Versuch mal in irgendeinem Modell (Newton, ART, egal) zu “erklären” was es bedeutet, dass ein Objekt an einem bestimmten Ort ist, dann merkst du vielleicht, was ich meine.

    (Siehe dazu und überhaupt zu allem hier auch mein Buch Isaac…)

  202. #203 Christian
    Wien
    10. Februar 2023

    Also Martin, mit “Verwellen der Raumzeit” meine ich Gravitationswellen. Da denke ich, dass es Unterschiede zu einer bloßen Krümmung gibt. Und ich frage mich eben, wie Gravitationswellen auftreten können, wenn die Raumzeit irgendetwas Nichtkörniges sein soll und wenn es keine Gravitonen gibt beziehungsweise diese entbehrlich sind, wie das die Relativitätstheorie postuliert. Deine Animationen oben bedienen sich ja auch schwingender Teilchen.

    Martin: “In der Maxwell-Theorie ist die Energiedichte E²+c²B², der Impulsübertrag auf geladene Teilchen lässt sich da auch über die Lorentzkraft leicht nachvollziehen.”

    Dann nehme ich noch einmal den Compton-Effekt als konkretes Beispiel: Lässt er sich alternativ ausschließlich über die Welleneigenschaft von elektromagnetischer Strahlung deuten oder nicht?

  203. #204 MartinB
    10. Februar 2023

    @Christian
    “Da denke ich, dass es Unterschiede zu einer bloßen Krümmung gibt. ”
    Ist sicher ne Definitionsfrage oder eine Frage, was du dir unter Krümmung vorstellst, aber im Sinne der ART nein. GW sind eine Form der Raumkrümmung.

    Der Comptoon-Effekt ist natürlich uantenmechanisch, oder besser gesagt, quantenfeldtheoretisch – da gibt es keine strenge Trennung zwischen Wellen und Teilchen mehr (gibt es eigentlich schon i der Qm nicht mehr wirklich, aber in der QFT schon gar nicht). Deswegen gibt es sogar einen Artikel mit dem Titel “There are no particles, there are only fields”.

    Ich habe sowohl bei Welle/Teilchen als auch bei dem Begriff Krümmung das Gefühl, dass du eine zu einfache Anschauung auf Phänomene überstülpst. Das führt (fast) immer in die Irre.

  204. #205 Christian
    Wien
    10. Februar 2023

    Also ist der Welle-Teilchen-Dualismus in der Quantenfeldtheorie ein nicht mehr notwendiges Konstrukt?

  205. #206 Christian
    Wien
    11. Februar 2023

    Also ist der Welle-Teilchen-Dualismus in den Quantenfeldtheorien ein nicht mehr notwendiges Konstrukt? Genial. Mathematisch bewältigen diese Theorien wohl nur noch wenige Menschen.
    Da es aber nach wie vor keine Quantenfeldtheorie der Gravitation gibt, komme ich zu meinem Ausgangskommentar #191 und der darin enthaltenen Aussage zurück: “Wie würde also gemäß Kaluza die Energie- und Impulsübertragung durch Licht funktionieren? Wenn es darauf eine Antwort gibt, dann gäbe es wohl auch einen Analogieschluss hin zur Gravitation.”
    Die Energie- und Impulsübertragung zwischen elektromagnetischen Wellen und Materie wird also durch die Quantenelektrodynamik ohne den in sich widersprüchlichen Welle-Teilchen-Dualismus sehr gut erfasst. In Kaluzas fünfdimensionaler Raumzeit ist der Elektromagnetismus ein Effekt der Raumzeitgeometrie. Wenn es also eine Quantenfeldtheorie der elektromagnetischen Felder gibt, warum gelingt es dann angesichts Kaluzas um eine Raumrichtung erweiterten Relativitätstheorie nicht, den Analogieschluss hin zur Gravitation herzustellen?

  206. #207 MartinB
    11. Februar 2023

    “Also ist der Welle-Teilchen-Dualismus in den Quantenfeldtheorien ein nicht mehr notwendiges Konstrukt? ”
    Der Welle-teilchen-Dualismus ist in meinen Augen der Versuch, quantenmechanische Objekte, die immer wellen- und teilchencharakter haben, in unserer Umgangssprache und klassischen Anschauung zu fassen. Aber in Fachbüchern wirst du dazu meist nicht viel lesen, weil es tatsächlich für die detaillierte Berechnung kein hilfreiches Konzept ist.

    Zu Kaluza kann ich nichts sagen, weil ich mich damit nicht beschäftigt habe – aber soweit ich weiß, gibt es keine Kaluza-Quantenfeldtheorie, die Theorie ist doch letztlich rein klassisch, oder irre ich mich? Insofern verstehe ich nicht wirklich, warum du glaubst, dass man diese klassische Theorie als Basis für eine Quantengravitation nehmen kann…

    Im Übrigen – man muss die ART nicht als Theorie der gekrümmten Raumzeit auffassen, man kann sie auch als eine Feldtheorie wie die Elektrodynamik sehen. Siehe Isaac Kap. 20 oder diverse Artikel hier im Blog.

  207. #208 Christian
    Wien
    11. Februar 2023

    “Insofern verstehe ich nicht wirklich, warum du glaubst, dass man diese klassische Theorie als Basis für eine Quantengravitation nehmen kann…”

    Bei Kaluza wird zusätzlich zur Gravitation der Elektromagnetismus geometrisiert, wodurch Einsteins Raumzeit eine weitere raumartige Richtung hinzugefügt wird. Einerseits gibt es nach Kaluza also eine klassische Relativitätstheorie des Elektromagnetismus und andererseits gibt es mit der Quantenelektrodynamik eine funktionierende Quantenfeldtheorie des Elektromagnetismus.

    Falls Kaluzas Theorie Phänomene des Elektromagnetismus genauso zuverlässig beschreibt wie die Relativitätstheorie (und naturgemäß auch die Kaluza-Theorie) Phänomene der Gravitation (immerhin sind in den Kaluza-Gleichungen die Maxwell-Gleichungen als Spezialfall enthalten), dann haben wir sowohl eine relativistische Beschreibung als auch eine quantisierte Beschreibung des Elektromagnetismus. Also das, wonach bei der Gravitation schon seit rund 100 Jahren gesucht wird. Warum klappt es also in Anbetracht der Kaluza-Theorie nicht mit dem Analogieschluss vom Elektromagnetismus hin zur Gravitation, oder hat das bisher noch keiner versucht? (Ich nenne die Theorie mit Absicht nicht Kaluza-Klein-Theorie, weil ich Oskar Kleins Beitrag zur Kaluza-Theorie, nämlich die Kompaktifizierung einer Raumrichtung, auch angesichts der zahlreichen experimentellen Ergebnisse, die es mittlerweile zu postulierten eingerollten Extradimensionen gibt, für unwahrscheinlich halte.)

    Aber gut, in deinem obigen Beitrag von vor fast 8 Jahren schreibst du ja: “Wie die Quantengravitation der Gravitonen funktioniert (oder eben nicht) und wo die Schwierigkeiten dabei stecken, das verschiebe ich auf ein anderes Mal.”
    Hast du zwischenzeitlich schon etwas dazu geschrieben? Übrigens, das möchte ich auch einmal betonen: Danke, dass du, aber auch andere wie Alderamin und Karl-Heinz euer umfangreiches, fundiertes Wissen z. B. über solche Blogs wie diesen hier, der interessierten Allgemeinheit zur Verfügung stellt! Denn ich konnte immer wieder beobachten, dass sich gute Naturwissenschaftler einerseits gar nicht so selten in eine Blase zurückziehen und andererseits auch in Worten nicht gut ausdrücken können, so wie ihr drei es schon könnt. (Ohne Rückmeldungen von außerhalb der Blase besteht immer die Gefahr, dass jemand seine Gedanken nicht oder nicht ausreichend an der Wirklichkeit kalibriert.)

  208. #209 MartinB
    11. Februar 2023

    @Christia
    Aber das kann ja nicht funktionieren, weil wir doch wissen, dass Elektromagnetismus nur ein teil der Elektroschwachen Wechselwirkung ist.

    Man kann natürlich überlegen, ob man, wenn man sich ein hypothetisches Universum denkt, in dem es nur Quantenelektrodynamik (ohne den ganzen elektroschwachen Kram) und Gravitation wäre, irgendwie diese Verbindung hinbekommt, aber ob das wirklich zielführend ist, weiß ich nicht, und kenne mich dazu auch zu wenig mit Kaluza-Klein aus. Ich vermute aber, da landet man nur bei dem, was man schon weiß: Wenn man die Gravitation analog zur Elektrodynamik quantisieren will, kommt Unsinn heraus.

    Und ja, zur Quantengravitation hatte ich auch was geschrieben (und natürlich steht dazu auch was in meinem Buch, hint, hint).
    https://scienceblogs.de/hier-wohnen-drachen/2015/04/19/quantengravitation/

  209. #210 MartinB
    18. Februar 2023

    @Karl-Heinz
    Ich habe jetzt aus der ChatGPT-Antowrt einen Blogartikel gemacht, ich hoffe, das ist so o.k. für dich. (Kann dich aber auch gern namentlich erwähnen, wenn du das möchtest):
    https://scilogs.spektrum.de/hier-wohnen-drachen/chatgpt-versteht-die-relativitaetstheorie-nicht/

  210. #211 Karl-Heinz
    Graz
    18. Februar 2023

    @MartinB

    Oh, echt?
    Bin schon gespannt
    1) welche Einstellung du zu ChatGPT hast?
    2) welche Erfahrung du mit ChatGPT gemacht hast?
    3) welche Auswirkung ChatGPT auf die Uni hat?

    Danke für deine Mühe.

  211. #212 MartinB
    18. Februar 2023

    @Karl-Heinz
    Ich habe bisher keine Erfahrungen, immer wenn ich ChatGPT versuchte, war er überlastet. Inzwischen muss man glaube ich einen account machen, dazu hatte ich bisher nicht die Lust…